Dogs Flashcards

1
Q

A 6-year old male neutered terrier mix weighing 10kg presents to you after having ingested 3 pieces of sugar-free gum containing xylitol. What bloodwork abnormality are you expecting to find?
Hypernatremia
Hyperglycemia
Hypoglycemia
Hypocalcemia
Hyponatremia

A

Answer: Hypoglycrmia

Explanation
Xylitol is a sugar alcohol present in sugar-free gum that, when ingested, causes a rapid release of insulin in dogs.
After ingestion, clinically significant hypoglycemia can develop within 30 minutes and can last for more than 12 hours. Acute hepatic necrosis and failure after higher doses of xylitol ingestion can occur as well.

How well did you know this?
1
Not at all
2
3
4
5
Perfectly
2
Q

A 6-year old female spayed English Pointer presents for lethargy and weight gain. The owner notes that the dog is eating and drinking a normal amount, but the dog is still gaining weight. A physical exam reveals weak pelvic limbs, facial nerve paralysis, a symmetrically patchy haircoat, and seborrhea. Lab work reveals a normocytic, normochromic anemia with a PCV of 29% (35-57%), lipemic serum, and cholesterol of 1090 mg/dl (135-278 mg/dl). What is the most likely diagnosis?

Hypothyroidism
Hyperthyroidism
Pituitary dependent hyperadrenocorticism
Adrenal dependent hyperadrenocorticism

A

Answer: Hypothyroidism.

Explanation
The correct answer is hypothyroidism. The clinical signs, physical exam findings, and lab work abnormalities are classical for hypothyroidism. Other common abnormalities seen with hypothyroidism include pyoderma, neuromuscular signs (ataxia, knuckling, vestibular signs, etc), markedly elevated triglycerides, and a mild normocytic, normochromic anemia. Hyperthyroidism rarely occurs in dogs. You would expect to see polyuria, polydipsia, and polyphagia with hyperadrenocorticism.

How well did you know this?
1
Not at all
2
3
4
5
Perfectly
3
Q

A 1-year old German Shepherd Dog presents for its first physical exam. Cardiac auscultation reveals a continuous murmur. Thoracic radiographs and echocardiography lead you to the diagnosis of a very large, cylinder-shaped patent ductus arteriosus with left-to-right shunting of blood. Which of the following statements is most true?
Furosemide and enalapril are the most appropriate forms of treatment for a cylinder-shaped PDA
Coil embolization is the most appropriate form of treatment for a cylinder-shaped PDA
Surgical ligation is the most appropriate form
of treatment for a cylinder-shaped PDA
Placement of constrictor ring is the most appropriate form of treatment for a cylinder-shaped PDA

A

Answer: Surgical ligation is the most appropriate form
of treatment for a cylinder-shaped PDA

The correct answer is surgical ligation of the PDA is the most appropriate form of treatment. A large cylindrical PDA is unlikely amenable to closure via traditional coil embolization. Most canine PDAs are funnel-shaped with the smaller end of the funnel attached to the pulmonary artery, and the wider end attached to the aorta. The funnel shape is what allows the coils to lodge in place and cause closure of the PDA. Coils would not stay in place in a large, cylindrical PDA, so surgical ligation is the treatment of choice for this patient. There are newer nickel-titanium devices (Amplatz Canine Ductal Occluder) that show promise for minimally-invasive closure of PDAs which do not taper, but this answer was not provided. Ameroid constrictor rings are typically used for ligating hepatic shunts, and not for PDAs. Medical management with diuretics and heart drugs are only indicated prior to correction of the PDA if the dog is in heart failure.

How well did you know this?
1
Not at all
2
3
4
5
Perfectly
4
Q

A 12 year old female spayed Spaniel mix presents for progressive lethargy over the last week. This morning, the owner noticed the dog was having a hard time breathing and had pale gums. A blood smear from the dog is shown below.
The larger arrow is pointing to which type of cell?

Nucleated red blood cell
Monocyte
Lymphocyte
Myelocyte

A

Answer: Nucleated red blood cell

The larger arrow is pointing to a nucleated red blood. These are immature erythrocytes that have been released early from the bone marrow in response to the dog’s anemia and increased need for oxygen carrying capacity.
These cells are typically larger than a mature erythrocyte, have a very dark purple nucleus, and have basophilic cytoplasm. There are 3 nucleated red blood cells in this picture.
Monocytes are essentially macrophages found in the blood whose function is phagocytosis. They have a blue-gray cytoplasm and a lobulated or bean shaped nucleus.
Lymphocytes are typically about the size of a red blood cell and are mostly made up of a nucleus with just a slender rim of cytoplasm. They are involved with antibody production as well as cell mediated immunity.
Myelocytes are immature granulocytes that should normally only be found in the bone marrow.

How well did you know this?
1
Not at all
2
3
4
5
Perfectly
5
Q

A 12-year-old male intact English bulldog presents to your hospital for a drop of blood noted coming from the penis. The dog has no other prior medical history. On examination, both testicles are descended and palpate normal in size. A rectal exam reveals a large, but symmetrical prostate gland. You extrude the penis and note what is illustrated in the image below. What is your recommendation to the owner?

There is an abscess of the penile tip, likely of bacterial origin. We should begin antibiotic therapy once we obtain a culture.
This is a urethral prolapse and can be repaired by performing a urethropexy.
There is a mass at the tip of the penis, which will likely respond to radiation therapy.
There is a mass at the tip of the penis suspicious for a mast cell tumor. You should amputate the tip and submit it for histopathology.

A

Answer: This is a urethral prolapse and can be repaired by performing a urethropexy.

The image is classic for a urethral prolapse. This condition is almost exclusive to young English bulldogs. It is interesting that this patient was incidentally 12 years of age. The cause of urethral prolapse is not always determined but may be secondary to excessive masturbation, sexual excitement, or an infection. Treatment involves either urethropexy to replace the prolapse or amputation of the tip. The patient should be examined closely to ensure there is no evidence of concurrent neoplasia or infection.

How well did you know this?
1
Not at all
2
3
4
5
Perfectly
6
Q

Lucy is an 11 year-old female spayed Labrador retriever presenting in acute respiratory distress after playing fetch on a hot summer day. On initial exam, Lucy is panting heavily and a loud stridor is audible. Upon further history-taking the owners tell you that Lucy’s bark has changed over the past couple of months. What is the most common complication/side effect of the recommended treatment for Lucy’s condition?

Drooling
Megaesophagus
Aspiration pneumonia
Tracheal collapse
Kidney disease

A

Answer: Aspiration Neumonia.

Explanation
Lucy’s presentation is typical for a dog with laryngeal paralysis. The arytenoid cartilages fail to abduct during inspiration, resulting in upper airway obstruction. These patients often present with acute clinical signs even though the disease is chronic in nature. Surgical arytenoid lateralization (tie-back) is the treatment of choice but with the chronically opened airway, aspiration pneumonia can be a common side effect. If surgery is not an option, prednisone can be given to help reduce laryngeal inflammation.

How well did you know this?
1
Not at all
2
3
4
5
Perfectly
7
Q

A 2-year-old neutered male greyhound presents for crusting along his ear pinnae and elbows. The owner also explains that the dog is intensely pruritic. The dog mainly lives indoors but is also let outside into the backyard. The owner lives on a large rural property that is heavily wooded. You perform cytology, which returns negative for bacteria and yeast. You are highly suspicious of Sarcoptes. Which of the following would be most useful to help in your diagnosis of Sarcoptic mange?

Superficial skin scrape, histopathology, trichogram
Deep skin scrape, pinnal-pedal reflex, clinical signs
Histopathology, deep skin scrape, trichogram
Pinnal-pedal reflex, clinical signs, superficial skin scrape
Clinical signs, superficial skin scrape, trichogram

A

Answer: Pinnal-pedal reflex, clinical signs, superficial skin scrape

A superficial skin scrape is most useful. A deep skin scrape is used for Demodex mites, which reside in the hair follicles. Superficial skin scraping is indicated for Sarcoptic mange, since this mite lives in the stratum corneum.

Pinnal-pedal reflex can be positive in 70% of Sarcoptic mange infestation but can also be positive in any dog with pruritus. Clinical signs are helpful because Sarcoptes typically infects the non-haired portions of the skin such as the ear pinnae, elbows, hocks, and ventral abdomen. If the mite is not found on skin scraping and you have clinical signs indicative of Sarcoptes then a therapeutic trial is warranted. Keep in mind that in most cases you will not be able to find the mite.

A trichogram would not be indicated. This method can be used to diagnose dermatophytosis and sometimes Demodex. Histopathology will reveal a perivascular dermatitis that is non-specific. Rarely are the mites found on histopathology.

How well did you know this?
1
Not at all
2
3
4
5
Perfectly
8
Q

A 10 year old female spayed Schipperke dog presents to your emergency clinic for profuse vomiting for several days and weakness. You perform initial bloodwork and find that her blood pH is 7.6 (7.35-7.45) and her potassium is 1.8 mmol/L (3.4-4.9 mmol/L). She weighs 12 kg. You immediately start her on intravenous fluids supplemented with potassium. What is the maximum rate of intravenous potassium that would be considered safe to administer to this dog?

6 mEq/hr
12 mEq/hr
24 mEq/hr
18 mEq/hr
3 mEq/hr

A

Answer: 6mEq/hr

Explanation
The maximum safe rate of potassium infusion is 0.5 mEq/kg/hr. As this dog weighs 12 kg, the maximum is 6 mEq/hr. This is one of those rates that you need to know. Administering potassium more rapidly than this can result in fatal arrhythmias.

How well did you know this?
1
Not at all
2
3
4
5
Perfectly
9
Q

The best diagnostic test to differentiate pituitary dependent hyperadrenocorticism from adrenal dependent hyperadrenocorticism in dogs is which of the following?

Urine cortisol: creatinine ratio
Serum cortisol
Low-dose dexamethasone suppression test
High-dose dexamethasone suppression test

A

Answer: High-dose dexamethasone suppression test

The correct answer is high-dose dexamethasone suppression test. HDDST will suppress cortisol secretion in about 75% of PDH patients 3-6 hours post dexamethasone administration. Cortisol secretion does not become suppressed with dexamethasone administration with adrenal dependent hyperadrenocorticism. If cortisol secretion is suppressed with a HDDST, PDH is diagnosed. If cortisol is not suppressed, there is a
50-50 chance that the hyperadrenocorticism is due to PDH or an adrenal tumor.
LDDST will suppress cortisol secretion in approximately 65% of pituitary dependent hyperadrenocorticism patients. The urine cortisol:creatinine ratio measures the significance of urine cortisol concentration and may be increased with both PDH and ADH. Serum cortisol levels may be affected in many different situations such as stress; therefore they are an unreliable indicator of disease. An elevated serum cortisol level will not differentiate between PDH and ADH.

How well did you know this?
1
Not at all
2
3
4
5
Perfectly
10
Q

Which is a cause of primary bacterial pneumonia in dogs?

Bacillus spp.
Bordetella bronchisepica
Pasteurella multocida
Pseudomonas spp.

A

Answer: Bordetella bronchiseptica.

Explanation
Although not common, Bordetella infections can develop into a primary pneumonia, particularly if the dog has a weakened immune system. The other choices require an underlying problem such as aspiration, foreign body, viral infection, neoplasia, etc. to be present in order to create an infection.
Secondary bacterial infections can be extremely severe and need to be diagnosed and treated appropriately and aggressively.

How well did you know this?
1
Not at all
2
3
4
5
Perfectly
11
Q

Which of the following is the most likely side effect of KBr therapy in epileptic dogs?

Sedation
Pancreatitis
Vomiting
Polyuria and polydipsia

A

Answer: Sedation.

Explanation
The correct answer is sedation. Possible side effects of KBr therapy include polyphagia, polyuria and polydipsia, ataxia, skin reactions, pancreatitis, and vomiting. However, sedation is more likely to occur than these.

How well did you know this?
1
Not at all
2
3
4
5
Perfectly
12
Q

A 7 year old Poodle presents for a left sided head tilt, a ventral strabismus on the left, and right horizontal nystagmus. The dog is obtunded, has conscious proprioception deficits in both right limbs, and a right sided facial paralysis. Where is the lesion?

Peripheral nerve on the right
Central nervous system on the right
Central nervous system on the left
Peripheral nerve on the left

A

Answer: Central nervous system on the right

Explanation
The correct answer is central nervous system on the right. This dog has paradoxical vestibular disease. This disease is usually caused by a destructive lesion in the CNS in which there is loss of inhibition of the vestibular output on the side of the lesion. There is therefore a relative increase in vestibular output on the side of the lesion, causing vestibular signs on the side opposite the lesion (yes, this can be confusing). In this case, the right sided lesion has vestibular signs that would normally localize the lesion to the left side. When the lesion causes vestibular signs and proprioceptive deficits, the lesion is ipsilateral to the side with the proprioceptive deficits.

How well did you know this?
1
Not at all
2
3
4
5
Perfectly
13
Q

A 9-year old female spayed German Shepherd presented for extreme lethargy. Physical examination and diagnostics were consistent with a hemoabdomen.
The patient was taken to surgery and a splenic mass was found (see image). The remainder of your abdominal exploration was unremarkable. What is the most common malignant tumor of the spleen in the dog?

Transitional cell carcinoma
Hemangiosarcoma
Lymphoma
Mast cell tumor

A

Answer: Hemangiosarcoma

The correct answer is hemangiosarcoma. This tumor accounts for approximately 2/3 of malignant splenic masses and will have metastasized in >90% of cases by the time they are diagnosed. Chemotherapy would be recommended for adjunct therapy after surgical removal of the spleen. Lymphoma is not as commonly found in the spleen in dogs, and you would have expected enlarged lymph nodes to be mentioned in the question. Mast cell tumors occur with some frequency in the spleen of cats but not so often in dogs. Transitional cell carcinomas are typically found in the urinary bladder, since that is where transitional cells are located.

How well did you know this?
1
Not at all
2
3
4
5
Perfectly
14
Q

A 3-year old Husky bitch comes to your clinic. The owner tells you that the bitch is in heat and has been receptive to males. Which of the following would you expect to see on vaginal cytology?

B
D
C
A

A

Answer: C

Dogs in estrus should have cellular vaginal cytology consisting predominantly (>90%) of cornified epithelial cells. This corresponds to image
C.
Image A is from a dog in anestrus, note that it is less cellular. Image B is from a dog in early diestrus.
Image D is from a dog in diestrus.

How well did you know this?
1
Not at all
2
3
4
5
Perfectly
15
Q

On routine physical exam, you find Dipylidium caninum segments on the perianal region of a dog. Which of the following assumptions can be made?

The dog was infected in utero by transplacental transmission
The dog acquired the infection by coprophagia
The dog acquired the infection by eating uncooked beef
The dog should be treated for fleas and
tapeworms

A

Answer: The dog should be treated for fleas and tapeworms

Explanation
The correct answer is the dog should be treated for fleas and tapeworms. Fleas are a required intermediate host of Dipylidium caninum. Dogs become infected by swallowing fleas that carry the tapeworm eggs. Finding proglottids of this tapeworm in the feces or perianally indicates the presence of fleas on the dog.

How well did you know this?
1
Not at all
2
3
4
5
Perfectly
16
Q

Which of the following is an appropriate objective in the repair of an articular fracture in a dog?
6-8 weeks of strict post-operative rest
Early return to function
Avoid rigid fixation of the fracture
50% or greater alignment

A

Answer: Early return to function.

Explanation
The correct answer is early return to function. With articular fractures, the 3 major goals of the clinician are:
Rigid fixation of the fracture fragments; neglecting to do this will result in loose fracture fragments within the joints that will promote osteoarthritis.

Anatomic realignment (50% or even 75% is not adequate with articular fractures, in contrast to long bone fractures)
Early return to function. With long bone fractures, extended rest is recommended to promote stability and healing of the bone. The opposite is true in articular fractures where prolonged rest after repair will promote fibrosis, causing decreased range of motion in the joint.

How well did you know this?
1
Not at all
2
3
4
5
Perfectly
17
Q

Which digit has been partially amputated in the picture below? (hint: this is the right forelimb of a dog)

5th digit
3rd digit
4th digit
1st digit
2nd digit

A

Answer: 3rd digit

Explanation
The correct answer is 3rd digit. Remember that a dog’s digits are numbered from 1 to 5, starting medially with the small 1st metacarpal that is associated with the dewclaw (essentially like the thumb on a primate). This digit is not well visible in the photograph, but you should still be able to see that it is the third digit that has had the ungual crest and process amputated.

How well did you know this?
1
Not at all
2
3
4
5
Perfectly
18
Q

You are treating an 8-year old mix breed terrier for ventricular tachycardia post splenectomy. You determine that you’ve given too much lidocaine to the patient. What clinical signs or laboratory findings are you most likely to see initially?

Muscle tremors and seizures
Bradycardia and hypotension
Increased respiratory rate and difficulty breathing
Methemoglobinemia
Diarrhea

A

Answer: Muscle tremors and seizures.

Explanation
Early clinical signs associated with lidocaine toxicity include neurological symptoms such as seizures and tremors. Nausea and vomiting may occur, but is usually transient. Cardiovascular and respiratory depression can also occur, but usually later on in the course of clinical signs.

Lidocaine toxicity in an 8-year-old mixed breed terrier initially presents with neurological symptoms like muscle tremors and seizures. While nausea and vomiting may occur, these are usually transient. Cardiovascular and respiratory depression are possible but typically occur later in the clinical course. Early recognition and intervention are critical to manage toxicity effectively.

How well did you know this?
1
Not at all
2
3
4
5
Perfectly
19
Q

You are treating an 8-year old mix breed terrier for ventricular tachycardia post splenectomy. You determine that you’ve given too much lidocaine to the patient. What clinical signs or laboratory findings are you most likely to see initially?

Muscle tremors and seizures
Bradycardia and hypotension
Increased respiratory rate and difficulty breathing
Methemoglobinemia
Diarrhea

A

Answer: Muscle tremors and seizures.

Explanation
Early clinical signs associated with lidocaine toxicity include neurological symptoms such as seizures and tremors. Nausea and vomiting may occur, but is usually transient. Cardiovascular and respiratory depression can also occur, but usually later on in the course of clinical signs.

Lidocaine toxicity in an 8-year-old mixed breed terrier initially presents with neurological symptoms like muscle tremors and seizures. While nausea and vomiting may occur, these are usually transient. Cardiovascular and respiratory depression are possible but typically occur later in the clinical course. Early recognition and intervention are critical to manage toxicity effectively.

How well did you know this?
1
Not at all
2
3
4
5
Perfectly
20
Q

A recently adopted 5-month-old female spayed mixed breed dog presents for vaccines, and you auscult a V/VI murmur at the left heart base. It is a very loud continuous murmur, which sounds like a washing machine. The owner reports normal activity, energy, and appetite. Her gums are pink with good refill. She has strong synchronous pulses. Based on your assessment, you most suspect which of the following?

Patent ductus arteriosus
Heartworm infection
Physiologic murmur
Atrial septal defect

A

Answer: Patent ductus arteriosus

Explanation
A patent ductus arteriosus typically is associated with a loud, continuous murmur that may be ausculted at the left heart base/axillary region. The sound is sometimes described as sounding like a “washing machine.” Subaortic and pulmonic stenosis also typically have loud systolic murmurs heard best at the left heart base.

An atrial septal defect is not typically associated with an intense murmur. More commonly, the murmur of ASD is Il-III/VI and is located at the left heart base due to increased flow across the pulmonary valve because of left-to-right shunting. (Note, you do not hear the flow across the atrial septum because it is of very low velocity).

The life cycle of the heartworm is approximately 6 months (this patient is only 5 months old), and heartworm does not typically cause this type of murmur. Physiologic murmurs are typically a grade I-II/VI, and are low intensity.

A patent ductus arteriosus (PDA) is likely in this 5-month-old dog, given the loud, continuous “washing machine” murmur at the left heart base. PDA is associated with such murmurs, unlike atrial septal defects, which usually have softer murmurs. Heartworm infection is improbable due to the dog’s age, and physiologic murmurs are typically low-intensity.

How well did you know this?
1
Not at all
2
3
4
5
Perfectly
21
Q

All of the following drugs are appropriate in the management of tachycardia associated with this arrhythmia in small animal patients except:

Isoproterenol
Atenolol
Digoxin
Procainamide
Diltiazem

A

Answer: Isoproterenol

This ECG shows atrial fibrillation. Atenolol is a beta-blocker and will slow AV nodal conduction to decrease the ventricular response rate to atrial fibrillation. Procainamide is a class 1A anti-arrhythmic and can be used in attempts to convert atrial fibrillation to a normal sinus rhythm, though it is rarely effective. Diltiazem is a calcium channel blocker and can be used to slow AV nodal conduction and ventricular response rate. Digoxin will increase vagal tone to the AV node to slow conduction and decrease the heart rate. Isoproterenol is a nonspecific beta receptor agonist, and will increase the ventricular rate in response to atrial fibrillation, thus is inappropriate in the management of this dysrhythmia.

Isoproterenol is inappropriate for managing atrial fibrillation in small animals as it increases ventricular rate. Atenolol (beta-blocker), Digoxin (increases vagal tone), and Diltiazem (calcium channel blocker) slow AV nodal conduction, decreasing the ventricular response rate. Procainamide (class 1A anti-arrhythmic) attempts to convert atrial fibrillation to sinus rhythm, though rarely effective.

How well did you know this?
1
Not at all
2
3
4
5
Perfectly
22
Q

A 10-year old male castrated Collie presents for what the owner calls heart failure. Which of the following is not a typical sign of right heart failure?

Exercise intolerance
Muscle wasting
Hepatomegaly
Tachypnea
Cough

A

Answer: Cough

Explanation
The correct answer is cough. Coughing is typically a sign of left heart failure that occurs with the development of pulmonary edema and/or bronchial compression from an enlarged left atrium. In right heart failure, tachypnea and dyspnea can develop due to pleural effusion. Hepatomegaly occurs due to venous congestion from the damming of blood into the liver. Exercise intolerance develops because a decrease in cardiac output causes a decrease in oxygenation of tissues. Muscle wasting occurs from loss of protein into effusions and from hepatic and Gl malfunction. Other signs of right heart failure include lethargy, weakness, venous distention, ascites, and peripheral edema.

Cough is not typical of right heart failure; it’s usually associated with left heart failure due to pulmonary edema or bronchial compression. Right heart failure signs include tachypnea, hepatomegaly, exercise intolerance, muscle wasting, lethargy, weakness, venous distention, ascites, and peripheral edema. These result from pleural effusion, venous congestion, and decreased cardiac output.

How well did you know this?
1
Not at all
2
3
4
5
Perfectly
23
Q

Which of the following is a non-adrenergic vasoconstrictor that can be used in dogs?

Dobutamine
Isoproterenol
Vasopressin
Epinephrine
Phenylephrine

A

Answer: Vasopressin

Explanation
The correct answer is vasopressin. Epinephrine and phenylephrine are adrenergic vasoconstrictors.

Isoproterenol and dobutamine cause vasodilation. Vasopressin (also known as anti-diuretic hormone) is a V-1 receptor, non-adrenergic, vasoconstrictor.

Vasopressin is a non-adrenergic vasoconstrictor used in dogs. Unlike epinephrine and phenylephrine, which are adrenergic vasoconstrictors, and isoproterenol and dobutamine, which cause vasodilation, vasopressin acts on V-1 receptors to induce vasoconstriction.

How well did you know this?
1
Not at all
2
3
4
5
Perfectly
24
Q

Dilated cardiomyopathy is a common disease seen in dogs. Which one of the following medications is indicated for the treatment of DCM?

Atropine
Prednisone
Enalapril
Adriamycin

A

Answer: Enalapril

Explanation
The correct answer is enalapril. All others would be contraindicated. Enalapril is an ACE-inhibitor, which blunts the adverse effects of the renin-agiotensin-aldosterone system thereby reducing sodium and fluid retention.

Additionally, ACE-inhibitors cause mild vasodilation by preventing the production of angiotensin-ll, which reduces ventricular afterload. Lastly, enalapril and other ACE-inhibitors are protective to cardiac muscle by blunting aldosterone and other hormones which induce cardiac remodeling.

How well did you know this?
1
Not at all
2
3
4
5
Perfectly
25
Q

An 8-year old male castrated Labrador Retriever presents for collapsing episodes. You record an ECG, which is shown here. What is your diagnosis?

Complete heart block
Sinus arrest
Normal sinus rhythm
Atrial premature complexes
Ventricular premature complexes

A

Answer: Complete heart block

The correct answer is complete heart block. The ECG shows frequent P waves and two ventricular escape beats from different foci. Complete heart block typically results from degeneration and fibrosis of the cardiac conduction system. The animal then relies on a slow escape rate, which often results in clinical symptoms of weakness and collapse. Sinus arrest would be seen as periods of flat line on the ECG with no atrial activity; normal sinus rhythm would show conduction of the P waves with QRS complexes following. Ventricular premature complexes is incorrect because the above 2 ventricular beats are escapes, they are not premature.

Atrial premature complexes is wrong as the atrial rhythm is normal.

How well did you know this?
1
Not at all
2
3
4
5
Perfectly
26
Q

Which of the following is the treatment of choice for pulmonic stenosis in the dog?

Valvulotomy by catheterization device
Balloon valvuloplasty
Surgical valvulotomy
Patch grafting of the outflow tract

A

Answer: Balloon valvuloplasty

Explanation
The correct answer is balloon valvuloplasty. This procedure is performed by inserting a catheter into either a jugular vein or a femoral vein and advancing it into the region of stenosis at the pulmonic outflow tract. Once in place, a balloon surrounding the catheter is inflated to stretch the stenotic region open. Surgical valvulotomy or patch grafting is the treatment used for severe cases that cannot be balloon-dilated.

Balloon valvuloplasty is the treatment of choice for pulmonic stenosis in dogs. It involves inserting a catheter through the jugular or femoral vein to the pulmonic outflow tract and inflating a balloon to stretch the stenotic region. Surgical valvulotomy or patch grafting is reserved for severe cases unresponsive to balloon dilation.

How well did you know this?
1
Not at all
2
3
4
5
Perfectly
27
Q

A 4-year old male Australian Shepherd is presented to your clinic for treatment of heartworm disease. Which of the following statements about heartworm disease is true?

strict activity restriction of the dog is very important once medical treatment is initiated
Doxycycline can be used to treat heartworms
Ivermectin is the treatment of choice for adult worms
The owners should be informed that dogs often can develop right heart failure post treatment even if the dog was clinically normal pre treatment

A

Answer: strict activity restriction of the dog is very important once medical treatment is initiated

Explanation
The correct answer is strict activity restriction of the dog is very important once treatment is initiated. Dogs should be restricted in activity at least 4-6 weeks post treatment to decrease the likelihood of thromboembolic events and to allow the lungs to recover from injury due to worm death. Dogs are not as likely to develop right heart failure after treatment of heartworm disease if they were previously clinically normal. Ivermectin is often used to prevent heartworm disease and for treating microfilariae, but the treatment of choice for adult worms is melarsomine IM. Doxycycline is often used to treat the Wolbachia bacteria that are often found with heartworms, but is ineffective at treating the worms themselves.

Strict activity restriction is crucial once heartworm treatment starts, to reduce thromboembolic events and allow lung recovery post worm death. Dogs clinically normal pre-treatment rarely develop right heart failure afterward. Ivermectin prevents heartworm and treats microfilariae, but melarsomine IM treats adult worms. Doxycycline targets Wolbachia bacteria associated with heartworms, not the worms themselves.

How well did you know this?
1
Not at all
2
3
4
5
Perfectly
28
Q

Which of the following is a non-adrenergic vasoconstrictor that can be used in dogs?
Epinephrine
Phenylephrine
Vasopressin
Dobutamine
Isoproterenol

A

Answer: Vasopressin.

Explanation
The correct answer is vasopressin. Epinephrine and phenylephrine are adrenergic vasoconstrictors.

Isoproterenol and dobutamine cause vasodilation. Vasopressin (also known as anti-diuretic hormone) is a V-1 receptor, non-adrenergic, vasoconstrictor.

How well did you know this?
1
Not at all
2
3
4
5
Perfectly
29
Q

Which of the following is the most common treatment for aortic stenosis in a dog?

Surgical repair
Medical management with beta blockers
Medical management with furosemide
Balloon dilation of the stenotic region by catheterization

A

Answer: Medical management with Beta-Blockers

Explanation
The correct answer is medical management with beta blockers. Beta blockers such as atenolol are often used to reduce myocardial oxygen demand, thus reducing the frequency of ventricular arrhythmias. The owners should also be warned that the dog will need prophylactic antibiotics for any situation in which the dog may develop a bacteremia, since it is at high risk of developing infective endocarditis. Balloon dilation by catheterization can be attempted in some patients, but the results are usually no better than medical therapy with beta blockers.

Surgical repair is difficult, expensive, and does not give very good results either. There is no indication for furosemide unless the dog is in heart failure.

Medical management with beta blockers, such as atenolol, is the most common treatment for aortic stenosis in dogs. Beta blockers reduce myocardial oxygen demand and ventricular arrhythmias. Prophylactic antibiotics are needed to prevent infective endocarditis. Balloon dilation offers no better results than beta blockers, and surgical repair is challenging and less effective. Furosemide is only indicated for heart failure.

How well did you know this?
1
Not at all
2
3
4
5
Perfectly
30
Q

A 1.5-year old male castrated Boxer presents for exercise intolerance and syncopal episodes. Cardiac auscultation reveals a grade 4/6 left systolic murmur heard best at the base of the heart. Thoracic radiographs show a mildly enlarged left ventricle and left atrium, and segmental enlargement of the ascending aorta. What test will best help you to confirm your tentative diagnosis?

Heartworm antigen test
CT scan
24 hour Holter ECG monitor
Coombs test
Echocardiogram

A

Answer: Echocardiogram

The breed, clinical history of syncope, radiographic findings, and auscultation findings are most consistent with aortic stenosis. Echocardiography would be the best method for diagnosing the disease in which you would find left ventricular concentric hypertrophy, stenosis of the aortic outflow tract, and post-stenotic dilatation of the ascending aorta.

A Holter monitor is the test of choice for arrhythmogenic right ventricular cardiomyopathy (ARVC) which is also seen in Boxers and causes syncope but does not fit with the physical exam or radiographic findings. Also called Boxer cardiomyopathy, ARVC is a disease associated with fibrofatty infiltrate into the right ventricle resulting in electrical conduction disturbances and resultant VPCs.

A CT scan would not add more information, and cardiac motion typically causes resolution problems on thoracic CT scans. The signs are not consistent with heartworm, which is primarily a right heart problem, or with hemolytic anemia, which a Coombs’ test might be useful for.

How well did you know this?
1
Not at all
2
3
4
5
Perfectly
31
Q

A 2 year old male castrated Boxer presents for exercise intolerance and syncopal episodes. Cardiac auscultation reveals a 4/6 left systolic murmur heard best at the base of the heart. Thoracic radiographs show a mildly enlarged left ventricle and left atrium, and segmental enlargement of the ascending aorta. What is the most likely diagnosis?

Pulmonic stenosis
Aortic stenosis
Myxomatous degeneration of the mitral valve
Boxer cardiomyopathy

A

Answer: Aortic stenosis.

Explanation
The correct answer is aortic stenosis. The breed, clinical history of syncope, radiographic findings, and auscultation findings are most consistent with aortic stenosis. Echocardiography would be the best method for diagnosing the disease in which you would find left ventricular hypertrophy, stenosis of the aortic outflow tract, and post-stenotic dilatation of the aorta.

Pulmonic stenosis would result in changes in the right heart and pulmonary outflow tract. Myxomatous degeneration of the mitral valve typically occurs in older animals, results in a murmur heard best at the left apex, and would not cause a dilation of the ascending aorta.

Boxer cardiomyopathy (or arrhythmogenic right ventricular cardiomyopathy) is principally an electrical disease causing frequent VPCs to occur. Dilation of the heart may occur in 10-15% of cases. The best way to diagnose Boxer cardiomyopathy is with a 24 hour cardiac Holter monitor.

The most likely diagnosis is aortic stenosis. The breed, syncope, radiographic findings (mildly enlarged left ventricle and atrium, segmental enlargement of the ascending aorta), and auscultation (4/6 left systolic murmur at heart base) support this. Echocardiography reveals left ventricular hypertrophy, aortic outflow tract stenosis, and post-stenotic aorta dilation. Pulmonic stenosis affects the right heart, myxomatous mitral valve degeneration affects older dogs with a murmur at the left apex, and Boxer cardiomyopathy involves frequent VPCs and is diagnosed with a Holter monitor.

How well did you know this?
1
Not at all
2
3
4
5
Perfectly
32
Q

A 9 month old shepherd cross presents for lethargy and exercise intolerance. Your physical exam reveals dark red mucous membranes and a heart murmur. A CBC reveals a hematocrit of 70% (35-57%). Which of the following could NOT explain the given findings?

Atrial septal defect with tricuspid stenosis
PDA with pulmonary hypertension
Atrial septal defect with systemic hypertension
Tetralogy of Fallot

A

Answer: Atrial septal defect with tricuspid stenosis

Explanation
The correct answer is atrial septal defect with systemic hypertension. This dog’s polycythemia developed due to shunting of venous blood into the arterial circulation (aka right-to-left shunting) leading to hypoxemia and an increased drive for erythropoiesis and secondary polycythemia. This can be caused by any of the other 3 answer choices. An atrial septal defect with systemic hypertension results in oxygenated blood shunting into the venous circulation or left-to-right shunting, which usually does not cause hypoxemia.

Atrial septal defect with systemic hypertension cannot explain the findings. This condition causes left-to-right shunting, leading to oxygenated blood entering the venous circulation, not hypoxemia. The dog’s polycythemia (hematocrit 70%) suggests right-to-left shunting, leading to hypoxemia and increased erythropoiesis. This can be caused by PDA with pulmonary hypertension, atrial septal defect with tricuspid stenosis, or Tetralogy of Fallot.

How well did you know this?
1
Not at all
2
3
4
5
Perfectly
33
Q

A 1-year old German Shepherd Dog presents for its first physical exam. Cardiac auscultation reveals a continuous murmur. Thoracic radiographs and echocardiography lead you to the diagnosis of a very large, cylinder-shaped patent ductus arteriosus with left-to-right shunting of blood. Which of the following statements is most true?

Placement of constrictor ring is the most appropriate form of treatment for a cylinder-shaped PDA
Coil embolization is the most appropriate form of treatment for a cylinder-shaped PDA
Furosemide and enalapril are the most appropriate forms of treatment for a cylinder-shaped PDA
Surgical ligation is the most appropriate form of treatment for a cylinder-shaped PDA

A

Answer: Surgical ligation is the most appropriate form of treatment for a cylinder-shaped PDA

Explanation
The correct answer is surgical ligation of the PDA is the most appropriate form of treatment. A large cylindrical PDA is unlikely amenable to closure via traditional coil embolization. Most canine PDAs are funnel-shaped with the smaller end of the funnel attached to the pulmonary artery, and the wider end attached to the aorta. The funnel shape is what allows the coils to lodge in place and cause closure of the PDA. Coils would not stay in place in a large, cylindrical PDA, so surgical ligation is the treatment of choice for this patient. There are newer nickel-titanium devices (Amplatz Canine Ductal Occluder) that show promise for minimally-invasive closure of PDAs which do not taper, but this answer was not provided. Ameroid constrictor rings are typically used for ligating hepatic shunts, and not for PDAs. Medical management with diuretics and heart drugs are only indicated prior to correction of the PDA if the dog is in heart failure.

Surgical ligation is the most appropriate treatment for a large, cylinder-shaped PDA. Traditional coil embolization is ineffective for cylindrical PDAs, as coils need a funnel shape to lodge properly. The newer Amplatz Canine Ductal Occluder might be effective but wasn’t an option. Ameroid constrictor rings are for hepatic shunts, and medical management with furosemide and enalapril is only for pre-correction heart failure.

How well did you know this?
1
Not at all
2
3
4
5
Perfectly
34
Q

Which of the following drugs and treatment intervals are appropriate for preventing heartworm disease in dogs?

Milbemycin twice daily
Ivermectin once a month
Milbemycin oxime once daily
Ivermectin once daily
Diethylcarbamazine once monthly

A

Answer: Ivermectin once a month.

Explanation
The correct answer is ivermectin once monthly. Diethylcarbamazine (DEC) can be used to prevent heartworm, but is administered once daily. Milbemycin oxime is another preventative used once monthly.

How well did you know this?
1
Not at all
2
3
4
5
Perfectly
35
Q

Which of the following has the least effect on heart rate?
Morphine
Ketamine
Propofol
Thiopental

A

Answer: Propofol.

Explanation
The correct answer is propofol. Ketamine causes an increase in heart rate and blood pressure. It is considered seizureogenic and has poor muscle relaxation effects, however it provides good analgesia. Thiopental is an ultra fast acting thiobarbiturate that causes an increased heart rate, increased blood pressure, and decreased cardiac contractility. It is also an arrhythmogenic agent. Morphine is an opioid that causes bradycardia. Propofol does not result in any significant cardiac changes. It is ultra-short in duration, can cause apnea and myoclonus, and is considered anticonvulsive. It may cause a mild bradycardia but you mainly see that when used in combination with other opiates. More recently propofol has been associated with myocardial depression and hypotension in critically ill dogs and may not be the induction agent of choice as a result. Despite these findings, propofol still has the least effect on heart rate.

Propofol has the least effect on heart rate. Ketamine increases heart rate and blood pressure, Thiopental increases heart rate and blood pressure but decreases cardiac contractility, and Morphine causes bradycardia. Propofol primarily causes mild bradycardia when combined with other opiates and has minimal cardiac impact, making it the best option regarding heart rate stability.

How well did you know this?
1
Not at all
2
3
4
5
Perfectly
36
Q

Which of the following is true about diltiazem?

Diltiazem slows the heart by antagonizing beta adrenergic receptors.
Diltiazem speeds up the heart because it is a beta adrenergic agonist.
Diltiazem slows the heart by inhibiting the influx of calcium into the myocardial cells.
Diltiazem speeds up the heart rate by increasing the influx of calcium into the myocardial cells.

A

Answer: Diltiazem slows the heart by inhibiting the influx of calcium into the myocardial cells.

Explanation
The correct answer is diltiazem slows the heart by inhibiting the influx of calcium into the myocardial cells.
Diltiazem is a calcium channel blocker used to slow the heart to treat supraventricular tachycardias, hypertrophic cardiomyopathy, and hypertension.

Diltiazem slows the heart by inhibiting the influx of calcium into myocardial cells. It is a calcium channel blocker used to manage supraventricular tachycardias, hypertrophic cardiomyopathy, and hypertension. It does not antagonize beta adrenergic receptors or act as a beta adrenergic agonist.

How well did you know this?
1
Not at all
2
3
4
5
Perfectly
37
Q

A 2-year old male castrated Beagle presents for vomiting and diarrhea. The blood smear for a CBC shows microfilariae in the peripheral blood. Which of the following parasites must be ruled out?

Acanthocheilonema reconditum
Filaroides osleri
Strongyloides stercoralis
Uncinaria stenocephala

A

Answer: Acanthocheilonema reconditum

Explanation
The correct answer is Acanthocheilonema reconditum (previously Dipetalonema reconditum).

Acanthocheilonema reconditum is a blood parasite that looks similar to the microfilariae of Dirofilaria immitis, the agent of Heartworm disease. The two parasites must be differentiated because Acanthocheilonema reconditum is not pathogenic and is therefore not treated. Uncinaria stenocephala, Strongyloides stercoralis, and Filaroides osleri would not be found in the blood.

Diltiazem slows the heart by inhibiting the influx of calcium into myocardial cells. It is a calcium channel blocker used to manage supraventricular tachycardias, hypertrophic cardiomyopathy, and hypertension. It does not antagonize beta adrenergic receptors or act as a beta adrenergic agonist.

How well did you know this?
1
Not at all
2
3
4
5
Perfectly
38
Q

Which of the following statements is true about central venous pressure?
It is a direct measure of preload
It is an estimation of afterload
it is a direct measure of blood pressure in the cranial vena cava
The measurement is independent of cardiac output
The measurement is independent of venous vessel tone

A

Answer: it is a direct measure of blood pressure in the cranial vena cava

Explanation
The correct answer is it is a direct measure of blood pressure in the cranial vena cava. Venous blood volume, venous vessel tone, and cardiac output determine the CVP. CVP is an estimate of preload, not a direct measurement. Cardiac output is one of the factors along with venous tone and venous blood volume in obtaining CVP. Afterload is the force against which the myocardium must contract.

Central venous pressure (CVP) is a direct measure of blood pressure in the cranial vena cava. It is influenced by venous blood volume, venous vessel tone, and cardiac output. CVP estimates preload but does not directly measure it. Afterload is the resistance against which the myocardium contracts.

How well did you know this?
1
Not at all
2
3
4
5
Perfectly
39
Q

Which abnormal heart sound are you least likely to be able to auscult in a dog?
Atrial fibrillation
Gallop rhythm
First degree AV block
Sinus arrhythmia

A

Answer: First degree AV block

Explanation
The correct answer is first degree AV block as this is a very subtle prolongation between the P and the R wave which can usually only be recognized by evaluation of an ECG strip.

First degree AV block is least likely to be ausculted in a dog, as it involves a subtle prolongation between the P and R wave detectable only via ECG. Other abnormal heart sounds like atrial fibrillation, gallop rhythm, and sinus arrhythmia are more easily ausculted.

How well did you know this?
1
Not at all
2
3
4
5
Perfectly
40
Q

You diagnose a dog with a larger left to right ventricular septal defect with advanced sequelae to the abnormality. Which of the following is the most likely radiographic appearance of this dog’s thorax?

Right ventricular and left atrial enlargement with pulmonary vasculature hypoperfusion
Left atrial and left ventricular enlargement with pulmonary hyperperfusion
Right ventricular enlargement with pulmonary hyperperfusion and a normal left heart
Enlargement of all 4 heart chambers and hyperperfusion of the pulmonary vasculature

A

Answer: Left atrial and left ventricular enlargement with pulmonary hyperperfusion

Explanation
The correct answer is left atrial and left ventricular enlargement with pulmonary hyperperfusion. In dogs with a VSD, the shunt occurs during systole when both ventricles are contracting so blood is shunted from the left ventricle almost directly into the pulmonary artery. The right ventricle acts as a conduit between the left ventricle and pulmonary artery and is less commonly affected by the shunted blood. The excess blood flow is found in the pulmonary vasculature, left atrium, and left ventricle, thus enlarging these structures.

The most likely radiographic appearance for a dog with a larger left to right ventricular septal defect and advanced sequelae is left atrial and left ventricular enlargement with pulmonary hyperperfusion. In VSD, blood shunts from the left ventricle to the pulmonary artery during systole, causing excess blood flow in the pulmonary vasculature, left atrium, and left ventricle, enlarging these structures.

How well did you know this?
1
Not at all
2
3
4
5
Perfectly
41
Q

A 15-year old male castrated terrier mix presents for cough and rapid, shallow breathing. You auscult a grade IV/VI left apical holosystolic murmur and crackles throughout both lung fields. An echocardiogram is performed, shown here. What findings are shown on this echocardiogram?

Tricuspid valve endocarditis
Mitral valve prolapse
Pulmonic stenosis
Left ventricular concentric hypertrophy
Right ventricular concentric hypertrophy

A

Answer: Mitral valve prolapse.

Explanation
The correct answer is mitral valve prolapse. Note how the mitral valve bends back into the left atrium consistent with mitral valve prolapse. The valve is also thick and mitral regurgitation is likely, which fits with the heart murmur reported. The left ventricular lumen is dilated and walls appear of normal thickness, so concentric hypertrophy is not present. The pulmonic valve is not seen in this image plane. The right ventricle is not well seen either, but in the very near field, the right ventricular wall appears normal. What can be seen of the tricuspid valve does not appear to have any vegetations consistent with endocarditis.

The echocardiogram shows mitral valve prolapse, evidenced by the mitral valve bending back into the left atrium. The valve is thickened, likely causing mitral regurgitation, consistent with the reported heart murmur. The left ventricular lumen is dilated with normal wall thickness, ruling out concentric hypertrophy. The pulmonic valve and right ventricle are not visible, and the tricuspid valve lacks vegetations indicative of endocarditis.

How well did you know this?
1
Not at all
2
3
4
5
Perfectly
42
Q

Mitral regurgitation in dogs eventually results in which of the following?
Eccentric hypertrophy of the left atrium and concentric hypertrophy of the left ventricle
Eccentric hypertrophy of the left atrium and left ventricle
Concentric hypertrophy of the left atrium and eccentric hypertrophy of the left ventricle
Concentric hypertrophy of the left atrium and left ventricle

A

Answer: Eccentric hypertrophy of the left atrium and left ventricle

Explanation
The correct answer is eccentric hypertrophy of the left atrium and left ventricle. Mitral regurgitation is the process of blood flowing back from the left ventricle to the left atrium due to a defect in the valve. The excess blood flow causes volume overload of the left atrium leading to eccentric hypertrophy or dilation of the chamber. The excess blood volume will then return to the left ventricle during the next diastole, resulting in dilation and eccentric hypertrophy of the left ventricle as well. Concentric hypertrophy occurs in cardiac chambers when they are pressure overloaded, such as in the case of pulmonic stenosis or aortic stenosis.

Mitral regurgitation in dogs leads to eccentric hypertrophy of the left atrium and left ventricle. The regurgitation causes volume overload, resulting in dilation and eccentric hypertrophy of the left atrium. This excess blood then flows into the left ventricle during diastole, causing its dilation and eccentric hypertrophy. Concentric hypertrophy is associated with pressure overload conditions like pulmonic or aortic stenosis.

How well did you know this?
1
Not at all
2
3
4
5
Perfectly
43
Q

Which of the following findings is consistent with Tetralogy of Fallot in a dog?
Left ventricular hypertrophy
Aortic stenosis
Polycythemia
Severe generalized cardiomegaly on thoracic radiography

A

Answer: Polycythemia

Explanation
Tetralogy of Fallot is a congenital defect of the heart which includes pulmonic stenosis (not aortic stenosis), dextro-positioned aorta (over-riding), RIGHT ventricular hypertrophy, and a ventricular septal defect. It causes shunting of venous blood across the VSD into the arterial circulation which leads to hypoxemia, and an increased drive for erythropoiesis, causing polycythemia.

Affected animals tend be underdeveloped in size and have a history of exercise intolerance, dyspnea, tachypnea, or syncope due to hypoxemia. Thoracic radiographs usually show a normal to mildly enlarged cardiac silhouette with right ventricular enlargement and pulmonary hypoperfusion due to the shunting of venous blood into the arterial circulation.

Polycythemia is consistent with Tetralogy of Fallot in dogs. This congenital defect includes pulmonic stenosis, dextro-positioned aorta, right ventricular hypertrophy, and a ventricular septal defect, leading to venous blood shunting into the arterial circulation. This results in hypoxemia, driving erythropoiesis and causing polycythemia. Affected dogs often exhibit exercise intolerance, dyspnea, tachypnea, or syncope, with thoracic radiographs showing a normal to mildly enlarged cardiac silhouette, right ventricular enlargement, and pulmonary hypoperfusion.

How well did you know this?
1
Not at all
2
3
4
5
Perfectly
44
Q

Dopamine is often used for which of the following reasons?
To increase contractility of the heart
To increase heart rate
To increase arterial blood pressure
To decrease blood pressure

A

Answer: To increase arterial blood pressure

Explanation
The correct answer is to increase blood pressure. High doses of dopamine are used as an alpha agonist to increase peripheral resistance and increase blood pressure. It is often used during anesthesia and emergency situations to alleviate hypotension.

Dopamine is often used to increase arterial blood pressure. At high doses, dopamine acts as an alpha agonist, increasing peripheral resistance and blood pressure. It is commonly administered during anesthesia and in emergency situations to counteract hypotension.

How well did you know this?
1
Not at all
2
3
4
5
Perfectly
45
Q

A 1-year old German Shepherd Dog presents for its first physical exam. Cardiac auscultation reveals a continuous murmur. Thoracic radiographs and echocardiography lead you to the diagnosis of a very large, cylinder-shaped patent ductus arteriosus with left-to-right shunting of blood. Which of the following statements is most true?

Furosemide and enalapril are the most appropriate forms of treatment for a cylinder-shaped PDA
Coil embolization is the most appropriate form of treatment for a cylinder-shaped PDA
Surgical ligation is the most appropriate form of treatment for a cylinder-shaped PDA
Placement of constrictor ring is the most appropriate form of treatment for a cylinder-shaped PDA

A

Answer: Surgical ligation is the most appropriate form of treatment for a cylinder-shaped PDA

Explanation
The correct answer is surgical ligation of the PDA is the most appropriate form of treatment. A large cylindrical PDA is unlikely amenable to closure via traditional coil embolization. Most canine PDAs are funnel-shaped with the smaller end of the funnel attached to the pulmonary artery, and the wider end attached to the aorta. The funnel shape is what allows the coils to lodge in place and cause closure of the PDA. Coils would not stay in place in a large, cylindrical PDA, so surgical ligation is the treatment of choice for this patient. There are newer nickel-titanium devices (Amplatz Canine Ductal Occluder) that show promise for minimally-invasive closure of PDAs which do not taper, but this answer was not provided. Ameroid constrictor rings are typically used for ligating hepatic shunts, and not for PDAs. Medical management with diuretics and heart drugs are only indicated prior to correction of the PDA if the dog is in heart failure.

Surgical ligation is the most appropriate treatment for a large, cylinder-shaped PDA in a 1-year-old German Shepherd Dog. Traditional coil embolization is ineffective for cylindrical PDAs, as coils require a funnel shape to lodge properly. Newer devices like the Amplatz Canine Ductal Occluder show promise but weren’t options here. Ameroid constrictor rings are for hepatic shunts, and medical management with diuretics and heart drugs is only for pre-correction heart failure.

How well did you know this?
1
Not at all
2
3
4
5
Perfectly
46
Q

Using phenothiazines in dogs causes which of the following effects?

Increase in hematocrit
Increases in myocardial sensitivity to catecholamine induced arrhythmias in the heart
Hypotension
Analgesia

A

Answer: Hypotension.

Explanation
The correct answer is hypotension. Phenothiazines cause sedation, but do not provide analgesia. They may decrease hematocrit due to splenic sequestration of red blood cells. They decrease the sensitivity of the myocardium to catecholamine-induced arrhythmias.

Phenothiazines cause hypotension in dogs. They provide sedation without analgesia, may decrease hematocrit due to splenic sequestration of red blood cells, and decrease myocardial sensitivity to catecholamine-induced arrhythmias.

How well did you know this?
1
Not at all
2
3
4
5
Perfectly
47
Q

A 7 year-old male neutered Doberman has recently been diagnosed with dilated cardiomyopathy. On your physical exam, you auscult a “tennis shoes in a dryer” sound. What heart rhythm are you describing?

Bradycardia
Tachycardia
Atrial fibrillation
Respiratory sinus arrhythmia
Atrioventricular block

A

Answer: Atrial fibrillation

Explanation
Patients with enlarged atria can develop the arrhythmia atrial fibrillation. Atrial fibrillation is characterized by a rapid irregularly irregular rhythm. Primary atrial fibrillation occurs idiopathically in some large-breed dogs, and patients are usually asymptomatic. Secondary atrial fibrillation is more common and occurs secondary to underlying cardiac disease.

Atrial fibrillation is the heart rhythm described by the “tennis shoes in a dryer” sound in a 7-year-old Doberman with dilated cardiomyopathy. It is characterized by a rapid, irregularly irregular rhythm and commonly occurs in dogs with enlarged atria due to underlying cardiac disease.

How well did you know this?
1
Not at all
2
3
4
5
Perfectly
48
Q

A 11-year old male castrated Golden Retriever presents for collapse with muffled heart sounds on examination. You ultrasound the heart and obtain the following image; the right atrium (RA) and right ventricle (RV) are labeled. A large mass is seen in the right atrioventricular groove with pericardial effusion. What is the likely diagnosis?

Hemangiosarcoma
Lymphoma
Chemodectoma
Idiopathic pericardial hemorrhage
Pulmonary adenocarcinoma

A

Answer: Hemangiosarcoma.

Explanation
The correct answer is hemangiosarcoma. Hemangiosarcoma of the heart has a predilection for the right auricle of dogs, with Golden Retrievers being predisposed. Stabilization of this patient will require pericardiocentesis; prognosis is guarded and may include chemotherapy, pericardiectomy, or rarely auriculectomy. Chemodectoma arises from the ascending aorta and heart base; lymphoma is rare in the heart of dogs and is more commonly metastatic than a single mass. Idiopathic pericardial hemorrhage is incorrect because of the clear evidence of a mass.

The likely diagnosis is hemangiosarcoma. Hemangiosarcoma often affects the right auricle in dogs, with Golden Retrievers being predisposed. Stabilization involves pericardiocentesis; prognosis is guarded and may involve chemotherapy, pericardiectomy, or rarely auriculectomy. Chemodectoma arises from the ascending aorta and heart base. Lymphoma in the heart is rare and typically metastatic. Idiopathic pericardial hemorrhage is excluded due to the evident mass.

How well did you know this?
1
Not at all
2
3
4
5
Perfectly
49
Q

Which of the following is used as a medical management option in congestive heart failure in a dog?
Lidocaine
Mirtazapine
Increased sodium intake
Furosemide

A

Answer: Furosemide.

Explanation
The correct answer is furosemide. Sodium-restricted diets, ACE inhibitors such as enalapril, and diuretics such as furosemide decrease blood volume and preload. ACE inhibitors also reduce afterload by preventing production of angiotensin-ll. a potent vasoconstrictor. Pimobendan is also beneficial in the management of congestive heart failure in the dog due to its positive inotropic effects.

Lidocaine is used for treating ventricular arrhythmias. Mirtazapine is an appetite stimulator and not a drug used to treat CHF.

How well did you know this?
1
Not at all
2
3
4
5
Perfectly
50
Q

A 5-year old Pug in an animal shelter dies after experiencing cough, exercise intolerance, and dyspnea as well as ascites and hepatomegaly secondary to congestive heart failure. You perform a necropsy and find the parasite shown in the image below residing primarily in the pulmonary artery. How is this parasite transmitted?

Mosquito
Aerosol
Tick
Fecal-oral
Flea

A

Answer: Mosquito.

Dirofilaria immitis or heartworm is the parasite in dogs that resides in the pulmonary artery and can cause congestive heart failure if not recognized and treated. Mosquitoes carrying infective heartworm larvae transmit the infection through bites. The larvae then grow and develop over several months to become sexually mature worms. They then release offspring (microfilariae) into the blood stream.

How well did you know this?
1
Not at all
2
3
4
5
Perfectly
51
Q

This ECG recording (bottom recording) is from a 10-year old miniature poodle with a history of heart disease that is untreated. She presented for a 3-day history of diarrhea and decreased appetite. On physical examination you auscult a grade Ill/VI heart murmur and clear lung sounds. There are no obvious pulse deficits palpated. What is the appropriate treatment for this arrhythmia?

Give the patient a bolus of lidocaine and begin continuous infusion for twelve hours and then try to wean off the lidocaine
Begin the patient on oral mexiletine and recheck an ECG strip in 5 days
No anti-arrhythmic therapy is necessary at this time
Give the patient an intravenous injection of furosemide and begin oral administration of furosemide

A

Answer: No anti-arrhythmic therapy is necessary at this time

There are several general guidelines/recommendations regarding treatment of ventricular premature contractions which is what is shown on the ECG strip. These include a heart rate of over 180 bpm, pulse deficits, clinical signs, and VPCs for over 20 seconds in duration. In this case, the heart rate is visible on the recording and clearly not elevated. The patient’s physical exam findings are not suggestive of an immediate problem secondary to the arrhythmia. There are many dogs with intermittent VPCs that are not of clinical importance.

However, it is recommended that the patient be evaluated regularly and ideally have a consultation with a cardiologist to ensure the progression of heart disease is being minimized.

Furosemide is not indicated as there is not an indication that the patient is fluid overloaded or in fulminant heart failure.

Mexiletine is a Class IB anti-arrhythmic and is not necessary given the lack of severity in the ECG findings.

No anti-arrhythmic therapy is necessary at this time. The ECG shows ventricular premature contractions (VPCs) without clinical signs such as an elevated heart rate over 180 bpm, pulse deficits, or sustained VPCs over 20 seconds. Regular evaluation and cardiologist consultation are recommended to monitor heart disease progression. Furosemide is unnecessary as there’s no indication of fluid overload or heart failure. Mexiletine, a Class IB anti-arrhythmic, is also not needed given the mild ECG findings.

How well did you know this?
1
Not at all
2
3
4
5
Perfectly
52
Q

A 6-year old female spayed Beagle presents for lethargy. On exam you detect a heart rate of 25 beats per minute and record an ECG, which is shown here. What is the treatment of choice?

Pacemaker implantation
Atropine administered intramuscularly
Doxycycline
Furosemide
Sotalol

A

Answer: Pacemaker implantation

Explanation
The correct answer is pacemaker implantation. The rhythm is complete heart block (also known as 3rd degree AV block). There is no consistent PR interval; the ventricular beats are escape beats. Atropine can be considered, but is typically not effective in 3rd degree AV block. The only way to restore a normal heart rate in this patient is by implanting a pacemaker, which will require referral to a cardiologist.

Sotalol is a ventricular antiarrhythmic, which would actually be dangerous in this patient because it may suppress the ventricular escape focus. Furosemide and doxycycline will not prove beneficial for this rhythm disturbance.

Pacemaker implantation is the treatment of choice for a 6-year-old Beagle with complete heart block (3rd degree AV block), indicated by a heart rate of 25 bpm and an ECG showing no consistent PR interval with escape beats. Atropine is typically ineffective in 3rd degree AV block. Sotalol, a ventricular antiarrhythmic, could suppress the ventricular escape focus, making it dangerous. Furosemide and doxycycline are not beneficial for this rhythm disturbance.

How well did you know this?
1
Not at all
2
3
4
5
Perfectly
53
Q

A 12-year old female Dalmatian has a Holter monitor placed for an arrhythmia heard on exam. The following trace is from that monitor; similar abnormalities were seen frequently throughout the 24hrs. What is your treatment recommendation for this arrhythmia?

Mexiletine
Diltiazem and furosemide
Amlodipine
Pimobendan

A

Answer: Mexiletine

Explanation
The correct answer is mexiletine. The Holter ECG shows a couplet of ventricular premature complexes. If frequent ventricular ectopy is seen, particularly with runs or R-on-T episodes, treatment is advised with oral ventricular antiarrhythmics. Mexiletine is an oral class I antiarrhythmic that is well-tolerated in dogs.

Pimobendan improves systolic function, but does not treat arrhythmias. Amlodipine is a calcium channel blocker used for patients with systemic hypertension. Diltiazem is used for supraventricular tachycardias to slow conduction down the AV node. Furosemide is a diuretic used in the treatment of heart failure, it does not have antiarrhythmic properties.

The correct treatment recommendation for the arrhythmia is mexiletine. The Holter ECG shows a couplet of ventricular premature complexes, and frequent ventricular ectopy, particularly with runs or R-on-T episodes, warrants treatment with oral ventricular antiarrhythmics. Mexiletine, a class I antiarrhythmic, is well-tolerated in dogs. Pimobendan, amlodipine, diltiazem, and furosemide do not address ventricular arrhythmias and serve different cardiac or hypertensive conditions.

How well did you know this?
1
Not at all
2
3
4
5
Perfectly
54
Q

A 7-year old male castrated Doberman Pinscher presents for lethargy and anorexia. Your physical exam reveals a 3/6 left systolic heart murmur and a rectal temperature of 103.5F (39.7 C). One of your differential diagnoses for this dog is infective endocarditis. Which area of the heart is most commonly affected in a dog with infective endocarditis?

Mitral and aortic valves
Pulmonic and aortic valves
Mitral and tricuspid valves
Ventricular endocardium
Tricuspid and pulmonic valves

A

Answer: Mitral and aortic valves

Explanation
The correct answer is mitral and aortic valves. Dogs with endocarditis are most commonly affected on the mitral and aortic valves. The tricuspid and pulmonary valves are rarely affected in dogs and cats, but are the more common sites of infection in large animal species.

The mitral and aortic valves are most commonly affected in dogs with infective endocarditis. In contrast, the tricuspid and pulmonary valves are rarely involved in dogs and cats but are more commonly infected in large animal species. Thus, in a 7-year-old Doberman Pinscher with signs suggestive of endocarditis, focus on the mitral and aortic valves.

How well did you know this?
1
Not at all
2
3
4
5
Perfectly
55
Q

A 9 year old female spayed Greyhound presents to you for evaluation of a cough of 2 weeks duration and respiratory difficulty. The owner reports that the dog collapsed while chasing her ball yesterday. On your exam, heart sounds are quiet and the dog’s mucous membranes appear pale. You elect to take thoracic radiographs which are shown below. What is an appropriate treatment?

Atenolol
Measure taurine levels
Heartworm test
Pericardiocentesis
Furosemide

A

Answer: Pericardiocentesis

The radiograph and findings are consistent with pericardial effusion. Note the markedly enlarged globoid cardiac silhouette on the DV projection. Pericardiocentesis is necessary for therapeutic and diagnostic purposes.

Atenolol and furosemide would be contraindicated. While this dog is showing signs of heart failure, furosemide would decrease preload on the heart and make diastolic filling even worse in cases of pericardial effusion.

Atenolol would decrease contractility. This is not a case of heartworm. Radiographic signs of heartworm include right sided cardiac hypertrophy and prominent pulmonary arteries. Taurine levels could be measured in dogs with dilated cardiomyopathy but will take days to get back and this dog needs to be treated immediately. If facilities permit, even a brief echocardiogram could be performed to confirm pericardial effusion and/or to guide the pericardiocentesis.

Pericardiocentesis is the appropriate treatment for a 9-year-old Greyhound with a cough, respiratory difficulty, and a markedly enlarged globoid cardiac silhouette on thoracic radiographs, indicating pericardial effusion. Atenolol and furosemide are contraindicated as they would worsen diastolic filling and decrease contractility, respectively. Heartworm test and taurine level measurement are unnecessary in this immediate context. An echocardiogram can confirm the diagnosis and guide pericardiocentesis.

How well did you know this?
1
Not at all
2
3
4
5
Perfectly
56
Q

An 11-year old female spayed Shih Tzu presents for her annual physical exam. She appears to be doing fine at home. Cardiac auscultation reveals a 2/6 left systolic heart murmur over the apex of the heart. Color Doppler echocardiography reveals a small regurgitant jet across the mitral valve. What is the most likely cause for the mitral regurgitation in this dog?

Congenital mitral dysplasia
Bacterial endocarditis
Myxomatous mitral valve degeneration
Systolic anterior motion

A

Answer: Myxomatous mitral valve degeneration

Explanation
The correct answer is myxomatous mitral valve degeneration. This disease is the most common cardiac disease in veterinary medicine and affects up to 60% of dogs older than 8 years of age. It commonly occurs in small breed dogs and can clinically range from having only a soft murmur to being in congestive heart failure. It is caused by degeneration and accumulation of acid-staining mucopolysaccharides in the cardiac valves and most commonly affects the mitral valve. A dog with bacterial endocarditis would show signs of systemic illness such as fever and lethargy. Congenital mitral dysplasia would usually result in more marked cardiac changes (louder murmur, more regurgitation, dilated heart chambers) at a much earlier age. Systolic anterior motion or SAM of the mitral valve occurs with hypertrophic cardiomyopathy as one of the leaflets of the mitral valve obstructs the left ventricular outflow tract as the chamber contracts.

The most likely cause of mitral regurgitation in this 11-year-old Shih Tzu is myxomatous mitral valve degeneration. This common cardiac disease in dogs over 8 years old, especially small breeds, ranges from a soft murmur to congestive heart failure. It involves degeneration and mucopolysaccharide accumulation in the mitral valve. Bacterial endocarditis, congenital mitral dysplasia, and systolic anterior motion are less likely due to their distinct presentations and associated symptoms.

How well did you know this?
1
Not at all
2
3
4
5
Perfectly
57
Q

A 5-month-old intact female mixed breed presents to your hospital for the progressive lesions seen on the image provided. The owner described the lesions as initially being papules and vesicles that eventually ruptured. Recently, the owner has also noticed that the muscles of the face appear to be atrophied. Muscle and skin biopsies were performed which came back supportive of a diagnosis of dermatomyositis. You start the patient on an immunosuppressive dose of steroids. Which other medication would be useful for the treatment of this condition?

Meloxicam
Zinc
Hycodan
Pentoxifylline

A

Answer: Pentoxifylline

Explanation
Dermatomyositis (DMS) is a relatively unusual condition that results in skin lesions as well as myositis. The skin lesions initially appear as papules and vesicles which then rupture, ulcerate, and may progress to crusty alopecic lesions. The myositis sets in a little later, and atrophy of the temporalis muscle is often seen. In more severe cases, the muscles of the distal extremities become involved, and megaesophagus may be observed. Since muscle changes usually lag, other diagnostic tests such as a skin scraping and cytology are performed first to rule out other common conditions. The condition has a variable prognosis, as the clinical signs may wax and wane. Medical management usually consists of pentoxifylline, prednisone, and cyclosporine. Prednisone and cyclosporine serve as immunomodulatory medications. Pentoxifylline is used to help improve blood flow.

Hycodan contains hydrocodone and is used as a cough suppressant. Meloxicam would be contraindicated in a patient on steroids. Although it looks similar to a zinc-responsive dermatosis, zinc would not be a treatment for DMS.

Pentoxifylline is the recommended additional medication for treating dermatomyositis (DMS) in this 5-month-old mixed breed dog, along with immunosuppressive steroids like prednisone. DMS causes skin lesions and myositis, with symptoms such as papules, vesicles, and facial muscle atrophy. Pentoxifylline improves blood flow, aiding treatment. Hycodan (a cough suppressant), meloxicam (contraindicated with steroids), and zinc (not effective for DMS) are not suitable for this condition.

How well did you know this?
1
Not at all
2
3
4
5
Perfectly
58
Q

A 1-year old female spayed Doberman Pinscher has presented after being hit by a car. Initial chest radiographs show mild contusions, and the patient appears to be otherwise stable. A right mid-shaft long oblique femoral fracture has been identified. Routine pre-operative blood work is unremarkable. A buccal mucosal bleeding test (BMBT) is elevated at 6 minutes. What will you administer prior to surgery?

1,25 dihydrocholecalciferol
Whole blood transfusion
Desmopressin acetate
Vitamin K

A

Answer: Desmopressin acetate

Explanation
The correct answer is desmopressin acetate (DDAVP). Administration of desmopressin results in release of von Willebrand factor, which will help this patient with clotting. Given this dog’s breed and elevated BMBT there is a very strong likelihood she is afflicted with von Willebrand’s disease. In Dobermans this results in an inability to form a clot. This can be life threatening if the dog is taken to surgery.

A whole blood transfusion does not provide an adequate source of von Willebrand factor but may be necessary if the patient’s bleeding cannot be controlled despite appropriate pre-operative measures. 1,25
dihydrocholecalciferol is the active form of vitamin D which aids intestinal resorption of calcium. The BMBT does not assess factors 2, 7, 9, or 10 and therefore vitamin K is not indicated.

Desmopressin acetate (DDAVP) is the appropriate treatment prior to surgery for this 1-year-old Doberman Pinscher, likely affected by von Willebrand’s disease, as indicated by an elevated BMBT. DDAVP releases von Willebrand factor to aid clotting. Whole blood transfusions do not provide sufficient von Willebrand factor and are not a primary treatment. 1,25 dihydrocholecalciferol and vitamin K are not indicated, as they do not address the underlying clotting deficiency.

How well did you know this?
1
Not at all
2
3
4
5
Perfectly
59
Q

The mean circulating life span of a neutrophil in the dog is approximately________

48 hours
120 minutes
5-9 hours
5-9 days

A

Answer: 5-9 hours

Explanation
The correct answer is 5-9 hours.

Neutrophil lifespans in all species, as far as we know, are less than 24 hours.

Cows-16 hours; Horses- 10-11 hours.

Platelets have a circulating half-life of 5-7 days and red blood cells have an approximate life span of 110 days in dogs

The approximate life span of red blood cells in other species (in days) is as follows: cats-70, cow-160, horse-145, pig-86, sheep-150.

The mean circulating lifespan of a neutrophil in dogs is approximately 5-9 hours. This lifespan is consistent with other species, generally being less than 24 hours. In contrast, platelets have a half-life of 5-7 days, and red blood cells last around 110 days in dogs.

How well did you know this?
1
Not at all
2
3
4
5
Perfectly
60
Q

A 7 year old female spayed Brittany Spaniel presents to your emergency clinic for a several day history of progressive weakness and lethargy. Although she was a hunting dog when she was younger, she has no recent history of travel or tick exposure. On exam, she has a temperature of 104.7F (40.4 C), heart rate of 128 beats per minute, and a respiratory rate of 42 breaths per minute. Her mucous membranes are pale pink with a capillary refill time of 1.5 seconds. Your laboratory is closed, so you perform a blood smear and see the following. Based on the most likely diagnosis for these findings, and assuming you have access to all of these treatment options, which of the following treatments is most appropriate?

Erythropoietin injections and iron supplementation
Platelet-rich-plasma transfusion and anti-inflammatory doses of corticosteroids
Packed red blood cell transfusion, immunosuppressive doses of corticosteroids
Plasma transfusion, vitamin K supplementation
Fresh whole blood transfusion, Ampicillin

A

Answer: Packed red blood cell transfusion, immunosuppressive doses of corticosteroids

Explanation
This is most compatible with a case of immune mediated hemolytic anemia (IMHA or IHA). The findings that should lead you to this diagnosis are the clinical signs of lethargy with fever, pale mucous membranes, and blood smear findings. On the blood smear, you should see that there are small numbers of erythrocytes compared to the numbers of platelets and leukocytes. You should appreciate that there is also polychromasia (variation in the color of the erythrocytes) with some red blood cells appearing very pale and others appearing a more purple color, consistent with a regenerative response from the bone marrow. It may be difficult to appreciate spherocytes on this blood smear, but this is another classic finding in cases of IMHA.

IMHA can be a primary (idiopathic) condition or can be secondary to other causes including infectious diseases, autoimmune, or neoplastic conditions. Regardless, treatment consists of managing the debilitating anemia with transfusions of red blood cells and suppression of the immune mediated destruction of erythrocytes with immunosuppressive doses of steroids. Searching for an underlying cause and treating it appropriately is also critical.

Plasma transfusion or whole blood transfusion with Vitamin K would be appropriate treatments for coagulopathy +/- disseminated intravascular coagulation (DIC), especially secondary to Vitamin K antagonism (i.e. rodenticide). This case should not be confused with rodenticide toxicity due to the fever and red blood cell findings on the smear. Erythropoietin and iron might be appropriate for anemia of chronic disease, especially renal disease, but are not indicated in this case. This animal’s fever is not necessarily indicative of infection, as immune-mediated disease can be just as pyrogenic as an infectious process. Ampicillin is a beta lactam antibiotic but is not the critical therapy for this animal at this time.

More information such as coagulation times and hematocrit would have helped you in this case and certainly would have been appropriate to run if asked or if managing this case in the real world; however, for a board exam, you may not always be given all of the information you want and must base your decision on the most likely diagnosis for the information that is provided.

The most appropriate treatment for this 7-year-old Brittany Spaniel, likely diagnosed with immune-mediated hemolytic anemia (IMHA), is a packed red blood cell transfusion and immunosuppressive doses of corticosteroids. IMHA is suggested by clinical signs of lethargy, fever, pale mucous membranes, and blood smear findings, including polychromasia and a decreased erythrocyte count. Immunosuppressive therapy is crucial to manage the immune-mediated destruction of erythrocytes. Additional treatments like erythropoietin, iron, plasma transfusion, vitamin K, or antibiotics are not indicated for this condition. Further investigation into potential underlying causes is also important.

How well did you know this?
1
Not at all
2
3
4
5
Perfectly
61
Q

A 7-year old female Lhasa Apso presents to you for lethargy and inappetence. On your exam, you detect mandibular lymphadenopathy and perform a fine needle aspirate. You see the aspirate depicted here. Which of these treatments would be given to this patient as part of a first line therapy?

Doxycycline
Milbemycin
Itraconazole
Prednisone
Carboplatin

A

Answer: Prednisone

Explanation
This is a case of lymphoma. The cytology depicts the classic finding of a population of lymphoid cells that are predominantly lymphoblasts based on their size and characteristics. If you were unsure about their size due to magnification, there is a neutrophil in the lower left corner for comparison. The lymphoblasts are considerably larger than the neutrophil. Mature lymphocytes would be smaller than a neutrophil. Note that there is a mitotic figure in the middle of the slide.

There are many treatments and protocols for lymphoma and some of the main agents known to have efficacy are prednisone, doxorubicin, cyclophosphamide, vincristine, L-asparaginase, and lomustine. There are many other efficacious chemotherapeutics for lymphoma but carboplatin is not considered a first line treatment for lymphoma in dogs.

The other drugs listed are antifungal (itraconazole), antibiotic (doxycycline) and anti-parasitic (milbemycin).

Prednisone is part of the first-line therapy for lymphoma, as seen in the cytology of the fine needle aspirate showing predominantly lymphoblasts. Lymphoma treatments often include prednisone, doxorubicin, cyclophosphamide, vincristine, L-asparaginase, and lomustine. Carboplatin is not a first-line treatment for lymphoma in dogs. The other options—doxycycline, milbemycin, and itraconazole—are not relevant for lymphoma, as they are an antibiotic, antiparasitic, and antifungal, respectively.

How well did you know this?
1
Not at all
2
3
4
5
Perfectly
62
Q

A 1-year-old male castrated Basset Hound presents for epistaxis from both nostrils, a small amount from blood from the right and a larger amount from the left. There is no history of trauma but he does go for frequent walks, and as a typical Basset, usually has his nose to the ground. His heart and lungs sound normal. You note a few ecchymotic hemorrhages on the thin skin under his abdominal region. You discuss the various causes of this issue with the owner and perform some labwork, including a coagulation panel, and the important values to note are as follows:
PCV 34%
Reticulocytes 110K
Platelets 180K/uL (143-448 K/uL)
Prothrombin Time (PT) 6.95 (6.3-13.3 seconds)
Partial Thromboplastin Time (PTT) 10.95 (10.6-16.8 seconds)
Based on this information, what is your top differential?

Immune-mediated thrombocytopenia
Rodenticide toxicity
Nasal foreign body
Canine thrombopathia

A

Answer: Canine thrombopathia

Explanation
The correct answer is canine thrombopathia. The disease is an autosomal recessive trait seen in Basset Hounds where platelets fail to aggregate and secrete their granules in response to normal stimuli. These dogs are at an increased risk for bleeding spontaneously, and an injury or surgical procedure could cause excessive hemorrhage. Platelet numbers and coagulation parameters are normal in this condition. It isn’t an issue with platelet number, but of function.

Rodenticide would cause elevated clotting times.

Immune-mediated thrombocytopenia would cause a low platelet count.

A nasal foreign body would be less likely to cause a bilateral bleed and would not attribute to the ecchymotic hemorrhages noted on the abdominal skin.

The top differential for the 1-year-old Basset Hound with epistaxis and ecchymotic hemorrhages is canine thrombopathia. This autosomal recessive trait in Basset Hounds results in platelets failing to aggregate and secrete granules normally, leading to increased bleeding risk despite normal platelet numbers and coagulation parameters. Rodenticide toxicity would elevate clotting times, immune-mediated thrombocytopenia would lower platelet count, and a nasal foreign body typically wouldn’t cause bilateral bleeding or ecchymotic hemorrhages.

How well did you know this?
1
Not at all
2
3
4
5
Perfectly
63
Q

Zinc toxicity from ingestion of pennies minted after 1983 can cause which of the following?

Muscle spasms
Coagulopathy
Hemolysis
Correct Answer
Thrombocytopenia

A

Answer: Hemolysis

Explanation
The correct answer is hemolysis. Zinc causes a Heinz body anemia and hemolysis.

How well did you know this?
1
Not at all
2
3
4
5
Perfectly
64
Q

Immune-mediated hemolytic anemia is classically associated with which type of hypersensitivity reaction?

Type IV
Type Il
Type I
Туре II!

A

Answer: Type II

Type I (aka immediate) hypersensitivity reactions occur when re-exposure of an antigen results in an allergic reaction. The antigen is presented and causes B cell production of IgE antibodies.

Type Il (aka cytotoxic) occurs when an antigen on a cell surface binds with an antibody and is then recognized by the body as being foreign. The antigen on the cell may have been a normal feature of the cell or it may have been acquired, such as a drug binding to the cell. Macrophages or dendritic cells then recognize the cell and “present the antigen” which then causes B cell proliferation and production of IgG and IgM antibodies. The antibodies bind to the cell and activate the complement cascade which results in cell lysis/destruction.

Type Ill (aka immune complex) occurs when there are more antigens than there are antibodies. The antigens float around in the circulation and multiple antigens may bind one antibody, thus forming an immune complex.

Large complexes can be cleared by macrophages, but smaller ones may evade the macrophages.

Type IV (delayed type hypersensitivity) takes several days to occur and rather than being antibody-mediated, it is actually cell-mediated. Cytotoxic CD8+ T cells and CD4+ helper T cells recognize antigen that is in a complex with major histocompatibility complex 1 or 2. This results in proliferation and activation of the cells. Activated CD8+ T cells then destroy the antigen containing cells while activated macrophages release hydrolytic enzymes.

Immune-mediated hemolytic anemia (IMHA) is associated with a Type II hypersensitivity reaction. In Type II hypersensitivity, antibodies target antigens on cell surfaces, leading to cell destruction. This can occur due to the immune system mistakenly recognizing normal cell surface antigens or acquired antigens, such as those from drug interactions, as foreign. The process involves macrophages or dendritic cells presenting the antigen, leading to B cell production of IgG and IgM antibodies, which activate the complement cascade, resulting in cell lysis. This contrasts with other hypersensitivity types that involve different immune mechanisms.

How well did you know this?
1
Not at all
2
3
4
5
Perfectly
65
Q

A 7-year old male neutered Golden Retriever presents for lethargy and pale mucous membranes. A purple top (EDTA) tube of blood is drawn. A blood smear is made and examined. There is a moderate regenerative anemia with an increase in spherocytes. Mild agglutination is suspected. Spherocytes are consistent with which disease condition?

Iron deficiency
Grape and raisin toxicity
Cobalamin and folate deficiency
Immune mediated hemolytic anemia

A

Answer: Immune mediated hemolytic anemia

Explanation
In immune mediated hemolytic anemia red blood cells are coated with immunoglobulins and are then engulfed by macrophages. As a result, the RBC will lose a portion of its cell membrane but not hemoglobin. With the loss of part of the cell membrane, spherocytes appear smaller (without volume loss), no central pallor, and stain deeply red.

In iron deficiency, RBC will be hypochromic (less red) and smaller (microcytic) due to less volume.

Cobalamin and folate are necessary for DNA synthesis. Giant schnauzers have an inherited cobalamin malabsorption which results in macrocytes and ovalocytes in circulation.

Grape and raisin toxicity result in acute renal failure.

Spherocytes are consistent with immune-mediated hemolytic anemia (IMHA). In IMHA, red blood cells are coated with immunoglobulins and partially phagocytized by macrophages, leading to a loss of cell membrane but not hemoglobin. This results in smaller, deeply red cells without central pallor. In iron deficiency, red blood cells are hypochromic and microcytic. Cobalamin and folate deficiencies lead to macrocytes and ovalocytes, and grape and raisin toxicity cause acute renal failure, not changes in red blood cell morphology.

How well did you know this?
1
Not at all
2
3
4
5
Perfectly
66
Q

Which of these treatments might be used for a dog with immune-mediated hemolytic anemia?

Enrofloxacin
Azathioprine
Maropitant
Piroxicam

A

Answer: Azathioprine

Explanation
The mainstays of therapy are immunosuppression with corticosteroids and other drugs such as azathioprine or cyclosporine. Transfusions with whole blood, packed red blood cells, or oxyglobin may be necessary.
Piroxicam is a non-steroidal anti-inflammatory and is contraindicated in any patient receiving corticosteroids due to risk of cumulative GI toxicity. Enrofloxacin is an antibiotic and would not commonly be prescribed for autoimmune disease. Maropitant (Cerenia) is used to treat nausea and vomiting.

Azathioprine is used for treating immune-mediated hemolytic anemia (IMHA) in dogs, often alongside corticosteroids for immunosuppression. Other options include cyclosporine and blood transfusions if needed. Piroxicam, a non-steroidal anti-inflammatory, is contraindicated with corticosteroids due to GI toxicity risk. Enrofloxacin is an antibiotic, and Maropitant (Cerenia) is used for nausea and vomiting, not for IMHA.

How well did you know this?
1
Not at all
2
3
4
5
Perfectly
67
Q

A 6-week old male intact puppy has presented to your clinic after being bitten 4 days ago. On physical examination, there is a moderate amount of purulent discharge noted from the bite wound on the left antebrachium. The wound is approximately 2 cm in length. Which antibiotic is LEAST appropriate for this patient?

Doxycycline
Cephalexin
Penicillin
Metronidazole

A

Answer: Doxycycline

Explanation
Doxycycline can result in delayed bone growth and discoloration of the teeth in young growing animals and is thus contraindicated. The other medications listed do not have a direct effect on young growing animals and are safe to administer; however, one must consider the likely pathogen present in the wound, what type of susceptibility pattern it has, as well as the likely penetration of your antibiotic into the area where you want it to exert its effect prior to making a choice.

How well did you know this?
1
Not at all
2
3
4
5
Perfectly
68
Q

A 3-year-old spayed female Rhodesian Ridgeback presents for mild pruritus and skin lesions. Upon examination, you notice diffuse papules and occasional pustules along the dorsal trunk. You conclude that the patient has folliculitis. Which of the following most commonly causes pustule formation and folliculitis such as this?

Sarcoptes scabiei
Staphylococcus
Trichodectes
Chyletiella

A

Answer: Staphylococcus

Explanation
Bacteria may be a cause of folliculitis. Pustules such as this are commonly associated with Staphylococcus.
Dermatophytes, and most species of Demodex, can also infect the hair follicle.

Sarcoptes scabiei, which is a non-seasonal, intensely pruritic, transmissible mite, affects the stratum corneum and is less likely to cause a folliculitis.

Chyletiella mites feed on the keratin layer of the skin as well and are less likely to cause folliculitis. These mites are known as “walking dandruff” and may cause skin flaking and typically intense pruritis.

Trichodectes is a chewing louse, which chews the skin but does not involve the hair follicle.

How well did you know this?
1
Not at all
2
3
4
5
Perfectly
69
Q

A 7-year-old pit bull mix presents with these skin lesions (see image) and mild pruritus. The owner has recently started him on a hypoallergenic diet. You perform a skin cytology and observe neutrophilic inflammation. What is a top differential for this condition?

Cheyletiella
Sarcoptes
Dermatophytosis
Bacterial pyoderma

A

Answer: Bacterial pyoderma

The lesions in the image are pustules, which form with neutrophilic inflammation and are accumulations of dead or dying polymorphonuclear leukocytes. Pustules are seen with differing types of dermatitis ranging from bacterial pyoderma, demodicosis, allergic reaction, and pemphigus. Determining the primary etiology will aid in proper treatment and resolution. Diagnostics that are always appropriate for diagnosing skin infections include skin scraping, impression smears, hair plucking for fungal culture, and biopsies if the preliminary diagnostics are unrewarding.

Demodicosis is the only mite infection that has been known to cause pustules or papules. Sarcoptes and Cheyletiella are not known to cause pustules. Sarcoptes is also usually associated with intense pruritis.

Dermatophytosis can cause pustules, but this is not routinely seen, which makes bacterial pyoderma the top differential from the given answer choices.

How well did you know this?
1
Not at all
2
3
4
5
Perfectly
70
Q

A chocolate lab presents for routine wellness examination and vaccinations. Your only abnormal finding on physical examination is seen here (see image). What treatment would you NOT recommend?

Dips
Manual removal
Pour on
Burn off

A

Answer: Burn off

Explanation
Burning may cause the ticks to become agitated and regurgitate their stomach contents into the surrounding tissues they are feeding from. This will cause severe inflammation and pruritis, and can also lead to tick borne diseases. Burning can also harm the patient. The correct way to remove ticks is to grasp them with forceps just behind their heads and gently apply traction until they release. Twisting will cause the heads to stay lodged under the skin. For large numbers or ticks, as seen in this image, Fipronil pour-ons and Permethin topicals can be used to kill off the ticks. Periodic pour-ons and bathing routinely can also be used in the face of severe infestations.

How well did you know this?
1
Not at all
2
3
4
5
Perfectly
71
Q

A 4-month old intact male Collie presents for severe dermatitis along his nose and ears (see image). The areas are sensitive on palpation but not pruritic. No other areas are affected. Skin scraping and cytology are negative for mites or bacteria. Fungal culture is negative. Anti-nuclear antibody tests are ordered and are also negative. How do you educate the owner based on your presumptive diagnosis?

Biopsy and culture are necessary, long-term therapy often needed.
Biopsy and culture are necessary, short-term therapy often needed.
The disease is not curable; euthanasia should be discussed.
The disease is familial, and is often cyclic and will regress.

A

Answer: The disease is familial, and is often cyclic and will regress.

Explanation
Familial dermatomyositis is a familial disease seen in Collies, Shetland sheepdogs, and other collie breeds.

Diagnosis can be made with skin or muscle biopsies, but can be difficult as the clinical and histopathologic signs can wax and wane. Inciting causes can include vaccinations, sunlight, viral infections, or drugs. Some cases; however, will come and go without obvious inciting causes. Some cases can become severe enough to include muscle atrophy; megaesophagus has also been seen with this disease.

Treatments are often symptomatic and supportive and include Vitamin E supplements, Omega 3 fish oils, treatments for any secondary bacterial infections, and pentoxifylline. Prednisone can also be used to get more severe cases under control. Avoiding prolonged sun exposure can also help.

Educating the owner on the fact that signs will reoccur despite treatment can alleviate some owner frustrations.

Often times signs will regress with or without treatments. Educate owners on neutering animals to prevent propagation of diseased animals.

How well did you know this?
1
Not at all
2
3
4
5
Perfectly
72
Q

A 2-year old female spayed Dachshund presents for further evaluation as a result of a mass on the pinna noted by the owner. On physical exam, the only abnormality noted was this mass. Cytology is consistent with a histiocytoma. Which of the following is the most reasonable treatment option?

Cryotherapy
Chemotherapy using lomustine
Radical surgical excision
Radiation therapy
Benign Neglect

A

Answer: Benign Neglect

Explanation
Histiocytomas are typically benign and will often present as a small, raised mass that may or may not be ulcerated. Fortunately, these masses usually regress on their own, and surgical or medical intervention is typically not necessary.

The most reasonable treatment option for a histiocytoma in a 2-year-old Dachshund is benign neglect. Histiocytomas are usually benign and often regress on their own without the need for surgical or medical intervention. They typically present as small, raised masses that may be ulcerated but usually resolve spontaneously.

How well did you know this?
1
Not at all
2
3
4
5
Perfectly
73
Q

Which of the following clinical signs may be seen with discoid lupus erythematosus (DLE) in dogs?

Hyperpigmentation of the ventral abdomen
Depigmentation of the nasal planum
Symmetrical truncal alopecia
Swollen, painful joints

A

Answer: Depigmentation of the nasal planum

Explanation
The correct answer is depigmentation of the nasal planum. Clinical signs consistent with DLE include depigmentation, erythema, scaling, and loss of the “cobblestone” appearance of the nasal planum. Alopecia, crusting, scaling, and ulcerations can also be seen on the face, muzzle, lips, pinnae, and periorbitally.

Truncal alopecia would be more common with endocrine disease, such as hypothyroidism or Cushing’s disease.

Discoid lupus is the cutaneous form, and the joints are not affected like they are with systemic lupus erythematosus (SLE).

How well did you know this?
1
Not at all
2
3
4
5
Perfectly
74
Q

A 5-year-old Doberman presents for chronic pruritus and recurrent bacterial skin infections. The patient has been on a 4-week course of Cephalexin and continues to get new skin lesions despite this treatment. You suspect methicillin resistance and recommend a skin culture. The owner declines a culture and would like you to select another antibiotic empirically. Which antibiotic would be the best choice considering side effects and efficacy?

Trimethoprim-sulfa
Chloramphenicol
Cefpodoxime (Simplicef)
Clindamycin
Cefovecin (Convenia)

A

Answer: Clindamycin

Explanation
Since you are suspicious of methicillin resistance, using any cephalosporins such as Convenia or Simplicef would not be useful. Trimethoprim-sulfa will probably be effective; however, this antibiotic should be avoided in Dobermans as they are prone to side effects such as anemia, thrombocytopenia, arthritis, and renal toxicity.

These side effects are due to a type-three hypersensitivity. Chloramphenicol may also be effective against the infection, but clindamycin would be a better choice, as it has fewer side effects. Chloramphenicol can cause peripheral neuropathy, gastrointestinal upset, bone marrow suppression, and hepatotoxicity. Additionally, chloramphenicol can also cause aplastic anemia in humans. Clindamycin can also cause gastrointestinal upset but seems to be better tolerated than chloramphenicol.

How well did you know this?
1
Not at all
2
3
4
5
Perfectly
75
Q

A 5-year-old Collie presents for erosive/ulcerative dermatitis of the nasal planum. The lesion is well-demarcated and confined to the nasal planum and philtrum. There is a loss of cobblestone appearance and hypopigmentation. Which of the following is top differential that would result in the above clinical signs?

Folliculitis
Discoid lupus erythematosus
Dermatophytosis
Zinc deficiency

A

Answer: Discoid lupus erythematosus

Explanation
Discoid lupus (along with Pemphigus foliaceus) are autoimmune conditions that can result in ulcerative nasal dermatitis and loss of cobblestone appearance.

Dermatophytosis (ringworm) often affects the muzzle. However, it rarely affects the nasal planum, because dermatophytes live in hair follicles and there are no hair follicles on the nasal planum. (This is also why it would not be folliculitis). Zinc-responsive dermatosis typically causes scaling and crusting but not usually erosions or ulcers.

The next step for this dog would be to biopsy.

76
Q

A 5-month-old Shih Tzu presents for greasy skin, pruritus, and mild hair loss of the face. You perform skin cytology and no bacteria or yeast are observed. What test, that is part of the minimum dermatological database, would you perform next?

Diet trial
Skin scrape
Intradermal skin test
Complete blood count
Bacterial culture

A

Answer: Skin scrape

Explanation
Demodex injai typically causes greasy seborrhea, especially on the face and dorsal trunk. Terriers and Shih Tzus may be predisposed. Some dogs may be pruritic. This mite is long and slender when compared with Demodex canis.

A diet trial and intradermal skin test may be indicated in the future if the skin scrape is negative. A bacterial culture may be needed if bacteria were seen on cytology. There is no indication to perform a CBC. None of the other tests would be considered part of a dermatological database.

77
Q

A 3 year old male Weimaraner presents to your clinic with complaints of anorexia, weight loss, and progressive exercise intolerance and dyspnea with a dry, hacking cough. On physical exam, he is cachectic (BCS 2/9), febrile (103.8 F, 39.9 C), and has diffuse peripheral lymphadenopathy. Thoracic radiographs show a diffuse nodular interstitial pattern in all lung fields. Cytology of a lymph node aspirate shows suppurative inflammation. What is the MOST likely diagnosis to explain all of these findings?

Blastomycosis (Blastomyces dermatiditis)
Lymphosarcoma
Kennel cough (Bordetella bronchiseptica)
Plague (Yersinia pestis)

A

Blastomycosis (Blastomyces dermatiditis)

Explanation
The correct answer is blastomycosis. These findings are most consistent with blastomycosis. Lymphoma would be a great differential but is less consistent with the nodular lung pattern and suppurative inflammation in the lymph node. Plague is an interesting differential for the lymphadenopathy and cytology findings but is not consistent with the lung pattern or history. This Weimaraner’s disease is too severe to be explained by kennel cough alone. While this diagnosis is sometimes made easy by visualization of organisms in aspirates or skin scrapings, it is not always seen as in this case. Remember, Blastomyces is seen as a broad-based budding yeast.

The next step to obtain a definitive diagnosis in this patient would be a transtracheal wash or bronchoalveolar lavage.

78
Q

What is the approximate resting energy requirement for a 25 kg dog?

800 kcal/day
500 kcal/day
1400 kcal/day
2100 kcal/day

A

Answer: 800 kcal/day

Explanation
The correct answer is 800 kcal/day. There are many formulas for calculating RER and they all should work out to be pretty close to this. The formula used here is body weight (kg) to the 0.75 power times 70. (70 * BW^0.75).

Another formula that has been recommended for this calculation is (30 * BW + 70). Note that the results will be different compared to the (70 * BW^0.75) formula but they both should get you close enough to the correct answer.

You will have access to an “on screen” calculator during the exam if necessary, but may want to remember multiple ways to do a calculation should there not be access to a “power” (^) function.

79
Q

Which are the most common serovars now thought to play a role in canine leptospirosis?

Icterohemorrhagiae, canicola, grippotyphosa
Grippotyphosa, pomona, bratislava
Pomona, bratislava, icterohemorrhagiae
Bratislava, canicola, icterohemorrhagiae

A

Answer: Grippotyphosa, pomona, bratislava

Explanation
The correct answer is grippotyphosa, pomona, and bratislava. Icterhemorrhagiae and canicola were the most common serovars isolated in the past. The leptospires penetrate mucous membranes or abraded skin and multiply in the blood stream and spread to organs. The antibody response usually limits the response to the renal tubular epithelial cells. Clinical signs include anorexia, pyrexia, vomiting, dehydration, PU/PD, anuria or oliguria. The standard diagnostic test is the microscopic agglutination test (MAT); be careful when interpreting titers. Remember, leptospirosis is zoonotic.

80
Q

A 9 year old Greyhound presents to your clinic with an ulcerating mast cell tumor, located subcutaneously over the left scapula. You immediately schedule the dog for a wide surgical excision after no evidence of metastatic disease is identified on a complete blood count, serum chemistry, thoracic radiographs, and abdominal ultrasound. Your technician asks you if there are any drugs she should avoid using in this dog. You tell her the following drugs are contraindicated in this patient:

Atracurium and acepromazine
Propofol and morphine
Ivermectin and thiopental
Acepromazine and ivermectin
Morphine and thiopental

A

Answer: Morphine and thiopental

Explanation
In this case, thiopental is contraindicated because the patient is a sighthound. Thiopental is an ultra-short acting barbiturate. Recovery depends on redistribution to tissues, including fat. Because sighthounds have very little fat, they have prolonged recoveries and greater complications with these drugs.

Morphine is contraindicated in this patient due to the ulcerating mast cell tumor. Morphine can cause histamine release and should be avoided in mast cell tumor patients.

81
Q

A 3-year old male neutered Labrador Retriever presents at your clinic for lethargy and inappetence of a few days in duration. On physical examination, the dog is approximately 7% dehydrated with pale and dry mucous membranes, a capillary refill time > 2 sec, heart rate > 170 bpm and weak femoral pulses. You diagnose hypovolemic shock and decide to start isotonic crystalloid fluid therapy. What is the isotonic crystalloid total “shock dose” in dogs?

90 ml/kg
10-20 ml/kg
5 ml/kg
180 ml/kg
40-60 ml/kg

A

Answer: 90 ml/kg

Explanation
The isotonic crystalloids shock dose in dogs is 90 ml/kg. 1/4 to 1/3 of this volume should be administered initially and the patient reassessed. In cats, the isotonic crystalloids shock dose is 40-60 ml/kg. Synthetic colloids can be administered at a dose of 10-20 ml/kg in dogs and 5-10 ml/kg in cats and hypertonic saline at a dose of 5 ml/kg.

82
Q

A 3-year old German Short Hair Pointer presents for vomiting, lethargy, and diarrhea. A fecal exam reveals the eggs of Nanophyetus salmincola, the salmon poisoning fluke. What treatment should be administered to treat the symptoms of the dog?

Tetracycline antibiotics
Fenbendazole
Penicillin
Praziquantel

A

Answer: Tetracycline antibiotics

Explanation
The correct answer is tetracycline antibiotics. The causative agent for the dog’s clinical signs is Neorickettsia helminthoeca. Treatment of choice for this rickettsial agent is tetracycline antibiotics. The fluke itself is not responsible for the clinical signs, but may be treated effectively with Praziquantel.

83
Q

An 8-year old female Chow Chow presents to you for difficulty eating. On oral exam, you see a 3 cm mass in the caudal maxilla. You are unable to obtain a fine needle aspirate because of the mass’s location but you detect an enlarged mandibular lymph node and aspirate it. A representative field from the aspirate is shown in the image below. In addition to surgery (+/- radiation), which of the following is an APPROVED treatment option for this disease that you would mention to the owner?

Doxorubicin
Palladia
ONCEPT vaccine!
Mitoxantrone
Vincristine

A

Answer: ONCEPT vaccine!

Explanation
The image shows a lymph node with metastatic melanoma (the large cell with black melanin pigment). The ONCEPT vaccine, also commonly referred to as the “melanoma vaccine” was given a conditional product license in 2007 and was USDA approved in 2010 as a therapeutic vaccine for cancer treatment.
Palladia is a receptor tyrosine kinase inhibitor approved for use in mast cell tumors in dogs. Doxorubicin, vincristine and mitoxantrone are human chemotherapy drugs commonly used off-label in veterinary oncology but not routinely for melanoma.

84
Q

A 3-year old German Shepherd presents for multiple subcutaneous nodules on the right front limb that have been present for about one week. The dog is otherwise healthy. The owner states that the dog lives mostly outdoors in a wooded area with ponds in Florida. You are highly suspicious of pythium; what is the best therapeutic option for this disease?

Ivermectin
Antifungal therapy
Lufenuron
Amputation and treatment with doxycycline
Amputation and treatment with an antifungal
Prednisone

A

Answer: Amputation and treatment with an antifungal

Pythium is an aquatic organism that inhabits ponds of the Southeast United States. These organisms have different cell walls than regular fungi, which makes them difficult to treat since most antifungal treatments target fungal cell wall synthesis. Prognosis is extremely poor with this disease and amputation (if the lesions are localized to an affected limb) is currently the best option. Amputation should always be followed with multiple long-term antifungal therapies. Owners should be warned that local postoperative recurrence is common. Dogs are most commonly affected, but this disease can also occur in cats. It has also been reported in humans and horses.

Doxycyline is an antibiotic and would not be effective for this disease. Prednisone would be contraindicated for pythium. Lufenuron is a chitin synthesis inhibitor used for flea control and would not be effective. Ivermectin is an anti-parasite drug and would not be effective.

85
Q

A 3 year old male Weimaraner presents to your clinic with complaints of anorexia, weight loss, and progressive exercise intolerance and dyspnea with a dry, hacking cough. On physical exam, he is cachectic (BCS 2/9), febrile (103.8 F, 39.9 C), and has diffuse peripheral lymphadenopathy. Thoracic radiographs show a diffuse nodular interstitial pattern in all lung fields. Cytology of a lymph node aspirate shows suppurative inflammation. What is the MOST likely diagnosis to explain all of these findings?

Kennel cough (Bordetella bronchiseptica)
Blastomycosis (Blastomyces dermatiditis)
Lymphosarcoma
Plague (Yersinia pestis)

A

Answer: Blastomycosis (Blastomyces dermatiditis)

Explanation
The correct answer is blastomycosis. These findings are most consistent with blastomycosis. Lymphoma would be a great differential but is less consistent with the nodular lung pattern and suppurative inflammation in the lymph node. Plague is an interesting differential for the lymphadenopathy and cytology findings but is not consistent with the lung pattern or history. This Weimaraner’s disease is too severe to be explained by kennel cough alone. While this diagnosis is sometimes made easy by visualization of organisms in aspirates or skin scrapings, it is not always seen as in this case. Remember, Blastomyces is seen as a broad-based budding yeast.

The next step to obtain a definitive diagnosis in this patient would be a transtracheal wash or bronchoalveolar lavage.

86
Q

You routinely give atropine as part of your pre-anesthetic protocol. Which of the following is an effect of atropine?

Increased drooling/salivation
Decreased gastrointestinal tone and motility
Treatment of ventricular premature complexes
Completely blocks the influence of the vagus nerve on the heart at moderate doses

A

Answer: Decreased gastrointestinal tone and motility

Explanation
Atropine is an anticholinergic, often used during premedication and intraoperatively to increase heart rate. It causes a decrease in tear secretion, salivary secretion, and Gl tone and motility. Atropine reduces the influence of the vagus nerve on the heart but does not block it completely. Lidocaine is used to treat VPCs.

87
Q

What adverse reaction are Dobermans predisposed to when using trimethoprim-sulfa?

Hypesenstvty reaction
Pancreatitis
Gastrointestinal ulcer formation
Vestibular disease

A

Answer: Hypesenstvty reaction

Explanation
The correct answer is hypersensitivity reaction. Doberman Pinschers are more susceptible to hypersensitivity reactions from sulfa drugs as compared to other breeds. Type Ill reactions can result in immune complex disease which can cause arthritis, nephritis, and uveitis. Other possible side effects from this class of antibiotics that are not specific to the Doberman include KCS, hepatitis, and blood dyscrasias.

88
Q

What region of the bone is affected in hypertrophic osteodystrophy?

Epiphysis
Diaphysis
Metaphysis
Physis

A

Answer: Metaphisis

Explanation
The correct answer is the metaphysis.
Hypertrophic osteodystrophy (HOD) is a condition seen in young, growing large breed dogs. It affects the metaphyses of all long bones and appears as an abnormal radiolucent line within the metaphysis. The disease is usually self-limiting but can cause limb deformities or systemic illness. The etiology is unknown but may be correlated with Ca/P levels and balance, vitamin C deficiency, and certain diseases including distemper.

89
Q

A 10 year old female spayed Labrador Retriever has presented for difficulty getting up in the hind and apparent back pain for the last 3-4 weeks. Upon questioning the owner, you are told that her appetite is diminished but she is still eating. An orthopedic exam finds no pain or discomfort elicited on manipulation of the hips or stifles. A neurologic exam identifies substantial pain in the lumbosacral region; however, conscious proprioception is intact, and patellar reflexes are normal. Radiographs of the lumbosacral region identify a lytic lesion at the L7-S1 endplates as well as surrounding bony proliferation. Which of the following diagnostic tests is likely to provide the most helpful additional information given your findings?

Urine culture
Chest radiographs
Stifle arthrocentesis
Blood draw for creatine kinase levels
Abdominal ultrasound

A

Answer: Urine culture

The diagnosis you should have in mind is discospondylitis. Be sure not to confuse this with spondylosis, which is typically not clinically significant and can be expected in most older dogs. The radiographic description is relatively specific for this condition. Neoplasia in the spine should be lower on your differential list because it typically does not cross joints. Disc herniation cannot be ruled out, and advanced imaging would be necessary to know for certain, but given the exam and radiographic findings, discospondylitis should be your top differential.

Discospondylitis is usually bacterial in origin, with Staphylococcus being the most common organism involved. Other organisms identified include Brucella canis, E. coli, Pasteurella, Aspergillus, and Streptococcus.

In an attempt to identify the causative agent, urine and blood cultures should be considered. The other answer choices are not as high yield in identifying the specific bacterial cause or underlying etiology.

90
Q

The following image shows a 3-D CT reconstruction of a 7-year old mixed breed small dog with a history of pain on opening the mouth for the past 2 weeks. The patient initially received a dental prophylaxis and cleaning under the assumption that the pain was related to moderate dental disease. When the pain persisted skull radiographs were performed and showed evidence of a mixed proliferative and lytic mass at the proximal aspect of the vertical ramus. What muscle attaches to this region?

Pterygoid
Digastricus
Masseter
Temporalis

A

Answer: Temporalis

The mass depicted in this three dimensional CT reconstruction involves the coronoid process. The ramus of the mandible consists of three distinct processes, the most dorsal being the coronoid process. It projects into the temporal fossa and provides an area for attachment of the temporalis muscle. The condylar (or condyloid) process is just ventral and slightly caudal to the coronoid process.

This process is a point of articulation between the maxilla (retroarticular process) and mandible. Disruption of this region will result in temporomandibular joint luxation. The most ventral process is the angular process. This structure provides an area for attachment of the digastricus muscle. The masseter muscle inserts laterally on the mandible, while the pterygoid muscle inserts medially. The masseter, pterygoid, and temporalis muscles function to close the mouth while the digastricus opens the mouth.

91
Q

A 2-year old female spayed Labrador Retriever presents for a right forelimb lameness and radial valgus in the right antebrachium. Which of the following is a possible therapeutic option for this dog?

Corrective osteotomy
Periosteal stripping of the concave aspect of the bone
Arthrodesis of the elbow
Casting the limb, activity restriction, and non-steroidal anti-inflammatory drugs

A

Answer: Corrective osteotomy

Explanation
The correct answer is corrective osteotomy.

Corrective osteotomies of the radius and ulna are often performed to relieve pain and the functional abnormalities induced by angular limb deformities.

92
Q

A dog arrives at your clinic that was bitten by a rabies suspect. This dog is unvaccinated what should you do?

Euthanize immediately or quarantine for 6 months and vaccinate 1 month before release
Euthanize immediately
Euthanize immediately or vaccinate immediately and quarantine for 45 days
Euthanize immediately and submit for testing or vaccinate immediately and quarantine for 4 months.
Vaccinate immediately and quarantine for 45 days

A

Answer: Euthanize immediately and submit for testing or vaccinate immediately and quarantine for 4 months.

Explanation
Per the 2016 Rabies Guidelines:
Since the dog is unvaccinated, it should either be euthanized and tissue submitted for rabies testing OR immediately vaccinated for rabies and quarantined for 4 months.

The Direct Fluorescent Antibody Test detects viral antigens and should be tested on two locations from the brain (brainstem and cerebellum) and is the test of choice for rabies diagnosis.

If the dog was current on rabies vaccination, the guidelines direct to booster the rabies vaccination immediately and have the owners observe closely for 45 days.

If owners are able to provide proof the dog had been previously vaccinated for rabies but is overdue: guidelines direct to booster rabies vaccination immediately and keep the dog under owner observation for 45 days.

93
Q

A common animal welfare concern for dogs living with families is the desire, on behalf of the owners, to do the following in order to shape dog behavior.

Overfeed
Physicaly punishingl
Crate
Over exercise
Restrain

A

Answer: Physicaly punishingl

Explanation
Owners physically punishing their dogs in an attempt to change the dog’s behavior is a common welfare concern.

94
Q

Zoonotic diseases commonly carried by raccoons include rabies and ______________
Toxocara
Baylisascaris
Echinococcus
Trichinella

A

Answer: Baylisascaris

Explanation
The correct answer is Baylisascaris. Baylisascaris procyonis is an ascarid parasite of raccoons that causes mild signs in raccoons but can undergo aberrant migration in humans and cause fatal central nervous system signs.

Baylisascaris Infection (Raccoon Roundworm): Comprehensive Veterinary Guide

Definition:
Baylisascaris infection, also known as raccoon roundworm infection, is caused by the nematode Baylisascaris procyonis. It primarily infects raccoons but can also affect humans and other animals, including dogs.

Pathophysiology:

•	Infection Mechanism: Humans and animals become infected by ingesting embryonated eggs from contaminated soil, water, or objects.
•	Larval Migration: Ingested larvae migrate through tissues, causing damage depending on the site of invasion, such as the brain (neural larva migrans), eyes (ocular larva migrans), or other organs (visceral larva migrans).

Causes:

•	Primary Host: Raccoons are the primary host, shedding eggs in their feces.
•	Transmission: Eggs become infectious in the environment after 2-4 weeks. Human infection usually occurs through accidental ingestion of contaminated material.

Symptoms:

•	General: Nausea, tiredness, liver enlargement.
•	Neurological: Loss of coordination, muscle control, and attention; potential for blindness or coma if the brain is affected.

Clinical Changes:

•	Infected individuals may exhibit neurological symptoms or damage to other organs, depending on larval migration pathways.

Diagnosis:

•	Challenges: Diagnosis is difficult due to the rarity of infection and nonspecific symptoms. It is often made by exclusion of other diseases.
•	Testing: No widely available tests; diagnosis may rely on patient history and clinical signs.

Treatment:

•	Medications: Albendazole, a broad-spectrum anthelmintic, is recommended, though not completely effective. Early treatment can help reduce severe damage.
•	Supportive Care: Symptomatic treatment based on the affected organs.

Prevention:

•	Environmental Management: Prompt removal of raccoon feces, avoiding contact with raccoons, and hygiene practices like handwashing can reduce the risk of infection.
95
Q

The 3-year old English Setter in the photos presents to you with severe pruritus affecting the face, axillary, and inguinal regions as well as the dorsal aspect of the paws. After a careful history and physical examination, you suspect the dog has atopy. What type of hypersensitivity (HPS) and what class of immunoglobulin are MOST IMPORTANT in the pathogenesis of atopic dermatitis in dogs?

Type IV HPS, IgE
Type IV HPS, IgA
Type I HPS, IgE
Type I HPS, IgA

A

Answer: Type I HPS, IgE

Atopic dermatitis is mediated primarly by IgE and is considered a Type I, or immediate, hypersensitivity reaction. In some species, IgG may also play a role.
It is interesting to note that not all atopic patients present with a typical Type 1 hypersensitivity. Some patients with atopic dermatitis do not have measurable IgE against environmental allergens. This syndrome is termed
“atopic-like.”

96
Q

A 7-year old female spayed Golden Retriever that you suspect has Addison’s disease has bloody diarrhea, inappetance, and is dehydrated. What should be your next step?

Start aggressive IV fluid therapy and run a chemistry panel
Give IV dexamethasone and subcutaneous
fluids
Give Lysodren
Run a chemistry panel and perform an abdominal ultrasound.

A

Answer: Start aggressive IV fluid therapy and run a chemistry panel

The correct answer is start aggressive IV fluid therapy and run a chemistry panel.
Addisonian patients often present in hypovolemic shock, so the first step to treating this dog is to restore vascular volume with IV fluids and to run a chemistry panel to check the extent of electrolyte abnormalities (hyponatremia, hyperkalemia, and elevated BUN). IV dexamethasone would not take precedence over restoring vascular volume, and subcutaneous fluid treatment is not aggressive enough for treatment of hypovolemia. Abdominal ultrasound would not be a priority in this case but could eventually be helpful in ruling in/out other differentials.
Lysodren is used for treating
hyperadrenocorticism. One possible complication of Lysodren treatment is to make a dog Addisonian, which often causes the animal to present with the signs described as in the question.

97
Q

A 1-year old bull dog mix presents for physical exam before breeding. Her physical exam is unremarkable with the exception of the abnormality seen in this image (prolapsed gland of the nictitans). What is the best method of treatment, and would you recommend breeding this bitch?
Surgical removal will be curative. Consider not breeding.
Surgical removal will be curative. Continue with breeding schedule.
Surgical replacement will be curative.
Continue with breeding schedule.
Surgical replacement will be curative. Consider not breeding.

A

Answer: Surgical replacement will be curative. Consider not breeding.

Prolapsed glands of the third eye lid (“cherry eye”) are congenital defects and will likely be passed on to future generations. Repair is straight forward with prolapsed gland replacement and tacking to periorbital rim (Kaswan anchoring technique) or using the Morgan pocket technique. Surgical removal is NOT recommend as you will remove the tear gland and predispose to iatrogenic keratoconjunctivits sicca.

98
Q

Which of these biochemical profiles are most consistent with the diagnosis of insulinoma in a dog?

Blood glucose=45 mg/dl, Serum insulin-high
Blood glucose=150 mg/dl, Serum insulin-high
Blood glucose=45 mg/dl, Serum insulin-low
Blood glucose=150 mg/dl, Serum insulin-low

A

Answer: Blood glucose=45 mg/dl, Serum insulin-high

Explanation
The correct answer is blood glucose-45 mg/dl, serum insulin-high. An insulinoma can result in an animal having normal to high insulin levels in the face of low blood glucose levels because the normal controls of insulin secretion are lost.

Comprehensive NAVLE Preparation: Insulinoma in Dogs
Definitions & Pathophysiology:
• Insulinoma: A rare, malignant, functional pancreatic β-cell tumor in dogs, secreting insulin independent of blood glucose levels. This results in persistent hypoglycemia and its associated clinical signs.
• Pathophysiology: The neoplastic β-cells lose their ability to regulate insulin secretion in response to hypoglycemia. This unregulated insulin release decreases blood glucose levels, leading to neuroglycopenic and adrenergic effects.

Causes:
• Insulinomas are malignant and often metastasize to lymph nodes and the liver. The exact cause is unknown, but it is most common in middle-aged to older dogs, with no specific sex predilection. Breeds like Golden Retrievers, Labrador Retrievers, and Boxers are overrepresented.

Clinical Signs:
• Early Signs: Episodic weakness, lethargy, ataxia, collapse, and seizures. These signs are often precipitated by fasting, exercise, or excitement.
• Advanced Signs: Prolonged or severe hypoglycemia can result in coma or death. Peripheral polyneuropathy may also occur, leading to muscle atrophy, paresis, and decreased reflexes.

Diagnosis:
• Whipple’s Triad: (1) Documented hypoglycemia (<70 mg/dL), (2) Clinical signs consistent with hypoglycemia, (3) Resolution of signs after glucose administration.
• Laboratory Findings: Persistent hypoglycemia with inappropriate normal to high insulin levels.
• Advanced Imaging: Abdominal ultrasound, CT, and MRI are used to identify pancreatic masses. Somatostatin receptor scintigraphy (SRS) may detect metastases.

Differential Diagnosis:
• Other causes of hypoglycemia include hypoadrenocorticism, hepatic insufficiency, sepsis, and iatrogenic causes like insulin overdose.

Treatment:
• Surgical Management: Partial pancreatectomy to remove the primary tumor and resect metastases when possible. Surgical removal improves survival times significantly but is rarely curative due to the high rate of metastasis.
• Medical Management:
• Dietary: Frequent meals high in protein, fats, and complex carbohydrates to minimize insulin secretion.
• Medications:
• Prednisone (0.5 mg/kg/day): Increases blood glucose by promoting gluconeogenesis and glycogenolysis.
• Diazoxide (5 mg/kg PO q12h): Inhibits insulin release, promotes glycogenolysis, and decreases cellular glucose uptake.
• Octreotide (2-4 μg/kg SC q8-12h): A somatostatin analog that inhibits insulin secretion.
• Glucagon (CRI 5 ng/kg/min): Used in refractory hypoglycemia to maintain euglycemia.
• Streptozocin (500 mg/m² IV q3 weeks with diuresis): Chemotherapeutic agent targeting pancreatic β-cells, used in cases with metastasis.

Prognosis:
• Surgical Resection: Median survival time is 12-14 months post-surgery, with survival rates higher in dogs without metastasis.
• Medical Management Alone: Median survival time is around 6 months. However, combining surgery and medical therapy can extend survival to 2-3 years.

Important Points for NAVLE:
• Recognize clinical signs associated with hypoglycemia and the significance of Whipple’s Triad in diagnosis.
• Understand the role of advanced imaging and laboratory diagnostics in identifying insulinomas.
• Familiarize yourself with both surgical and medical management strategies, emphasizing the importance of managing hypoglycemia and the potential for metastasis.
• Be aware of the drugs used in managing insulinoma and their mechanisms, including glucocorticoids, diazoxide, and somatostatin analogs, as well as the use of chemotherapeutic agents in metastatic cases.

99
Q

You are seeing a dog that you suspect has iatrogenic Cushing’s disease from high-dose prednisone administration. What would you expect on blood samples submitted for ACTH and cortisol levels submitted to a commercial lab?

ACTH low, cortisol high
ACTH low, cortisol low
ACTH high, cortisol low
ACTH high, cortisol high

A

Answer: ACTH low, cortisol high

Explanation
The correct answer is ACTH low, cortisol high. Prednisone is a synthetic corticosteroid that cross reacts with assays measuring endogenous cortisol levels, causing an artificially elevated cortisol measurement on blood tests. The feedback mechanism on the pituitary gland inhibits ACTH production, making it low.

ACTH low, cortisol high is expected in a dog with iatrogenic Cushing’s disease from high-dose prednisone. Prednisone, a synthetic corticosteroid, cross-reacts with cortisol assays, resulting in artificially elevated cortisol levels. The negative feedback on the pituitary gland suppresses ACTH production, leading to low ACTH levels.

100
Q

The best diagnostic test to differentiate pituitary dependent hyperadrenocorticism from adrenal dependent hyperadrenocorticism in dogs is which of the following?

Urine cortisol: creatinine ratio
Serum cortisol
X Low-dose dexamethasone suppression test
High-dose dexamethasone suppression test

A

Answer: High-dose dexamethasone suppression test

Explanation
The correct answer is high-dose dexamethasone suppression test. HDDST will suppress cortisol secretion in about 75% of PDH patients 3-6 hours post dexamethasone administration. Cortisol secretion does not become suppressed with dexamethasone administration with adrenal dependent hyperadrenocorticism. If cortisol secretion is suppressed with a HDDST, PDH is diagnosed. If cortisol is not suppressed, there is a 50-50 chance that the hyperadrenocorticism is due to PDH or an adrenal tumor.

LDDST will suppress cortisol secretion in approximately 65% of pituitary dependent hyperadrenocorticism patients. The urine cortisol:creatinine ratio measures the significance of urine cortisol concentration and may be increased with both PDH and ADH. Serum cortisol levels may be affected in many different situations such as stress; therefore they are an unreliable indicator of disease. An elevated serum cortisol level will not differentiate between PDH and ADH.

High-dose dexamethasone suppression test (HDDST) is the best diagnostic test to differentiate pituitary-dependent hyperadrenocorticism (PDH) from adrenal-dependent hyperadrenocorticism (ADH) in dogs. In about 75% of PDH cases, cortisol secretion is suppressed 3-6 hours post-dexamethasone administration, confirming PDH. In contrast, cortisol secretion remains unsuppressed in ADH. If suppression occurs, PDH is diagnosed. If not, the hyperadrenocorticism could be either PDH or an adrenal tumor, with a 50-50 chance. Other tests, like LDDST, urine cortisol: creatinine ratio, and serum cortisol levels, are less reliable for distinguishing between PDH and ADH.

101
Q

A 7-year old female spayed Golden Retriever that you suspect has Addison’s disease has bloody diarrhea, inappetance, and is dehydrated. What should be your next step?

Give Lysodren
Start aggressive IV fluid therapy and run a chemistry panel
Give IV dexamethasone and subcutaneous fluids
Run a chemistry panel and perform an abdominal ultrasound.

A

Answer: Start aggressive IV fluid therapy and run a chemistry panel

Explanation
The correct answer is start aggressive IV fluid therapy and run a chemistry panel.
Addisonian patients often present in hypovolemic shock, so the first step to treating this dog is to restore vascular volume with IV fluids and to run a chemistry panel to check the extent of electrolyte abnormalities (hyponatremia, hyperkalemia, and elevated BUN). IV dexamethasone would not take precedence over restoring vascular volume, and subcutaneous fluid treatment is not aggressive enough for treatment of hypovolemia.

Abdominal ultrasound would not be a priority in this case but could eventually be helpful in ruling in/out other differentials.

Lysodren is used for treating hyperadrenocorticism. One possible complication of Lysodren treatment is to make a dog Addisonian, which often causes the animal to present with the signs described as in the question.

Start aggressive IV fluid therapy and run a chemistry panel for a dog suspected of having Addison’s disease. Addisonian patients often present in hypovolemic shock, so restoring vascular volume with IV fluids is the top priority. The chemistry panel will help assess electrolyte imbalances, such as hyponatremia, hyperkalemia, and elevated BUN. IV dexamethasone and subcutaneous fluids are insufficient for treating hypovolemia, and abdominal ultrasound, while useful later, is not a priority. Lysodren is for hyperadrenocorticism treatment and not appropriate here.

102
Q

Administration of which of the following would not adversely affect the results of an ACTH stimulation test?

Prednisone
Flucortisone acetate
Cortisone
Dexamethasone
Prednisolone

A

Answer: Dexamethasone

Explanation
The correct answer is dexamethasone. Administration of dexamethasone or desoxycorticosterone acetate (DOCA) prior to ACTH stimulation test does not confound the test results. Administration of prednisone, cortisone, prednisolone, and flucortisone acetate would show false elevations in test results.

Dexamethasone administration would not adversely affect the results of an ACTH stimulation test. Unlike prednisone, cortisone, prednisolone, and flucortisone acetate, which can cause false elevations, dexamethasone and desoxycorticosterone acetate (DOCA) do not interfere with the test, making them safe to use before the ACTH stimulation test.

103
Q

You have a 6-year old female spayed Golden Retriever patient with inappetence and vomiting. You diagnose diabetic ketoacidosis based on a blood glucose of 641 mg/dL (76-119 mg/dL), 3+ glucose (normal- negative) and 3+ ketones (normal- negative) in the urine, and blood pH of 7.12 (7.35-7.45). You would like to treat with the shortest acting, most potent insulin type. Which of the following would you choose?

NPH
Lente
PZI
Regular
Ultralente

A

Answer: Regular

Explanation
Listed from shortest acting and most potent to longest acting and least potent: Regular, NPH, Lente, PZI, Ultralente. The treatment of DKA involves IV fluids and regular insulin, given either by CRI or the intermittent IM dosing technique. Regular insulin is continued until the patient is eating well and is hydrated, and can be switched to a longer-acting insulin, to be given SQ at home.

Regular insulin is the best choice for treating diabetic ketoacidosis (DKA) in this case. Regular insulin is the shortest acting and most potent, ideal for managing DKA. Treatment involves IV fluids and regular insulin, administered via continuous rate infusion (CRI) or intermittent IM dosing. Once the patient stabilizes, eating well, and hydrated, they can be transitioned to a longer-acting insulin for home use.

Comprehensive NAVLE Study Guide: Diabetic Ketoacidosis (DKA) in Small Animals
Definitions and Etiology
• Diabetic Ketoacidosis (DKA): A life-threatening complication of diabetes mellitus characterized by hyperglycemia, ketonemia, and metabolic acidosis.
• Causes: DKA occurs due to absolute or relative insulin deficiency, often precipitated by concurrent illness, such as infection, pancreatitis, or neoplasia.

Pathophysiology

•	Insulin Deficiency: Leads to hyperglycemia due to increased gluconeogenesis, glycogenolysis, and decreased glucose utilization.
•	Ketogenesis: In the absence of insulin, free fatty acids are converted into ketones in the liver. Accumulation of ketones (e.g., β-hydroxybutyrate, acetoacetate) results in metabolic acidosis.
•	Counterregulatory Hormones: Glucagon, cortisol, epinephrine, and growth hormone exacerbate hyperglycemia and ketogenesis by promoting lipolysis and gluconeogenesis.
•	Electrolyte and Acid-Base Disturbances: Significant fluid and electrolyte losses occur due to osmotic diuresis, leading to dehydration, hypokalemia, and metabolic acidosis.

Clinical Signs
• History: Polyuria, polydipsia, weight loss, and acute symptoms like vomiting, diarrhea, anorexia, lethargy, and dehydration.
• Physical Examination: Tachycardia, weak pulses, poor skin turgor, dry mucous membranes, Kussmaul respirations (deep and labored breathing), and possible neurologic signs like weakness or altered mentation.

Diagnosis
• Laboratory Tests:
• Blood Glucose: Marked hyperglycemia (>300 mg/dL).
• Ketones: Detected in blood and urine (β-hydroxybutyrate being the primary ketone in blood).
• Electrolytes: Hyponatremia, hypokalemia, and hypophosphatemia common.
• Acid-Base Status: Metabolic acidosis (pH < 7.3, bicarbonate < 15 mEq/L).
• Serum Osmolality: Increased, especially in cases of hyperglycemic hyperosmolar syndrome (HHS).

Treatment
• Fluid Therapy:
• Rehydration: Critical initial step; typically with isotonic crystalloids (e.g., 0.9% NaCl). Adjustments may be needed based on electrolyte status.
• Monitoring: Frequent reassessment of hydration status, electrolytes, and acid-base balance.
• Insulin Therapy:
• Regular Insulin: Administered initially as a low-dose continuous rate infusion (CRI) or intermittent intramuscular injections.
• Glucose Monitoring: Target glucose reduction of 50-75 mg/dL per hour; once glucose <250-300 mg/dL, add dextrose to fluids to prevent hypoglycemia while continuing insulin to resolve ketonemia.
• Electrolyte Management:
• Potassium: Replacement essential as insulin therapy shifts potassium intracellularly. Monitoring is crucial to avoid life-threatening hypokalemia.
• Phosphorus: Monitor and replace if hypophosphatemia is severe, as it may lead to hemolysis and weakness.
• Bicarbonate: Generally not recommended unless severe acidosis (pH < 7) persists after fluid therapy.
• Addressing Concurrent Illnesses: Identifying and treating underlying causes such as infections, pancreatitis, or other comorbidities is essential for successful management.

Complications
• Cerebral Edema: Rare but severe complication, particularly in overzealous fluid or insulin therapy. It necessitates careful monitoring and gradual correction of hyperglycemia and electrolyte imbalances.
• Hypoglycemia: Can occur if insulin therapy is not carefully titrated.

Prognosis
• Mortality: Depends on the severity of the initial presentation and the presence of concurrent diseases. Mortality rates for DKA range from 20-30%, with worse outcomes in cases complicated by HHS.

104
Q

What is the primary goal in the initial treatment of diabetic ketoacidotic dogs?

Getting the animal to eat
Controlling obesity or concurrent disorders causing insulin resistance
Lowering the elevated blood glucose
Correction of acidosis, electrolyte abnormalities, and hyperosmolality

A

Answer: Correction of acidosis, electrolyte abnormalities, and hyperosmolality

Explanation
The correct answer is correction of acidosis, electrolyte abnormalities, and hyperosmolality. Initial treatment of DKA patients is aimed at correcting acidosis by administering bicarbonate (if the total bicarbonate is markedly low), correcting electrolyte abnormalities with IV fluids, potassium and phosphorus supplementation, and correction of hyperosmolality. Regulating blood glucose, getting the animal to eat, controlling obesity and concurrent disease should be addressed after immediate life-threatening issues are resolved.

Correction of acidosis, electrolyte abnormalities, and hyperosmolality is the primary goal in the initial treatment of diabetic ketoacidotic (DKA) dogs. Treatment involves administering bicarbonate for severe acidosis, correcting electrolyte imbalances with IV fluids, potassium, and phosphorus supplementation, and addressing hyperosmolality. Once these life-threatening issues are managed, focus can shift to regulating blood glucose, encouraging eating, and addressing obesity or concurrent disorders.

105
Q

Pheochromocytomas are not very common in dogs; however, their presence can result in hypertension, tachyarrhythmias, seizures, and collapse. What is the substance secreted by this tumor?

Aldosterone
Cortisol
Calcium
Estrogen
Catecholamines

A

Answer: **Catecholamines*

Explanation
A pheochromocytoma is a tumor of the adrenal medulla. To review, the adrenal gland is composed of the adrenal cortex (outer layer) which is divided into the zona glomerulosa, zona fasciculata, and zona reticularis.

The zona glomerulosa is the main site of aldosterone production. The zona fasciculata produces glucocorticoids (mainly cortisol). The zona reticularis is the site of androgen production.

The adrenal medulla is the inner part of the adrenal gland and consists of chromaffin cells. These cells are responsible for making catecholamines (epinephrine and norepinephrine).

Catecholamines are secreted by pheochromocytomas, which are tumors of the adrenal medulla. The adrenal gland consists of the adrenal cortex and adrenal medulla. The adrenal medulla, composed of chromaffin cells, produces catecholamines like epinephrine and norepinephrine. These hormones are responsible for the hypertension, tachyarrhythmias, seizures, and collapse seen in affected dogs. Other substances like aldosterone, cortisol, calcium, and estrogen are produced in different parts of the adrenal gland but are not associated with pheochromocytomas.

Comprehensive NAVLE Study Guide: Pheochromocytoma in Dogs
Definition and Etiology
• Pheochromocytoma: A catecholamine-producing tumor that originates from the adrenal medulla or, less commonly, extra-adrenal chromaffin tissue.
• Causative Agent: Tumors derived from chromaffin cells that secrete excessive catecholamines, mainly norepinephrine, and epinephrine.
• Pathogenesis: Tumors cause excessive and unpredictable catecholamine release, leading to variable clinical signs. These tumors can be slow-growing but often malignant, with a potential for local invasion and metastasis.

Clinical Signs
• Signs of Catecholamine Excess: Tachycardia, hypertension, tachypnea, panting, cardiac arrhythmias, and episodic collapse.
• Nonspecific Signs: Anorexia, weight loss, lethargy, and episodic weakness.
• Neurological Signs: Anxiety, muscle tremors, pacing, and seizures.
• Signs from Tumor Size/Invasiveness: Abdominal distension, ascites, hind-limb edema, and potential retroperitoneal hemorrhage from tumor rupture.

Diagnosis
• Clinical Suspicion: Based on clinical signs and detection of an adrenal mass via ultrasonography.
• Urine Normetanephrine:Creatinine Ratio: Elevated levels suggest pheochromocytoma.
• Imaging: Abdominal ultrasonography, CT, and MRI for tumor localization and assessing invasiveness.
• Blood Pressure Measurement: Persistent hypertension is a hallmark but is only present in ~50% of cases.

Differential Diagnoses
• Other Adrenal Tumors: Cortisol-producing tumors, aldosteronoma, non-functional masses like myelolipoma or cysts.
• Systemic Diseases: Conditions affecting the cardiovascular, neuromuscular, and endocrine systems should be considered.

Treatment
• Adrenalectomy: Surgical removal of the tumor is the treatment of choice.
• Preoperative Management: Phenoxybenzamine (α-adrenergic blocker) to reduce perioperative mortality by controlling hypertension and vasoconstriction.
• Medical Management: For non-resectable tumors or metastasis, phenoxybenzamine is used to manage clinical signs without affecting tumor growth.

Prognosis
• Surgical Outcome: Favorable if perioperative complications are managed. Long-term survival is possible.
• Medical Management: Limited survival, generally less than a year, with clinical signs controlled but not tumor progression.

106
Q

What tumor type is most frequently implicated in causing feminization syndrome in dogs including gynecomastia, sexual attraction of males, and bilaterally symmetric alopecia?

Mammary carcinoma
Sertoli cell tumor
Seminoma
Leydig cell tumor

A

Answer: Sertoli cell tumor

Explanation
The correct answer is Sertoli cell tumors. The tumor sometimes secretes estrogen and other hormones, which can lead to this syndrome.

Sertoli cell tumors are most frequently associated with feminization syndrome in dogs, including symptoms like gynecomastia, sexual attraction of males, and bilaterally symmetric alopecia. These tumors can secrete estrogen and other hormones, leading to the observed clinical signs.

107
Q

A 6-year old male castrated Borzoi dog presents to you with a one-year history of polyuria and polydipsia with no other clinical signs. A urinalysis confirms a specific gravity of 1.003 and no other abnormalities.

Serum chemistry shows:
Ca 10.1 mg/dl (9.1-11.7 mg/di)
P 3.2 mg/dl (2.9-5.3 mg/dl)
bilirubin O.l mg/dl (0-0.3 mg/dl)
albumin 2.9 g/dl (2.3-3.1 g/dl)
globulin 2.5 g/dl (2.7-4.4 g/di)
ALT 40 U/L (10-109 U/L)
glucose104 mg/dl (76-119 mg/di)
BUN 8 mg/dl (8-28 mg/dl)
Creatinine 0.6 mg/dl (0.5-1.7 mg/dl)
cholesterol 120 mg/dl (135-278 mg/dl)

You tentatively diagnose central diabetes insipidus and elect to try the dog on desmopressin (DDAVP) to see if the PU/PD resolves. Which of the following reasons best explains why you might not see a response within the first 24-48 hours, even if your diagnosis is correct?

It may take up to 3 days to reactivate the antidiuretic hormone (ADH) receptors in the kidneys
Trauma to the hypothalamus or pituitary causing CDI will never respond to antidiuretic hormone (ADH)
It may take up to 3 days to overcome medullary washout from being polyuric and polydipsic
It takes a few days for the hypothalamus to respond to DDAVP and to start making antidiuretic hormone
(ADH)

A

Answer: It may take up to 3 days to overcome medullary washout from being polyuric and polydipsic

The correct answer is it may take up to 3 days to overcome medullary washout from being polyuric and polydipsic. DAVP is a synthetic ADH used to replace the lacking endogenous hormone. DDAVP does not induce the hypothalamus to make ADH. Medullary washout commonly occurs with CDI from prolonged PU and PD. It may take up to 3 days to get the medullary sodium concentration and concentrating mechanisms back in order. ADH receptors in the kidneys are not affected in CDI.

It may take up to 3 days to overcome medullary washout from being polyuric and polydipsic. In central diabetes insipidus (CDI), prolonged polyuria and polydipsia can lead to medullary washout, impairing the kidney’s ability to concentrate urine. Desmopressin (DDAVP) is a synthetic ADH replacement, but it may take up to 3 days for the medullary sodium concentration and concentrating mechanisms to recover. ADH receptors in the kidneys are not affected in CDI, and DDAVP does not stimulate the hypothalamus to produce ADH.

Diabetes Insipidus in Animals: Comprehensive Veterinary Guide
Definition:
Diabetes insipidus (DI) is a disorder characterized by the inability to concentrate urine, resulting in excessive urination (polyuria) and thirst (polydipsia). It is caused by a deficiency in antidiuretic hormone (ADH) or the kidneys’ inability to respond to ADH.

Forms:
1. Central DI: Caused by inadequate production or secretion of ADH due to damage to the hypothalamus, infundibular stalk, or pars nervosa, often from trauma, tumors, or inflammatory lesions.
2. Nephrogenic DI: Due to the kidneys’ failure to respond to ADH, often secondary to genetic mutations or other renal dysfunctions.

Pathophysiology:
• Central DI: Results from damage to the hypothalamus or pituitary gland, reducing ADH production.
• Nephrogenic DI: The kidneys’ receptors for ADH are dysfunctional, preventing water reabsorption despite normal or elevated ADH levels.

Symptoms:
• Chronic polyuria (large volumes of dilute urine)
• Polydipsia (excessive water intake)
• Dehydration signs if water intake is insufficient
Diagnosis:
• Clinical Signs: Persistent polyuria and polydipsia not due to other causes like diabetes mellitus.
• Water Deprivation Test: To evaluate the body’s ability to concentrate urine under dehydration. Caution is needed to prevent severe dehydration.
• ADH Response Test: Measures urine concentration response to exogenous ADH, distinguishing between central and nephrogenic DI.

Treatment:
• Central DI: Desmopressin acetate, a synthetic analogue of ADH, is administered to control polyuria and polydipsia. Dosage is tailored to achieve the minimal effective dose, typically given intranasally or orally.
• Nephrogenic DI: Managed with dietary changes, thiazide diuretics, and ensuring constant water availability.

Key Considerations:
• Differential Diagnosis: Exclude other causes of polyuria such as diabetes mellitus, chronic kidney disease, and psychogenic polydipsia.
• Prognosis: Lifelong management is typically required, focusing on symptom control and preventing complications like dehydration.

108
Q

You have been treating a 10-year-old miniature poodle for diabetes. She has always been well-regulated until now. Which of the following is a common problem leading to poor glycemic control in dogs with diabetes mellitus?

Continuous deposition of amyloid in pancreatic islets
Feeding canned food
Concurrent endocrine disease
Keeping insulin in the refrigerator

A

Answer: Concurrent endocrine disease

Explanation
The correct answer is concurrent endocrine disease. Concurrent diseases such as infection, hyperadrenocorticism, and pancreatitis can commonly cause increased insulin resistance, making diabetes difficult to regulate.

Amyloid deposition is a cause for DM in cats but does not change the effectiveness of insulin therapy. DM in dogs is not caused by amyloid deposition. Most insulins are supposed to be kept in the refrigerator. Feeding canned food should not contribute, so long as the diet is of good quality and adequate for diabetic regulation.

Owners will commonly overdose their animals by administering additional insulin than what had been prescribed or administering insulin when the animal has not eaten. Improper storage, handling, and administration are other common causes of ineffective insulin therapy.

Concurrent endocrine disease is a common problem leading to poor glycemic control in dogs with diabetes mellitus. Conditions like infection, hyperadrenocorticism, and pancreatitis can increase insulin resistance, making diabetes harder to manage. Amyloid deposition affects cats, not dogs, and does not impact insulin therapy effectiveness. Proper insulin storage, including refrigeration, and a quality diet are essential but generally do not cause poor control. Overdosing or improper insulin administration by owners can also contribute to ineffective regulation.

109
Q

A 6-year old female spayed English Pointer presents for lethargy and weight gain. The owner notes that the dog is eating and drinking a normal amount, but the dog is still gaining weight. A physical exam reveals weak pelvic limbs, facial nerve paralysis, a symmetrically patchy haircoat, and seborrhea. Lab work reveals a normocytic, normochromic anemia with a PCV of 29% (35-57%), lipemic serum, and cholesterol of 1090 mg/dl (135-278 mg/dl). What is the most likely diagnosis?

Hypothyroidism
Hyperthyroidism
Pituitary dependent hyperadrenocorticism
Adrenal dependent hyperadrenocorticism

A

Answer: Hypothyroidism

Explanation
The correct answer is hypothyroidism. The clinical signs, physical exam findings, and lab work abnormalities are classical for hypothyroidism. Other common abnormalities seen with hypothyroidism include pyoderma, neuromuscular signs (ataxia, knuckling, vestibular signs, etc), markedly elevated triglycerides, and a mild normocytic, normochromic anemia. Hyperthyroidism rarely occurs in dogs. You would expect to see polyuria, polydipsia, and polyphagia with hyperadrenocorticism.

Hypothyroidism is the most likely diagnosis. The dog’s clinical signs, including weight gain despite normal eating, weak pelvic limbs, facial nerve paralysis, and a patchy haircoat, along with lab findings of normocytic, normochromic anemia and significantly elevated cholesterol, are classic for hypothyroidism. Hyperthyroidism is rare in dogs, and hyperadrenocorticism would typically present with polyuria, polydipsia, and polyphagia, not seen here.

110
Q

A 7-year old female Standard Poodle presents to you for acute onset of vomiting and weakness after the owners returned from vacation. On physical exam, you estimate that the dog is 7% dehydrated, has a respiratory rate of 36 breaths per minute, heart rate of 44 beats per minute and temperature of 99 degrees F. You quickly attach ECG leads and expect to see which of the following abnormalities?

Inverted T waves
Tall, narrow QRS complexes
No P waves
Ventricular tachycardia

A

Answer:No P waves

Explanation
Based on the history and the physical exam findings of dehydration and bradycardia, you should be concerned about hypoadrenocorticism. Hypoadrenocorticism patients are typically hyperkalemic. ECG abnormalities with hyperkalemia include wide, flat, or absent P waves, widened QRS complexes, tall spiking T waves, and bradycardia.

No P waves is the expected ECG finding. The dog’s history and physical exam, including dehydration and bradycardia, suggest hypoadrenocorticism, commonly associated with hyperkalemia. Hyperkalemia can cause ECG abnormalities such as wide, flat, or absent P waves, widened QRS complexes, tall spiking T waves, and bradycardia.

111
Q

What test should be run to monitor the efficacy of treatment for an Addisonian dog being treated with supplemental mineralocorticoids and glucocorticoids?

Serum chloride and phosphorus
ACTH stimulation test
Resting cortisol level
Serum sodium and potassium
Urine cortisol creatinine ratio

A

Answer: Serum sodium and potassium

Explanation
The correct answer is serum sodium and potassium. Addisonian patients deficient in mineralocorticoids will usually exhibit the classic findings of hyponatremia and hyperkalemia. Monitoring serum sodium and potassium concentration while treating with exogenous mineralocorticoids is the best way to determine if the level and frequency of dosage is adequate for the patient.

An ACTH stimulation test can be used to monitor the efficacy of treatment of a patient with Cushing’s disease, but does nothing for an Addisonian.

Serum sodium and potassium should be monitored to assess the efficacy of treatment in an Addisonian dog receiving mineralocorticoids and glucocorticoids. Addisonian patients often present with hyponatremia and hyperkalemia, so monitoring these electrolytes ensures that the dosage and frequency of mineralocorticoid supplementation are appropriate. The ACTH stimulation test is more relevant for monitoring Cushing’s disease, not Addison’s.

112
Q

Which of the following is true about post-operative care in a dog that had a parathyroid adenoma removed?

Serum calcium post-operatively should be kept below the normal range to stimulate the production of
PTH from the atrophied chief cells of the normal parathyroid tissue.
Post-operative care is minimal and requires checking serum calcium once a week for one month.
The patient is less likely to be hypocalcemic post-operatively if the patient’s pre-operative serum calcium is markedly high.
The patient’s serum calcium should be monitored daily for about a week to check for the development of hypocalcemia.
It is most important to monitor serum calcium post-operatively to make sure the patient does not continue to be hypercalcemic.

A

Answer: The patient’s serum calcium should be monitored daily for about a week to check for the development of hypocalcemia.

Explanation
The correct answer is the patient’s serum calcium should be monitored daily for about a week to check for the development of hypocalcemia. Post-operatively, it is important to monitor for hypocalcemia daily for the first seven days regardless of the absence of clinical signs for hypocalcemia. The higher the pre-operative serum calcium concentration, the more likely the patient will become hypocalcemic post-operatively. Post-operative monitoring of serum calcium should be daily for the first seven days, then weekly for 4 weeks. Vitamin D and calcium supplementation should be given accordingly. Serum calcium concentration should be maintained in the low normal range, not below normal, to stimulate production of PTH by the parathyroid cells.

The patient’s serum calcium should be monitored daily for about a week to check for the development of hypocalcemia. After parathyroid adenoma removal, hypocalcemia can occur, especially if the pre-operative calcium levels were high. Daily monitoring for the first seven days is crucial, followed by weekly checks for four weeks. Calcium and vitamin D supplementation may be necessary, and serum calcium should be maintained in the low normal range to stimulate PTH production by the parathyroid cells.

113
Q

Which statement is true regarding vasopressin (anti-diuretic hormone or ADH) and central diabetes insipidus in dogs?

The production and release of ADH (vasopressin) is controlled by serum osmolality and blood volume in a normal dog.
The patient’s hypothalamus and posterior pituitary produces and releases too much ADH.
The patient’s urine specific gravity is usually 1.015-1.020.
The kidneys are not responsive to ADH.

A

Answer: The production and release of ADH (vasopressin) is controlled by serum osmolality and blood volume in a normal dog.

Explanation
The correct answer is the production and release of ADH (vasopressin) is controlled by serum osmolality and blood volume in a normal dog. With central diabetes insipidus, the patient has a problem with the function of the hypothalamus (ADH production) and/or neurohypophysis/posterior pituitary (ADH release) so the patient cannot respond to the changes in serum osmolality or blood volume and pressure. The urine specific gravity of a CDI patient is usually 1.001-1.007. The kidneys have no problem responding to ADH in CDI patients. There is a lack of the hormone, not an over-abundance.

The production and release of ADH (vasopressin) is controlled by serum osmolality and blood volume in a normal dog. In central diabetes insipidus (CDI), the hypothalamus or posterior pituitary fails to produce or release enough ADH, so the dog cannot respond to changes in serum osmolality or blood volume. CDI patients typically have a urine specific gravity of 1.001-1.007, and their kidneys are responsive to ADH, but there is a lack of the hormone, not an excess.

114
Q

An 8-year old mixed breed male neutered dog presents with a several-month history of lethargy, hair loss, and greasy skin. The patient’s face, feet, and tail are not clinically affected and he is not pruritic. What is the most likely cause of the hair loss?

Sarcoptes
Flea allergy
Malignant melanoma
Endocrinopathy

A

Answer: Endocrinopathy

Explanation
The lesion distribution is most consistent with endocrinopathy; flea allergy and sarcoptic mange would be extremely pruritic in a patient with these lesions. The dark coloration, although alarming to clients, reflects hyperpigmentation associated with inflammation, not neoplasia.

Endocrinopathy is the most likely cause of the hair loss. The lesion distribution, along with the absence of pruritus and involvement of the face, feet, and tail, is typical for an endocrine disorder. Flea allergy and sarcoptic mange would cause significant itching, and the hyperpigmentation is due to inflammation, not neoplasia like malignant melanoma.

115
Q

You are treating an 8-year old mix breed terrier for ventricular tachycardia post splenectomy. You determine that you’ve given too much lidocaine to the patient. What clinical signs or laboratory findings are you most likely to see initially?

Muscle tremors and seizures
Bradycardia and hypotension
Increased respiratory rate and difficulty breathing
Methemoglobinemia
Diarrhea

A

Answer: Muscle tremors and seizures.

Explanation
Early clinical signs associated with lidocaine toxicity include neurological symptoms such as seizures and tremors. Nausea and vomiting may occur, but is usually transient. Cardiovascular and respiratory depression can also occur, but usually later on in the course of clinical signs.

Lidocaine toxicity in an 8-year-old mixed breed terrier initially presents with neurological symptoms like muscle tremors and seizures. While nausea and vomiting may occur, these are usually transient. Cardiovascular and respiratory depression are possible but typically occur later in the clinical course. Early recognition and intervention are critical to manage toxicity effectively.

116
Q

A recently adopted 5-month-old female spayed mixed breed dog presents for vaccines, and you auscult a V/VI murmur at the left heart base. It is a very loud continuous murmur, which sounds like a washing machine. The owner reports normal activity, energy, and appetite. Her gums are pink with good refill. She has strong synchronous pulses. Based on your assessment, you most suspect which of the following?

Patent ductus arteriosus
Heartworm infection
Physiologic murmur
Atrial septal defect

A

Answer: Patent ductus arteriosus

Explanation
A patent ductus arteriosus typically is associated with a loud, continuous murmur that may be ausculted at the left heart base/axillary region. The sound is sometimes described as sounding like a “washing machine.” Subaortic and pulmonic stenosis also typically have loud systolic murmurs heard best at the left heart base.

An atrial septal defect is not typically associated with an intense murmur. More commonly, the murmur of ASD is Il-III/VI and is located at the left heart base due to increased flow across the pulmonary valve because of left-to-right shunting. (Note, you do not hear the flow across the atrial septum because it is of very low velocity).

The life cycle of the heartworm is approximately 6 months (this patient is only 5 months old), and heartworm does not typically cause this type of murmur. Physiologic murmurs are typically a grade I-II/VI, and are low intensity.

A patent ductus arteriosus (PDA) is likely in this 5-month-old dog, given the loud, continuous “washing machine” murmur at the left heart base. PDA is associated with such murmurs, unlike atrial septal defects, which usually have softer murmurs. Heartworm infection is improbable due to the dog’s age, and physiologic murmurs are typically low-intensity.

Patent Ductus Arteriosus (PDA) in Animals
Concepts
Patent ductus arteriosus (PDA) is a congenital heart defect where the ductus arteriosus fails to close after birth, leading to abnormal blood flow between the aorta and pulmonary artery.

Etiology
• Non-Infectious: Congenital defect present at birth.
• Genetic Predisposition: More common in certain dog breeds (e.g., Poodles, German Shepherds).

Pathogenesis
• Normal Fetal Circulation: Blood bypasses the lungs via the ductus arteriosus.
• Postnatal Changes: Ductus arteriosus should close to direct blood through the lungs. If it remains open (patent), it causes left-to-right shunting of blood, leading to volume overload of the left heart and pulmonary circulation.

Symptoms
• Left-to-Right Shunt: Continuous murmur (machinery murmur) at the left base, bounding pulses, and signs of left heart failure (cough, exercise intolerance).
• Right-to-Left Shunt (Reverse PDA): Cyanosis, especially caudal body parts, lethargy, and exercise intolerance.

Diagnostic Methods
• Clinical Signs: Auscultation revealing continuous murmur, bounding femoral pulses.
• Imaging: Radiographs show left atrial and ventricular enlargement; echocardiography confirms diagnosis.
• Electrocardiography (ECG): Tall R waves indicating left ventricular enlargement, arrhythmias.
Treatment
• Left-to-Right Shunt:
• Interventional: Transvenous occlusion using devices like the Amplatz Canine Ductal Occluder.
• Surgical: Ligation of the ductus arteriosus.
• Medical Management: CHF treatment if present.
• Right-to-Left Shunt (Reverse PDA):
• Medical Management: Focuses on controlling pulmonary hypertension and managing polycythemia.
• Interventional/Surgical: Contraindicated unless pulmonary hypertension is managed.
Prognosis
• Left-to-Right Shunt: Excellent if treated early before significant clinical signs develop.
• Right-to-Left Shunt: Guarded to poor due to complications from pulmonary hypertension.

117
Q

All of the following drugs are appropriate in the management of tachycardia associated with this arrhythmia in small animal patients except:

Isoproterenol
Atenolol
Digoxin
Procainamide
Diltiazem

A

Answer: Isoproterenol

This ECG shows atrial fibrillation. Atenolol is a beta-blocker and will slow AV nodal conduction to decrease the ventricular response rate to atrial fibrillation. Procainamide is a class 1A anti-arrhythmic and can be used in attempts to convert atrial fibrillation to a normal sinus rhythm, though it is rarely effective. Diltiazem is a calcium channel blocker and can be used to slow AV nodal conduction and ventricular response rate. Digoxin will increase vagal tone to the AV node to slow conduction and decrease the heart rate. Isoproterenol is a nonspecific beta receptor agonist, and will increase the ventricular rate in response to atrial fibrillation, thus is inappropriate in the management of this dysrhythmia.

Isoproterenol is inappropriate for managing atrial fibrillation in small animals as it increases ventricular rate. Atenolol (beta-blocker), Digoxin (increases vagal tone), and Diltiazem (calcium channel blocker) slow AV nodal conduction, decreasing the ventricular response rate. Procainamide (class 1A anti-arrhythmic) attempts to convert atrial fibrillation to sinus rhythm, though rarely effective.

118
Q

A 10-year old male castrated Collie presents for what the owner calls heart failure. Which of the following is not a typical sign of right heart failure?

Exercise intolerance
Muscle wasting
Hepatomegaly
Tachypnea
Cough

A

Answer: Cough

Explanation
The correct answer is cough. Coughing is typically a sign of left heart failure that occurs with the development of pulmonary edema and/or bronchial compression from an enlarged left atrium. In right heart failure, tachypnea and dyspnea can develop due to pleural effusion. Hepatomegaly occurs due to venous congestion from the damming of blood into the liver. Exercise intolerance develops because a decrease in cardiac output causes a decrease in oxygenation of tissues. Muscle wasting occurs from loss of protein into effusions and from hepatic and Gl malfunction. Other signs of right heart failure include lethargy, weakness, venous distention, ascites, and peripheral edema.

Cough is not typical of right heart failure; it’s usually associated with left heart failure due to pulmonary edema or bronchial compression. Right heart failure signs include tachypnea, hepatomegaly, exercise intolerance, muscle wasting, lethargy, weakness, venous distention, ascites, and peripheral edema. These result from pleural effusion, venous congestion, and decreased cardiac output.

119
Q

Which of the following is a non-adrenergic vasoconstrictor that can be used in dogs?

Dobutamine
Isoproterenol
Vasopressin
Epinephrine
Phenylephrine

A

Answer: Vasopressin

Explanation
The correct answer is vasopressin. Epinephrine and phenylephrine are adrenergic vasoconstrictors.

Isoproterenol and dobutamine cause vasodilation. Vasopressin (also known as anti-diuretic hormone) is a V-1 receptor, non-adrenergic, vasoconstrictor.

Vasopressin is a non-adrenergic vasoconstrictor used in dogs. Unlike epinephrine and phenylephrine, which are adrenergic vasoconstrictors, and isoproterenol and dobutamine, which cause vasodilation, vasopressin acts on V-1 receptors to induce vasoconstriction.

120
Q

Dilated cardiomyopathy is a common disease seen in dogs. Which one of the following medications is indicated for the treatment of DCM?

Atropine
Prednisone
Enalapril
Adriamycin

A

Answer: Enalapril

Explanation
The correct answer is enalapril. All others would be contraindicated. Enalapril is an ACE-inhibitor, which blunts the adverse effects of the renin-agiotensin-aldosterone system thereby reducing sodium and fluid retention.

Additionally, ACE-inhibitors cause mild vasodilation by preventing the production of angiotensin-ll, which reduces ventricular afterload. Lastly, enalapril and other ACE-inhibitors are protective to cardiac muscle by blunting aldosterone and other hormones which induce cardiac remodeling.

Enalapril is indicated for the treatment of dilated cardiomyopathy (DCM) in dogs. As an ACE inhibitor, it reduces the adverse effects of the renin-angiotensin-aldosterone system, decreasing sodium and fluid retention, causing mild vasodilation, and reducing ventricular afterload. Additionally, enalapril helps protect cardiac muscle by preventing harmful cardiac remodeling. Other medications like atropine, prednisone, and adriamycin are contraindicated in this condition.

Dilated Cardiomyopathy (DCM) in Dogs and Cats

Definitions and Causative Agents:
• DCM: A disease characterized by the progressive loss of myocyte number and/or function, leading to decreased cardiac contractility.
• Causative Agents: Genetic predispositions, nutritional deficiencies (e.g., taurine in cats), chemical (e.g., doxorubicin), viral (e.g., parvovirus), and idiopathic causes.

Clinical Findings:
• Symptoms: Often subclinical in early stages; advanced cases exhibit signs of heart failure including cough, dyspnea, exercise intolerance, syncope, and weight loss.
• Lesions: Enlarged heart with thin walls, particularly affecting the left ventricle and atrium.
• Physical Exam: Soft systolic heart murmur, gallop rhythm, weak femoral pulses, arrhythmias (ventricular ectopy in Doberman Pinschers and Boxers, atrial fibrillation in giant breeds).

Diagnosis:
• Echocardiography: Best method to confirm diagnosis, showing left ventricular and atrial enlargement and reduced contractility.
• ECG: Detects ventricular premature complexes, ventricular tachycardia, and atrial fibrillation.
• Radiographs: Reveal cardiomegaly and signs of congestive heart failure (CHF).
Treatment:
• Medications:
• Pimobendan: Increases contractility and reduces preload/afterload.
• ACE Inhibitors (e.g., enalapril, benazepril): Mitigate the effects of the renin-angiotensin-aldosterone system (RAAS).
• Furosemide: Diuretic for managing CHF.
• Antiarrhythmics: Used particularly in breeds prone to ventricular ectopy.
• Taurine Supplementation: Effective in taurine-responsive DCM cases.
• Supportive Care: Oxygen therapy, dietary management.

Prognosis:
• Dogs: Poor long-term prognosis, with most dying within 6 months of diagnosis; better outcomes in taurine-responsive cases.
• Cats: Severe respiratory signs, poor prognosis in non-taurine-responsive cases, better prognosis if taurine-responsive and treated early.

121
Q

An 8-year old male castrated Labrador Retriever presents for collapsing episodes. You record an ECG, which is shown here. What is your diagnosis?

Complete heart block
Sinus arrest
Normal sinus rhythm
Atrial premature complexes
Ventricular premature complexes

A

Answer: Complete heart block.

The correct answer is complete heart block. The ECG shows frequent P waves and two ventricular escape beats from different foci. Complete heart block typically results from degeneration and fibrosis of the cardiac conduction system. The animal then relies on a slow escape rate, which often results in clinical symptoms of weakness and collapse. Sinus arrest would be seen as periods of flat line on the ECG with no atrial activity; normal sinus rhythm would show conduction of the P waves with QRS complexes following. Ventricular premature complexes is incorrect because the above 2 ventricular beats are escapes, they are not premature.

Atrial premature complexes is wrong as the atrial rhythm is normal.

The diagnosis is complete heart block. The ECG shows frequent P waves with two ventricular escape beats from different foci, indicating a complete dissociation between atrial and ventricular activity. Complete heart block is often due to degeneration and fibrosis of the cardiac conduction system, leading to a slow escape rate, which can cause clinical symptoms like weakness and collapse. Sinus arrest, normal sinus rhythm, ventricular premature complexes, and atrial premature complexes do not match this ECG pattern.

122
Q

Which of the following is the treatment of choice for pulmonic stenosis in the dog?

Valvulotomy by catheterization device
Balloon valvuloplasty
Surgical valvulotomy
Patch grafting of the outflow tract

A

Answer: Balloon valvuloplasty

Explanation
The correct answer is balloon valvuloplasty. This procedure is performed by inserting a catheter into either a jugular vein or a femoral vein and advancing it into the region of stenosis at the pulmonic outflow tract. Once in place, a balloon surrounding the catheter is inflated to stretch the stenotic region open. Surgical valvulotomy or patch grafting is the treatment used for severe cases that cannot be balloon-dilated.

Balloon valvuloplasty is the treatment of choice for pulmonic stenosis in dogs. It involves inserting a catheter through the jugular or femoral vein to the pulmonic outflow tract and inflating a balloon to stretch the stenotic region. Surgical valvulotomy or patch grafting is reserved for severe cases unresponsive to balloon dilation.

Condition Overview: Pulmonic Stenosis (PS) in Dogs
• Definition: Pulmonic Stenosis (PS) is a congenital heart defect characterized by the obstruction of blood flow from the right ventricle to the pulmonary artery due to narrowing at, below, or above the pulmonary valve.
• Classification:
• Subvalvular: Below the valve.
• Valvular: At the valve (most common).
• Supravalvular: Above the valve.
• Valvular Subtypes:
• Type A: Leaflet fusion with systolic doming.
• Type B: Thickened, hypomobile leaflets with annulus hypoplasia (associated with poorer prognosis).

Pathophysiology:
• Hemodynamic Impact: PS increases resistance to blood flow from the right ventricle, leading to right ventricular hypertrophy and, if severe, right-sided congestive heart failure (RCHF).
• Doppler Gradient (DG):
• Mild PS: DG < 50 mmHg.
• Moderate PS: DG between 50 and 80 mmHg.
• Severe PS: DG > 80 mmHg.
• Consequences of Severe PS:
• Increased right ventricular pressure.
• Decreased cardiac output.
• Potential for arrhythmias, syncope, RCHF, and sudden death.

Clinical Signs:
• Symptoms:
• Exercise intolerance.
• Syncope (fainting).
• Signs of RCHF: Ascites, pleural effusion, hepatomegaly.
• Prognostic Factors:
• Age at Diagnosis: Younger age at diagnosis is associated with worse outcomes.
• Valve Morphology: Type B morphology correlates with a poorer prognosis due to more severe malformations.
• Presence of Clinical Signs: Clinical signs at diagnosis are a strong predictor of a poorer prognosis.

Diagnostic Evaluation:
• Echocardiography:
• Assess severity of PS, right ventricular hypertrophy, and tricuspid regurgitation.
• Valvular Morphology: Important to differentiate between Type A and Type B for prognosis.
• Doppler Echocardiography: Measures the peak transvalvular pulmonary gradient to assess the severity of stenosis.

Treatment Options:
• Pulmonary Balloon Valvuloplasty (PBV):
• Indications:
• Severe PS (DG > 80 mmHg).
• Moderate PS with clinical signs or rapidly worsening condition.
• Outcomes:
• PBV significantly improves survival in dogs with severe PS.
• Dogs with moderate PS may benefit, particularly if symptomatic.
• Medical Management:
• Beta-blockers (Atenolol):
• Dose: 0.5 mg/kg orally every 12-24 hours.
• Indications: To manage symptoms in both PBV and non-PBV patients, particularly in moderate to severe PS.
• Atenolol may be discontinued post-PBV unless there is a poor reduction in DG or significant right ventricular hypertrophy persists.

Prognosis:
• Survival Rates:
• Severe PS without PBV: High mortality, particularly in the first year (53%).
• Severe PS with PBV: Significantly lower mortality (4-6% in the first year).
• Moderate PS: Generally favorable prognosis, though clinical signs indicate a higher risk of cardiac-related death.
• Long-term Considerations:
• Right Ventricular Function: Critical in determining the prognosis post-PBV. Poor right ventricular function often leads to poor outcomes even after intervention.

Risk Factors:
• Negative Predictors of Survival:
• Younger Age at Diagnosis: High hazard ratio (HR 12.82, P<0.01) indicating increased risk of cardiac-related death.
• Type B Valve Morphology: Associated with a higher risk of restenosis and poorer outcomes (HR 10.20, P<0.01).
• Presence of Clinical Signs: Strongly correlated with cardiac death, particularly in severe PS (HR 5.41, P<0.01).

Considerations for Specific Cases:
• Right-Sided Congestive Heart Failure (RCHF):
• Prognosis: Poor, with a median survival time of 12 months.
• Treatment Decisions: PBV may be beneficial, but careful evaluation of right ventricular function is necessary before proceeding.
• Syndromic Associations:
• Type B PS in brachycephalic breeds (e.g., Bulldogs, Boxers) may resemble genetic syndromes like Noonan syndrome in humans, characterized by congenital cardiac malformations.

Clinical Guidelines:
• Intervention Timing: Early intervention, particularly before the onset of clinical signs, may improve outcomes.
• Post-Treatment Monitoring: Regular echocardiographic evaluations post-PBV are crucial, particularly for monitoring DG and right ventricular function.

123
Q

A 4-year old male Australian Shepherd is presented to your clinic for treatment of heartworm disease. Which of the following statements about heartworm disease is true?

strict activity restriction of the dog is very important once medical treatment is initiated
Doxycycline can be used to treat heartworms
Ivermectin is the treatment of choice for adult worms
The owners should be informed that dogs often can develop right heart failure post treatment even if the dog was clinically normal pre treatment

A

Answer: strict activity restriction of the dog is very important once medical treatment is initiated

Explanation
The correct answer is strict activity restriction of the dog is very important once treatment is initiated. Dogs should be restricted in activity at least 4-6 weeks post treatment to decrease the likelihood of thromboembolic events and to allow the lungs to recover from injury due to worm death. Dogs are not as likely to develop right heart failure after treatment of heartworm disease if they were previously clinically normal. Ivermectin is often used to prevent heartworm disease and for treating microfilariae, but the treatment of choice for adult worms is melarsomine IM. Doxycycline is often used to treat the Wolbachia bacteria that are often found with heartworms, but is ineffective at treating the worms themselves.

Strict activity restriction is crucial once heartworm treatment starts, to reduce thromboembolic events and allow lung recovery post worm death. Dogs clinically normal pre-treatment rarely develop right heart failure afterward. Ivermectin prevents heartworm and treats microfilariae, but melarsomine IM treats adult worms. Doxycycline targets Wolbachia bacteria associated with heartworms, not the worms themselves.

General Information Heartworm disease in animals:
• Causative Agent: Dirofilaria immitis (filarial nematode).
• Vector: Mosquitoes (e.g., Aedes, Anopheles, Culex).
• Species Affected: Primarily dogs, but also cats, ferrets, and occasionally other species like coyotes and foxes.

Pathogenesis:
• Life Cycle: Mosquitoes transmit infective larvae (L3), which migrate into the host, molt, and mature into adults in the pulmonary arteries.
• Pulmonary Disease: The disease affects the pulmonary arteries, causing inflammation, endarteritis, and pulmonary hypertension. Long-term infections can lead to chronic lesions, scarring, and heart failure.
• Immunological Response: Wolbachia bacteria within the heartworms contribute to the inflammatory response and disease severity. Antigenic stimulation leads to immune responses, causing further damage.

Clinical Findings:
• Dogs: Most common signs include cough, exercise intolerance, and signs of right-sided heart failure (e.g., ascites). Severe cases can lead to caval syndrome with signs like hemolysis, anemia, and hepatorenal dysfunction.
• Cats and Ferrets: Often subclinical or mild, but severe cases can present with respiratory distress, weight loss, lethargy, vomiting, and sudden death. Cats can suffer from HARD (heartworm-associated respiratory disease).

Diagnosis:
• Serology: Antigen testing is the primary method in dogs, while antibody testing is more useful in cats due to the lower sensitivity of antigen tests in cats.
• Microfilaria Detection: Direct smear or modified Knott test; useful as an adjunct in dogs.
• Imaging: Thoracic radiography, echocardiography (particularly important in cats and ferrets).

Treatment:
• Dogs: Adulticide therapy with melarsomine (an organic arsenic compound), typically in a three-dose protocol. Adjunct therapy with doxycycline to target Wolbachia. Supportive care and exercise restriction are crucial.
• Cats and Ferrets: No safe adulticide therapy; treatment focuses on managing symptoms with corticosteroids, oxygen therapy, and occasionally surgical removal of heartworms.

Prevention:
• Macrocyclic Lactones: Regular administration of ivermectin, milbemycin, selamectin, or moxidectin for prevention. For dogs, these drugs also serve as microfilaricides.

Stages of Disease:
1. Stage I: Subclinical or minimal signs.
2. Stage II: Mild to moderate signs, typically including cough.
3. Stage III: Severe signs, such as weight loss, respiratory distress, and right heart failure.
4. Stage IV: Caval syndrome with severe cardiopulmonary compromise.

https://www.merckvetmanual.com/circulatory-system/heartworm-disease/heartworm-disease-in-dogs-cats-and-ferrets

124
Q

Which of the following is a non-adrenergic vasoconstrictor that can be used in dogs?

Epinephrine
Phenylephrine
Vasopressin
Dobutamine
Isoproterenol

A

Answer: Vasopressin

Explanation
The correct answer is vasopressin. Epinephrine and phenylephrine are adrenergic vasoconstrictors.

Isoproterenol and dobutamine cause vasodilation. Vasopressin (also known as anti-diuretic hormone) is a V-1 receptor, non-adrenergic, vasoconstrictor.

125
Q

Which of the following is the most common treatment for aortic stenosis in a dog?

Surgical repair
Medical management with beta blockers
Medical management with furosemide
Balloon dilation of the stenotic region by catheterization

A

Answer: Medical management with Beta Blockers

Explanation
The correct answer is medical management with beta blockers. Beta blockers such as atenolol are often used to reduce myocardial oxygen demand, thus reducing the frequency of ventricular arrhythmias. The owners should also be warned that the dog will need prophylactic antibiotics for any situation in which the dog may develop a bacteremia, since it is at high risk of developing infective endocarditis. Balloon dilation by catheterization can be attempted in some patients, but the results are usually no better than medical therapy with beta blockers.

Surgical repair is difficult, expensive, and does not give very good results either. There is no indication for furosemide unless the dog is in heart failure.

Medical management with beta blockers, such as atenolol, is the most common treatment for aortic stenosis in dogs. Beta blockers reduce myocardial oxygen demand and ventricular arrhythmias. Prophylactic antibiotics are needed to prevent infective endocarditis. Balloon dilation offers no better results than beta blockers, and surgical repair is challenging and less effective. Furosemide is only indicated for heart failure.

126
Q

A 1.5-year old male castrated Boxer presents for exercise intolerance and syncopal episodes. Cardiac auscultation reveals a grade 4/6 left systolic murmur heard best at the base of the heart. Thoracic radiographs show a mildly enlarged left ventricle and left atrium, and segmental enlargement of the ascending aorta. What test will best help you to confirm your tentative diagnosis?

Heartworm antigen test
CT scan
24 hour Holter ECG monitor
Coombs test
Echocardiogram

A

Answer: Echocardiogram

The breed, clinical history of syncope, radiographic findings, and auscultation findings are most consistent with aortic stenosis. Echocardiography would be the best method for diagnosing the disease in which you would find left ventricular concentric hypertrophy, stenosis of the aortic outflow tract, and post-stenotic dilatation of the ascending aorta.

A Holter monitor is the test of choice for arrhythmogenic right ventricular cardiomyopathy (ARVC) which is also seen in Boxers and causes syncope but does not fit with the physical exam or radiographic findings. Also called Boxer cardiomyopathy, ARVC is a disease associated with fibrofatty infiltrate into the right ventricle resulting in electrical conduction disturbances and resultant VPCs.

A CT scan would not add more information, and cardiac motion typically causes resolution problems on thoracic CT scans. The signs are not consistent with heartworm, which is primarily a right heart problem, or with hemolytic anemia, which a Coombs’ test might be useful for.

Echocardiogram is best for confirming aortic stenosis in this 1.5-year-old Boxer, given the breed, syncope, and findings of a grade 4/6 systolic murmur, left ventricular and atrial enlargement, and ascending aorta dilation. It reveals left ventricular concentric hypertrophy, aortic outflow tract stenosis, and post-stenotic dilation. Holter ECG monitors are for arrhythmogenic right ventricular cardiomyopathy, which doesn’t match this case’s findings.

127
Q

A 2 year old male castrated Boxer presents for exercise intolerance and syncopal episodes. Cardiac auscultation reveals a 4/6 left systolic murmur heard best at the base of the heart. Thoracic radiographs show a mildly enlarged left ventricle and left atrium, and segmental enlargement of the ascending aorta. What is the most likely diagnosis?

Pulmonic stenosis
Aortic stenosis
Myxomatous degeneration of the mitral valve
Boxer cardiomyopathy

A

Answer: Aortic stenosis

Explanation
The correct answer is aortic stenosis. The breed, clinical history of syncope, radiographic findings, and auscultation findings are most consistent with aortic stenosis. Echocardiography would be the best method for diagnosing the disease in which you would find left ventricular hypertrophy, stenosis of the aortic outflow tract, and post-stenotic dilatation of the aorta.

Pulmonic stenosis would result in changes in the right heart and pulmonary outflow tract. Myxomatous degeneration of the mitral valve typically occurs in older animals, results in a murmur heard best at the left apex, and would not cause a dilation of the ascending aorta.

Boxer cardiomyopathy (or arrhythmogenic right ventricular cardiomyopathy) is principally an electrical disease causing frequent VPCs to occur. Dilation of the heart may occur in 10-15% of cases. The best way to diagnose Boxer cardiomyopathy is with a 24 hour cardiac Holter monitor.

The most likely diagnosis is aortic stenosis. The breed, syncope, radiographic findings (mildly enlarged left ventricle and atrium, segmental enlargement of the ascending aorta), and auscultation (4/6 left systolic murmur at heart base) support this. Echocardiography reveals left ventricular hypertrophy, aortic outflow tract stenosis, and post-stenotic aorta dilation. Pulmonic stenosis affects the right heart, myxomatous mitral valve degeneration affects older dogs with a murmur at the left apex, and Boxer cardiomyopathy involves frequent VPCs and is diagnosed with a Holter monitor.

128
Q

A 9 month old shepherd cross presents for lethargy and exercise intolerance. Your physical exam reveals dark red mucous membranes and a heart murmur. A CBC reveals a hematocrit of 70% (35-57%). Which of the following could NOT explain the given findings?

Atrial septal defect with tricuspid stenosis
PDA with pulmonary hypertension
Atrial septal defect with systemic hypertension
Tetralogy of Fallot

A

Answer: Atrial septal defect with systemic hypertension

Explanation
The correct answer is atrial septal defect with systemic hypertension. This dog’s polycythemia developed due to shunting of venous blood into the arterial circulation (aka right-to-left shunting) leading to hypoxemia and an increased drive for erythropoiesis and secondary polycythemia. This can be caused by any of the other 3 answer choices. An atrial septal defect with systemic hypertension results in oxygenated blood shunting into the venous circulation or left-to-right shunting, which usually does not cause hypoxemia.

Atrial septal defect with systemic hypertension cannot explain the findings. This condition causes left-to-right shunting, leading to oxygenated blood entering the venous circulation, not hypoxemia. The dog’s polycythemia (hematocrit 70%) suggests right-to-left shunting, leading to hypoxemia and increased erythropoiesis. This can be caused by PDA with pulmonary hypertension, atrial septal defect with tricuspid stenosis, or Tetralogy of Fallot.

129
Q

A 9 month old shepherd cross presents for lethargy and exercise intolerance. Your physical exam reveals dark red mucous membranes and a heart murmur. A CBC reveals a hematocrit of 70% (35-57%). Which of the following could NOT explain the given findings?

Atrial septal defect with tricuspid stenosis
PDA with pulmonary hypertension
Atrial septal defect with systemic hypertension
Tetralogy of Fallot

A

Answer: Atrial septal defect with systemic hypertension

Explanation
The correct answer is atrial septal defect with systemic hypertension. This dog’s polycythemia developed due to shunting of venous blood into the arterial circulation (aka right-to-left shunting) leading to hypoxemia and an increased drive for erythropoiesis and secondary polycythemia. This can be caused by any of the other 3 answer choices. An atrial septal defect with systemic hypertension results in oxygenated blood shunting into the venous circulation or left-to-right shunting, which usually does not cause hypoxemia.

Atrial septal defect with systemic hypertension cannot explain the findings. This condition causes left-to-right shunting, leading to oxygenated blood entering the venous circulation, not hypoxemia. The dog’s polycythemia (hematocrit 70%) suggests right-to-left shunting, leading to hypoxemia and increased erythropoiesis. This can be caused by PDA with pulmonary hypertension, atrial septal defect with tricuspid stenosis, or Tetralogy of Fallot.

Key Information on Patent Ductus Arteriosus (PDA) in Animals
Definitions
• Patent Ductus Arteriosus (PDA): A congenital heart defect where the ductus arteriosus fails to close after birth, causing abnormal blood flow between the aorta and pulmonary artery.

Pathophysiology
• Fetal Circulation: Blood bypasses the lungs via the ductus arteriosus.
• Postnatal Changes: Normally closes to become the ligamentum arteriosum.
• PDA Pathology: Persistent patency causes left-to-right shunting, leading to volume overload and left heart failure.

Clinical Findings
• Symptoms: Continuous murmur, bounding femoral pulses, left heart failure (pulmonary edema).
• Reverse PDA: Right-to-left shunting, differential cyanosis, polycythemia.
• Diagnostic Tests: Radiography (left atrial/ventricular enlargement), echocardiography (turbulent flow in pulmonary artery).

Diagnosis
• Electrocardiography: Tall R waves in lead II.
• Radiography: Aortic ductus diverticulum, left heart enlargement.
• Echocardiography: Continuous turbulence in pulmonary artery, left heart enlargement.

Treatment
• Early Closure: Recommended to prevent heart failure.
• Methods: Interventional transvenous occlusion (ACDO), surgical ligation.
• Reverse PDA Management: Medical management of pulmonary hypertension and polycythemia, not closure.

https://www.merckvetmanual.com/circulatory-system/congenital-and-inherited-anomalies-of-the-cardiovascular-system/patent-ductus-arteriosus-in-animals?redirectid=27882

Tetralogy of Fallot in Animals
Definition:
• Congenital heart defect with four components: pulmonic stenosis, ventricular septal defect, right ventricular hypertrophy, and overriding aorta.

Affected Species:
• Dogs (e.g., Keeshonds, English Bulldogs), cats.

Clinical Signs:
• Cyanosis, exercise intolerance, stunted growth, collapse, seizures.
• Loud ejection heart murmur, precordial thrill.

Diagnosis:
• Echocardiography: Confirms structural defects.
• Radiographs: Right heart enlargement.
• ECG: Right ventricular enlargement.

Treatment:
• Beta-adrenergic blockers to manage outflow obstruction.
• Periodic phlebotomy for polycythemia.
• Surgical options: Palliative techniques, valvuloplasty.

Control:
• Manage based on severity, supportive care, and specific surgical interventions.

https://www.merckvetmanual.com/circulatory-system/congenital-and-inherited-anomalies-of-the-cardiovascular-system/tetralogy-of-fallot-in-animals?query=tetralogy%20of%20fallot

130
Q

A 1-year old German Shepherd Dog presents for its first physical exam. Cardiac auscultation reveals a continuous murmur. Thoracic radiographs and echocardiography lead you to the diagnosis of a very large, cylinder-shaped patent ductus arteriosus with left-to-right shunting of blood. Which of the following statements is most true?

Placement of constrictor ring is the most appropriate form of treatment for a cylinder-shaped PDA
Coil embolization is the most appropriate form of treatment for a cylinder-shaped PDA
Furosemide and enalapril are the most appropriate forms of treatment for a cylinder-shaped PDA
Surgical ligation is the most appropriate form of treatment for a cylinder-shaped PDA

A

Answer: Surgical ligation is the most appropriate form of treatment for a cylinder-shaped PDA

Explanation
The correct answer is surgical ligation of the PDA is the most appropriate form of treatment. A large cylindrical PDA is unlikely amenable to closure via traditional coil embolization. Most canine PDAs are funnel-shaped with the smaller end of the funnel attached to the pulmonary artery, and the wider end attached to the aorta. The funnel shape is what allows the coils to lodge in place and cause closure of the PDA. Coils would not stay in place in a large, cylindrical PDA, so surgical ligation is the treatment of choice for this patient. There are newer nickel-titanium devices (Amplatz Canine Ductal Occluder) that show promise for minimally-invasive closure of PDAs which do not taper, but this answer was not provided. Ameroid constrictor rings are typically used for ligating hepatic shunts, and not for PDAs. Medical management with diuretics and heart drugs are only indicated prior to correction of the PDA if the dog is in heart failure.

Surgical ligation is the most appropriate treatment for a large, cylinder-shaped PDA. Traditional coil embolization is ineffective for cylindrical PDAs, as coils need a funnel shape to lodge properly. The newer Amplatz Canine Ductal Occluder might be effective but wasn’t an option. Ameroid constrictor rings are for hepatic shunts, and medical management with furosemide and enalapril is only for pre-correction heart failure.

Heartworm Disease in Small Animals.
Causative Agent:
• Dirofilaria immitis: A parasitic nematode transmitted by mosquitoes, causing heartworm disease in dogs, cats, and ferrets.

Life Cycle:
• Mosquitoes acquire microfilariae from infected hosts, developing into L3 larvae. These larvae migrate to the heart and pulmonary arteries, maturing into adults in 6-7 months.

Pathophysiology:
• Adult heartworms cause pulmonary artery inflammation, vascular damage, and pulmonary hypertension. Severe infections lead to right-sided heart failure and caval syndrome. In cats, heartworm-associated respiratory disease (HARD) is due to the immune response to dying larvae.

Clinical Signs:
• Dogs: Cough, exercise intolerance, weight loss, dyspnea, hemoptysis, syncope, ascites.
• Cats: Coughing, dyspnea, vomiting, weight loss, sudden death.
• Ferrets: Similar to dogs but with more pronounced signs due to their small size.

Diagnosis:
• Serology: Antigen tests (dogs, cats, ferrets), antibody tests (cats).
• Microfilaria Detection: Direct smear, modified Knott test.
• Imaging: Thoracic radiography, echocardiography (especially important in cats).

Treatment:
• Dogs: Melarsomine (3 injections), doxycycline (to eliminate Wolbachia), supportive care. Preventive medications like ivermectin, milbemycin, moxidectin, or selamectin.
• Cats/Ferrets: Primarily supportive care. Doxycycline may help, but adulticide therapy is generally avoided.

Prevention:
• Macrocyclic lactones: Regular prophylaxis is crucial (e.g., ivermectin, milbemycin).

Special Considerations:
• Wolbachia: Symbiotic bacteria essential for heartworm survival, targeted with doxycycline to enhance treatment efficacy.

131
Q

Which of the following drugs and treatment intervals are appropriate for preventing heartworm disease in dogs?

Milbemycin twice daily
Ivermectin once a month
Milbemycin oxime once daily
Ivermectin once daily
Diethylcarbamazine once monthly

A

Answer: Ivermectin once a month

Explanation
The correct answer is ivermectin once monthly. Diethylcarbamazine (DEC) can be used to prevent heartworm, but is administered once daily. Milbemycin oxime is another preventative used once monthly.

Ivermectin once a month is the correct heartworm prevention treatment. Diethylcarbamazine (DEC) is used daily, and Milbemycin oxime is another monthly preventative. Other options, like Milbemycin twice daily or Ivermectin daily, are incorrect.

132
Q

Which of the following has the least effect on heart rate?

Morphine
Ketamine
Propofol
Thiopental

A

Answer: Propofol

Explanation
The correct answer is propofol. Ketamine causes an increase in heart rate and blood pressure. It is considered seizureogenic and has poor muscle relaxation effects, however it provides good analgesia. Thiopental is an ultra fast acting thiobarbiturate that causes an increased heart rate, increased blood pressure, and decreased cardiac contractility. It is also an arrhythmogenic agent. Morphine is an opioid that causes bradycardia. Propofol does not result in any significant cardiac changes. It is ultra-short in duration, can cause apnea and myoclonus, and is considered anticonvulsive. It may cause a mild bradycardia but you mainly see that when used in combination with other opiates. More recently propofol has been associated with myocardial depression and hypotension in critically ill dogs and may not be the induction agent of choice as a result. Despite these findings, propofol still has the least effect on heart rate.

Propofol has the least effect on heart rate. Ketamine increases heart rate and blood pressure, Thiopental increases heart rate and blood pressure but decreases cardiac contractility, and Morphine causes bradycardia. Propofol primarily causes mild bradycardia when combined with other opiates and has minimal cardiac impact, making it the best option regarding heart rate stability.

133
Q

Which of the following is true about diltiazem?

Diltiazem slows the heart by antagonizing beta adrenergic receptors.
Diltiazem speeds up the heart because it is a beta adrenergic agonist.
Diltiazem slows the heart by inhibiting the influx of calcium into the myocardial cells.
Diltiazem speeds up the heart rate by increasing the influx of calcium into the myocardial cells.

A

Answer: Diltiazem slows the heart by inhibiting the influx of calcium into the myocardial cells.

Explanation
The correct answer is diltiazem slows the heart by inhibiting the influx of calcium into the myocardial cells.
Diltiazem is a calcium channel blocker used to slow the heart to treat supraventricular tachycardias, hypertrophic cardiomyopathy, and hypertension.

Diltiazem slows the heart by inhibiting the influx of calcium into myocardial cells. It is a calcium channel blocker used to manage supraventricular tachycardias, hypertrophic cardiomyopathy, and hypertension. It does not antagonize beta adrenergic receptors or act as a beta adrenergic agonist.

134
Q

A 2-year old male castrated Beagle presents for vomiting and diarrhea. The blood smear for a CBC shows microfilariae in the peripheral blood. Which of the following parasites must be ruled out?

Acanthocheilonema reconditum
Filaroides osleri
Strongyloides stercoralis
Uncinaria stenocephala

A

Answer: Acanthocheilonema reconditum

Explanation
The correct answer is Acanthocheilonema reconditum (previously Dipetalonema reconditum).

Acanthocheilonema reconditum is a blood parasite that looks similar to the microfilariae of Dirofilaria immitis, the agent of Heartworm disease. The two parasites must be differentiated because Acanthocheilonema reconditum is not pathogenic and is therefore not treated. Uncinaria stenocephala, Strongyloides stercoralis, and Filaroides osleri would not be found in the blood.

The parasite that must be ruled out in a dog with microfilariae in the peripheral blood is Acanthocheilonema reconditum. This blood parasite resembles Dirofilaria immitis (heartworm), but unlike heartworm, A. reconditum is not pathogenic and does not require treatment.

135
Q

A 2-year old male castrated Beagle presents for vomiting and diarrhea. The blood smear for a CBC shows microfilariae in the peripheral blood. Which of the following parasites must be ruled out?

Acanthocheilonema reconditum
Filaroides osleri
Strongyloides stercoralis
Uncinaria stenocephala

A

Answer: Acanthocheilonema reconditum

Explanation
The correct answer is Acanthocheilonema reconditum (previously Dipetalonema reconditum).

Acanthocheilonema reconditum is a blood parasite that looks similar to the microfilariae of Dirofilaria immitis, the agent of Heartworm disease. The two parasites must be differentiated because Acanthocheilonema reconditum is not pathogenic and is therefore not treated. Uncinaria stenocephala, Strongyloides stercoralis, and Filaroides osleri would not be found in the blood.

The parasite that must be ruled out in a dog with microfilariae in the peripheral blood is Acanthocheilonema reconditum. This blood parasite resembles Dirofilaria immitis (heartworm), but unlike heartworm, A. reconditum is not pathogenic and does not require treatment.

136
Q

Which of the following statements is true about central venous pressure?

It is a direct measure of preload
It is an estimation of afterload
it is a direct measure of blood pressure in the cranial vena cava
The measurement is independent of cardiac output
The measurement is independent of venous vessel tone

A

Answer: it is a direct measure of blood pressure in the cranial vena cava

Explanation
The correct answer is it is a direct measure of blood pressure in the cranial vena cava. Venous blood volume, venous vessel tone, and cardiac output determine the CVP. CVP is an estimate of preload, not a direct measurement. Cardiac output is one of the factors along with venous tone and venous blood volume in obtaining CVP. Afterload is the force against which the myocardium must contract.

Central venous pressure (CVP) is a direct measure of blood pressure in the cranial vena cava. It is influenced by venous blood volume, venous vessel tone, and cardiac output. CVP estimates preload but does not directly measure it. Afterload is the resistance against which the myocardium contracts.

137
Q

Which abnormal heart sound are you least likely to be able to auscult in a dog?

Atrial fibrillation
Gallop rhythm
First degree AV block
Sinus arrhythmia

A

Answer: First degree AV block

Explanation
The correct answer is first degree AV block as this is a very subtle prolongation between the P and the R wave which can usually only be recognized by evaluation of an ECG strip.

First degree AV block is least likely to be ausculted in a dog, as it involves a subtle prolongation between the P and R wave detectable only via ECG. Other abnormal heart sounds like atrial fibrillation, gallop rhythm, and sinus arrhythmia are more easily ausculted.

138
Q

You diagnose a dog with a larger left to right ventricular septal defect with advanced sequelae to the abnormality. Which of the following is the most likely radiographic appearance of this dog’s thorax?

Right ventricular and left atrial enlargement with pulmonary vasculature hypoperfusion
Left atrial and left ventricular enlargement with pulmonary hyperperfusion
Right ventricular enlargement with pulmonary hyperperfusion and a normal left heart
Enlargement of all 4 heart chambers and hyperperfusion of the pulmonary vasculature

A

Answer: Left atrial and left ventricular enlargement with pulmonary hyperperfusion

Explanation
The correct answer is left atrial and left ventricular enlargement with pulmonary hyperperfusion. In dogs with a VSD, the shunt occurs during systole when both ventricles are contracting so blood is shunted from the left ventricle almost directly into the pulmonary artery. The right ventricle acts as a conduit between the left ventricle and pulmonary artery and is less commonly affected by the shunted blood. The excess blood flow is found in the pulmonary vasculature, left atrium, and left ventricle, thus enlarging these structures.

The most likely radiographic appearance for a dog with a larger left to right ventricular septal defect and advanced sequelae is left atrial and left ventricular enlargement with pulmonary hyperperfusion. In VSD, blood shunts from the left ventricle to the pulmonary artery during systole, causing excess blood flow in the pulmonary vasculature, left atrium, and left ventricle, enlarging these structures.

139
Q

A 15-year old male castrated terrier mix presents for cough and rapid, shallow breathing. You auscult a grade IV/VI left apical holosystolic murmur and crackles throughout both lung fields. An echocardiogram is performed, shown here. What findings are shown on this echocardiogram?

Tricuspid valve endocarditis
Mitral valve prolapse
Pulmonic stenosis
Left ventricular concentric hypertrophy
Right ventricular concentric hypertrophy

A

Answer: Mitral valve prolapse

Explanation
The correct answer is mitral valve prolapse. Note how the mitral valve bends back into the left atrium consistent with mitral valve prolapse. The valve is also thick and mitral regurgitation is likely, which fits with the heart murmur reported. The left ventricular lumen is dilated and walls appear of normal thickness, so concentric hypertrophy is not present. The pulmonic valve is not seen in this image plane. The right ventricle is not well seen either, but in the very near field, the right ventricular wall appears normal. What can be seen of the tricuspid valve does not appear to have any vegetations consistent with endocarditis.

The echocardiogram shows mitral valve prolapse, evidenced by the mitral valve bending back into the left atrium. The valve is thickened, likely causing mitral regurgitation, consistent with the reported heart murmur. The left ventricular lumen is dilated with normal wall thickness, ruling out concentric hypertrophy. The pulmonic valve and right ventricle are not visible, and the tricuspid valve lacks vegetations indicative of endocarditis.

140
Q

Mitral regurgitation in dogs eventually results in which of the following?

Eccentric hypertrophy of the left atrium and concentric hypertrophy of the left ventricle
Eccentric hypertrophy of the left atrium and left ventricle
Concentric hypertrophy of the left atrium and eccentric hypertrophy of the left ventricle
Concentric hypertrophy of the left atrium and left ventricle

A

Answer: Eccentric hypertrophy of the left atrium and left ventricle

Explanation
The correct answer is eccentric hypertrophy of the left atrium and left ventricle. Mitral regurgitation is the process of blood flowing back from the left ventricle to the left atrium due to a defect in the valve. The excess blood flow causes volume overload of the left atrium leading to eccentric hypertrophy or dilation of the chamber. The excess blood volume will then return to the left ventricle during the next diastole, resulting in dilation and eccentric hypertrophy of the left ventricle as well. Concentric hypertrophy occurs in cardiac chambers when they are pressure overloaded, such as in the case of pulmonic stenosis or aortic stenosis.

Mitral regurgitation in dogs leads to eccentric hypertrophy of the left atrium and left ventricle. The regurgitation causes volume overload, resulting in dilation and eccentric hypertrophy of the left atrium. This excess blood then flows into the left ventricle during diastole, causing its dilation and eccentric hypertrophy. Concentric hypertrophy is associated with pressure overload conditions like pulmonic or aortic stenosis.

141
Q

Which of the following findings is consistent with Tetralogy of Fallot in a dog?

Left ventricular hypertrophy
Aortic stenosis
Polycythemia
Severe generalized cardiomegaly on thoracic radiography

A

Answer: Polycythemia

Explanation
Tetralogy of Fallot is a congenital defect of the heart which includes pulmonic stenosis (not aortic stenosis), dextro-positioned aorta (over-riding), RIGHT ventricular hypertrophy, and a ventricular septal defect. It causes shunting of venous blood across the VSD into the arterial circulation which leads to hypoxemia, and an increased drive for erythropoiesis, causing polycythemia.

Affected animals tend be underdeveloped in size and have a history of exercise intolerance, dyspnea, tachypnea, or syncope due to hypoxemia. Thoracic radiographs usually show a normal to mildly enlarged cardiac silhouette with right ventricular enlargement and pulmonary hypoperfusion due to the shunting of venous blood into the arterial circulation.

Polycythemia is consistent with Tetralogy of Fallot in dogs. This congenital defect includes pulmonic stenosis, dextro-positioned aorta, right ventricular hypertrophy, and a ventricular septal defect, leading to venous blood shunting into the arterial circulation. This results in hypoxemia, driving erythropoiesis and causing polycythemia. Affected dogs often exhibit exercise intolerance, dyspnea, tachypnea, or syncope, with thoracic radiographs showing a normal to mildly enlarged cardiac silhouette, right ventricular enlargement, and pulmonary hypoperfusion.

142
Q

Which of the following findings is consistent with Tetralogy of Fallot in a dog?

Left ventricular hypertrophy
Aortic stenosis
Polycythemia
Severe generalized cardiomegaly on thoracic radiography

A

Answer: Polycythemia

Explanation
Tetralogy of Fallot is a congenital defect of the heart which includes pulmonic stenosis (not aortic stenosis), dextro-positioned aorta (over-riding), RIGHT ventricular hypertrophy, and a ventricular septal defect. It causes shunting of venous blood across the VSD into the arterial circulation which leads to hypoxemia, and an increased drive for erythropoiesis, causing polycythemia.

Affected animals tend be underdeveloped in size and have a history of exercise intolerance, dyspnea, tachypnea, or syncope due to hypoxemia. Thoracic radiographs usually show a normal to mildly enlarged cardiac silhouette with right ventricular enlargement and pulmonary hypoperfusion due to the shunting of venous blood into the arterial circulation.

Polycythemia is consistent with Tetralogy of Fallot in dogs. This congenital defect includes pulmonic stenosis, dextro-positioned aorta, right ventricular hypertrophy, and a ventricular septal defect, leading to venous blood shunting into the arterial circulation. This results in hypoxemia, driving erythropoiesis and causing polycythemia. Affected dogs often exhibit exercise intolerance, dyspnea, tachypnea, or syncope, with thoracic radiographs showing a normal to mildly enlarged cardiac silhouette, right ventricular enlargement, and pulmonary hypoperfusion.

143
Q

A 7 month old male Labrador Retriever presents for lethargy, a distended abdomen, and exercise intolerance. Your physical exam reveals a 4/6 systolic murmur heard best at the right mid thorax. Thoracic radiographs show severe cardiomegaly with marked right atrial enlargement and right ventricular enlargement. The caudal vena cava is enlarged, and part of the abdomen in the film shows loss of serosal detail and hepatomegaly. What is your most likely diagnosis?

Pulmonic stenosis
Tricuspid dysplasia
Hemangiosarcoma
Hepatocellular carcinoma

A

Answer: Tricuspid dysplasia

Explanation
The correct answer is tricuspid dysplasia. Tricuspid dysplasia is similar to mitral valve dysplasia in the irregular valve leaflets, chordae tendinae, or papillary muscles of the valve. Labrador Retrievers, German Shepherd Dogs, and other large breed males are predisposed to the disease. Signs of right heart failure, such as ascites and hepatomegaly occur in advanced disease. The prognosis for these animals is guarded to poor depending on the severity of the regurgitation. Pulmonic stenosis causes a pressure overload of the right heart and causes concentric hypertrophy, which may or may not be detectable by radiography. The murmur with PS is heard better at the left heart base. Hemangiosarcoma and hepatocellular carcinoma are less likely in a 7 month old puppy.

Tricuspid dysplasia is the most likely diagnosis for the 7-month-old Labrador Retriever presenting with lethargy, distended abdomen, and exercise intolerance. The condition involves irregular tricuspid valve leaflets, chordae tendineae, or papillary muscles, leading to right atrial and ventricular enlargement, ascites, and hepatomegaly. Labrador Retrievers and other large breed males are predisposed. Pulmonic stenosis would cause right heart concentric hypertrophy and a murmur at the left heart base. Hemangiosarcoma and hepatocellular carcinoma are less likely in such a young dog.

144
Q

Dopamine is often used for which of the following reasons?

To increase contractility of the heart
To increase heart rate
To increase arterial blood pressure
To decrease blood pressure

A

Answer: To increase arterial blood pressure

Explanation
The correct answer is to increase blood pressure. High doses of dopamine are used as an alpha agonist to increase peripheral resistance and increase blood pressure. It is often used during anesthesia and emergency situations to alleviate hypotension.

Dopamine is often used to increase arterial blood pressure. At high doses, dopamine acts as an alpha agonist, increasing peripheral resistance and blood pressure. It is commonly administered during anesthesia and in emergency situations to counteract hypotension.

145
Q

A 1-year old German Shepherd Dog presents for its first physical exam. Cardiac auscultation reveals a continuous murmur. Thoracic radiographs and echocardiography lead you to the diagnosis of a very large, cylinder-shaped patent ductus arteriosus with left-to-right shunting of blood. Which of the following statements is most true?

Furosemide and enalapril are the most appropriate forms of treatment for a cylinder-shaped PDA
Coil embolization is the most appropriate form of treatment for a cylinder-shaped PDA
Surgical ligation is the most appropriate form of treatment for a cylinder-shaped PDA
Placement of constrictor ring is the most appropriate form of treatment for a cylinder-shaped PDA

A

Answer: Surgical ligation is the most appropriate form of treatment for a cylinder-shaped PDA

Explanation
The correct answer is surgical ligation of the PDA is the most appropriate form of treatment. A large cylindrical PDA is unlikely amenable to closure via traditional coil embolization. Most canine PDAs are funnel-shaped with the smaller end of the funnel attached to the pulmonary artery, and the wider end attached to the aorta. The funnel shape is what allows the coils to lodge in place and cause closure of the PDA. Coils would not stay in place in a large, cylindrical PDA, so surgical ligation is the treatment of choice for this patient. There are newer nickel-titanium devices (Amplatz Canine Ductal Occluder) that show promise for minimally-invasive closure of PDAs which do not taper, but this answer was not provided. Ameroid constrictor rings are typically used for ligating hepatic shunts, and not for PDAs. Medical management with diuretics and heart drugs are only indicated prior to correction of the PDA if the dog is in heart failure.

Surgical ligation is the most appropriate treatment for a large, cylinder-shaped PDA in a 1-year-old German Shepherd Dog. Traditional coil embolization is ineffective for cylindrical PDAs, as coils require a funnel shape to lodge properly. Newer devices like the Amplatz Canine Ductal Occluder show promise but weren’t options here. Ameroid constrictor rings are for hepatic shunts, and medical management with diuretics and heart drugs is only for pre-correction heart failure.

146
Q

Using phenothiazines in dogs causes which of the following effects?

Increase in hematocrit
Increases in myocardial sensitivity to catecholamine induced arrhythmias in the heart
Hypotension
Analgesia

A

Answer: Hypotension

Explanation
The correct answer is hypotension. Phenothiazines cause sedation, but do not provide analgesia. They may decrease hematocrit due to splenic sequestration of red blood cells. They decrease the sensitivity of the myocardium to catecholamine-induced arrhythmias.

Phenothiazines cause hypotension in dogs. They provide sedation without analgesia, may decrease hematocrit due to splenic sequestration of red blood cells, and decrease myocardial sensitivity to catecholamine-induced arrhythmias.

147
Q

A 7 year-old male neutered Doberman has recently been diagnosed with dilated cardiomyopathy. On your physical exam, you auscult a “tennis shoes in a dryer” sound. What heart rhythm are you describing?

Bradycardia
Tachycardia
Atrial fibrillation
Respiratory sinus arrhythmia
Atrioventricular block

A

Answer: Atrial fibrillation

Explanation
Patients with enlarged atria can develop the arrhythmia atrial fibrillation. Atrial fibrillation is characterized by a rapid irregularly irregular rhythm. Primary atrial fibrillation occurs idiopathically in some large-breed dogs, and patients are usually asymptomatic. Secondary atrial fibrillation is more common and occurs secondary to underlying cardiac disease.

Atrial fibrillation is the heart rhythm described by the “tennis shoes in a dryer” sound in a 7-year-old Doberman with dilated cardiomyopathy. It is characterized by a rapid, irregularly irregular rhythm and commonly occurs in dogs with enlarged atria due to underlying cardiac disease.

148
Q

A 11-year old male castrated Golden Retriever presents for collapse with muffled heart sounds on examination. You ultrasound the heart and obtain the following image; the right atrium (RA) and right ventricle (RV) are labeled. A large mass is seen in the right atrioventricular groove with pericardial effusion. What is the likely diagnosis?

Hemangiosarcoma
Lymphoma
Chemodectoma
Idiopathic pericardial hemorrhage
Pulmonary adenocarcinoma

A

Answer: Hemangiosarcoma

Explanation
The correct answer is hemangiosarcoma. Hemangiosarcoma of the heart has a predilection for the right auricle of dogs, with Golden Retrievers being predisposed. Stabilization of this patient will require pericardiocentesis; prognosis is guarded and may include chemotherapy, pericardiectomy, or rarely auriculectomy. Chemodectoma arises from the ascending aorta and heart base; lymphoma is rare in the heart of dogs and is more commonly metastatic than a single mass. Idiopathic pericardial hemorrhage is incorrect because of the clear evidence of a mass.

The likely diagnosis is hemangiosarcoma. Hemangiosarcoma often affects the right auricle in dogs, with Golden Retrievers being predisposed. Stabilization involves pericardiocentesis; prognosis is guarded and may involve chemotherapy, pericardiectomy, or rarely auriculectomy. Chemodectoma arises from the ascending aorta and heart base. Lymphoma in the heart is rare and typically metastatic. Idiopathic pericardial hemorrhage is excluded due to the evident mass.

149
Q

Which of the following is used as a medical management option in congestive heart failure in a dog?

Lidocaine
Mirtazapine
Increased sodium intake
Furosemide

A

Answer: Furosemide

Explanation
The correct answer is furosemide. Sodium-restricted diets, ACE inhibitors such as enalapril, and diuretics such as furosemide decrease blood volume and preload. ACE inhibitors also reduce afterload by preventing production of angiotensin-ll. a potent vasoconstrictor. Pimobendan is also beneficial in the management of congestive heart failure in the dog due to its positive inotropic effects.

Lidocaine is used for treating ventricular arrhythmias. Mirtazapine is an appetite stimulator and not a drug used to treat CHF.

150
Q

A 9 year old female spayed black Labrador Retriever presents to your clinic with the complaint of collapse and lethargy. On exam, the dog is subdued but responsive with a temperature of 100.9F (38.3 C), heart rate of 170 beats per minute, and respiratory rate of 40 breaths per minute. You perform chest radiographs which are shown below. A CBC shows a hematocrit of 31% (35-57%), neutrophil count of 6,275/ul (2,900-12,000/ul), monocyte count of 984/ul (100-1,400/ul) and eosinophil count of 630/ul (0-1,300/ul). What is the treatment of choice?

Terbutaline
Melarsomine
Broad spectrum antibiotics
Furosemide
Pericardiocentesis

A

Answer: Pericardiocentesis

Explanation
The correct answer is pericardiocentesis. This radiograph and case are a good example of pericardial effusion with the classic finding of a markedly enlarged heart with a globoid shape. The most common type of effusion by far is hemorrhage which can be idiopathic or from neoplasia such as hemangiosarcoma. Based on this finding, in conjunction with the dog’s tachycardia, a pericardial tap for both diagnostic and therapeutic purposes is indicated. Furosemide is contraindicated in cases of pericardial effusion because it will decrease preload in the right side of the heart and can promote circulatory failure. Melarsomine is the treatment for heartworm.

Antibiotics are very unlikely to help this dog unless the effusion is from some sort of infectious pericarditis, which would be extremely unusual in dogs.

Annotations:
Red: The cardiac silhouette is globoid.
Yellow: Note the pulmonary metastatic nodules.
Green: Concurrent pleural effusion.

151
Q

A 5-year old Pug in an animal shelter dies after experiencing cough, exercise intolerance, and dyspnea as well as ascites and hepatomegaly secondary to congestive heart failure. You perform a necropsy and find the parasite shown in the image below residing primarily in the pulmonary artery. How is this parasite transmitted?

Mosquito
Aerosol
Tick
Fecal-oral
Flea

A

Answer: Mosquito

Dirofilaria immitis or heartworm is the parasite in dogs that resides in the pulmonary artery and can cause congestive heart failure if not recognized and treated. Mosquitoes carrying infective heartworm larvae transmit the infection through bites. The larvae then grow and develop over several months to become sexually mature worms. They then release offspring (microfilariae) into the blood stream.

152
Q

This ECG recording (bottom recording) is from a 10-year old miniature poodle with a history of heart disease that is untreated. She presented for a 3-day history of diarrhea and decreased appetite. On physical examination you auscult a grade Ill/VI heart murmur and clear lung sounds. There are no obvious pulse deficits palpated. What is the appropriate treatment for this arrhythmia?

Give the patient a bolus of lidocaine and begin continuous infusion for twelve hours and then try to wean off the lidocaine
Begin the patient on oral mexiletine and recheck an ECG strip in 5 days
No anti-arrhythmic therapy is necessary at this time
Give the patient an intravenous injection of furosemide and begin oral administration of furosemide

A

Answer: No anti-arrhythmic therapy is necessary at this time

There are several general guidelines/recommendations regarding treatment of ventricular premature contractions which is what is shown on the ECG strip. These include a heart rate of over 180 bpm, pulse deficits, clinical signs, and VPCs for over 20 seconds in duration. In this case, the heart rate is visible on the recording and clearly not elevated. The patient’s physical exam findings are not suggestive of an immediate problem secondary to the arrhythmia. There are many dogs with intermittent VPCs that are not of clinical importance.

However, it is recommended that the patient be evaluated regularly and ideally have a consultation with a cardiologist to ensure the progression of heart disease is being minimized.

Furosemide is not indicated as there is not an indication that the patient is fluid overloaded or in fulminant heart failure.

Mexiletine is a Class IB anti-arrhythmic and is not necessary given the lack of severity in the ECG findings.

No anti-arrhythmic therapy is necessary at this time. The ECG shows ventricular premature contractions (VPCs) without clinical signs such as an elevated heart rate over 180 bpm, pulse deficits, or sustained VPCs over 20 seconds. Regular evaluation and cardiologist consultation are recommended to monitor heart disease progression. Furosemide is unnecessary as there’s no indication of fluid overload or heart failure. Mexiletine, a Class IB anti-arrhythmic, is also not needed given the mild ECG findings.

153
Q

A 6-year old female spayed Beagle presents for lethargy. On exam you detect a heart rate of 25 beats per minute and record an ECG, which is shown here. What is the treatment of choice?

Pacemaker implantation
Atropine administered intramuscularly
Doxycycline
Furosemide
Sotalol

A

Answer: Pacemaker implantation

Explanation
The correct answer is pacemaker implantation. The rhythm is complete heart block (also known as 3rd degree AV block). There is no consistent PR interval; the ventricular beats are escape beats. Atropine can be considered, but is typically not effective in 3rd degree AV block. The only way to restore a normal heart rate in this patient is by implanting a pacemaker, which will require referral to a cardiologist.

Sotalol is a ventricular antiarrhythmic, which would actually be dangerous in this patient because it may suppress the ventricular escape focus. Furosemide and doxycycline will not prove beneficial for this rhythm disturbance.

Pacemaker implantation is the treatment of choice for a 6-year-old Beagle with complete heart block (3rd degree AV block), indicated by a heart rate of 25 bpm and an ECG showing no consistent PR interval with escape beats. Atropine is typically ineffective in 3rd degree AV block. Sotalol, a ventricular antiarrhythmic, could suppress the ventricular escape focus, making it dangerous. Furosemide and doxycycline are not beneficial for this rhythm disturbance.

154
Q

A 12-year old female Dalmatian has a Holter monitor placed for an arrhythmia heard on exam. The following trace is from that monitor; similar abnormalities were seen frequently throughout the 24hrs. What is your treatment recommendation for this arrhythmia?

Mexiletine
Diltiazem and furosemide
Amlodipine
Pimobendan

A

Answer: Mexiletine

Explanation
The correct answer is mexiletine. The Holter ECG shows a couplet of ventricular premature complexes. If frequent ventricular ectopy is seen, particularly with runs or R-on-T episodes, treatment is advised with oral ventricular antiarrhythmics. Mexiletine is an oral class I antiarrhythmic that is well-tolerated in dogs.

Pimobendan improves systolic function, but does not treat arrhythmias. Amlodipine is a calcium channel blocker used for patients with systemic hypertension. Diltiazem is used for supraventricular tachycardias to slow conduction down the AV node. Furosemide is a diuretic used in the treatment of heart failure, it does not have antiarrhythmic properties.

The correct treatment recommendation for the arrhythmia is mexiletine. The Holter ECG shows a couplet of ventricular premature complexes, and frequent ventricular ectopy, particularly with runs or R-on-T episodes, warrants treatment with oral ventricular antiarrhythmics. Mexiletine, a class I antiarrhythmic, is well-tolerated in dogs. Pimobendan, amlodipine, diltiazem, and furosemide do not address ventricular arrhythmias and serve different cardiac or hypertensive conditions.

155
Q

A 7-year old male castrated Doberman Pinscher presents for lethargy and anorexia. Your physical exam reveals a 3/6 left systolic heart murmur and a rectal temperature of 103.5F (39.7 C). One of your differential diagnoses for this dog is infective endocarditis. Which area of the heart is most commonly affected in a dog with infective endocarditis?

Mitral and aortic valves
Pulmonic and aortic valves
Mitral and tricuspid valves
Ventricular endocardium
Tricuspid and pulmonic valves

A

Answer: Mitral and aortic valves

Explanation
The correct answer is mitral and aortic valves. Dogs with endocarditis are most commonly affected on the mitral and aortic valves. The tricuspid and pulmonary valves are rarely affected in dogs and cats, but are the more common sites of infection in large animal species.

The mitral and aortic valves are most commonly affected in dogs with infective endocarditis. In contrast, the tricuspid and pulmonary valves are rarely involved in dogs and cats but are more commonly infected in large animal species. Thus, in a 7-year-old Doberman Pinscher with signs suggestive of endocarditis, focus on the mitral and aortic valves.

156
Q

A 9 year old female spayed Greyhound presents to you for evaluation of a cough of 2 weeks duration and respiratory difficulty. The owner reports that the dog collapsed while chasing her ball yesterday. On your exam, heart sounds are quiet and the dog’s mucous membranes appear pale. You elect to take thoracic radiographs which are shown below. What is an appropriate treatment?

Atenolol
Measure taurine levels
Heartworm test
Pericardiocentesis
Furosemide

A

Answer: Pericardiocentesis

The radiograph and findings are consistent with pericardial effusion. Note the markedly enlarged globoid cardiac silhouette on the DV projection. Pericardiocentesis is necessary for therapeutic and diagnostic purposes.

Atenolol and furosemide would be contraindicated. While this dog is showing signs of heart failure, furosemide would decrease preload on the heart and make diastolic filling even worse in cases of pericardial effusion.

Atenolol would decrease contractility. This is not a case of heartworm. Radiographic signs of heartworm include right sided cardiac hypertrophy and prominent pulmonary arteries. Taurine levels could be measured in dogs with dilated cardiomyopathy but will take days to get back and this dog needs to be treated immediately. If facilities permit, even a brief echocardiogram could be performed to confirm pericardial effusion and/or to guide the pericardiocentesis.

Pericardiocentesis is the appropriate treatment for a 9-year-old Greyhound with a cough, respiratory difficulty, and a markedly enlarged globoid cardiac silhouette on thoracic radiographs, indicating pericardial effusion. Atenolol and furosemide are contraindicated as they would worsen diastolic filling and decrease contractility, respectively. Heartworm test and taurine level measurement are unnecessary in this immediate context. An echocardiogram can confirm the diagnosis and guide pericardiocentesis.

157
Q

An 11-year old female spayed Shih Tzu presents for her annual physical exam. She appears to be doing fine at home. Cardiac auscultation reveals a 2/6 left systolic heart murmur over the apex of the heart. Color Doppler echocardiography reveals a small regurgitant jet across the mitral valve. What is the most likely cause for the mitral regurgitation in this dog?

Congenital mitral dysplasia
Bacterial endocarditis
Myxomatous mitral valve degeneration
Systolic anterior motion

A

Answer: Myxomatous mitral valve degeneration

Explanation
The correct answer is myxomatous mitral valve degeneration. This disease is the most common cardiac disease in veterinary medicine and affects up to 60% of dogs older than 8 years of age. It commonly occurs in small breed dogs and can clinically range from having only a soft murmur to being in congestive heart failure. It is caused by degeneration and accumulation of acid-staining mucopolysaccharides in the cardiac valves and most commonly affects the mitral valve. A dog with bacterial endocarditis would show signs of systemic illness such as fever and lethargy. Congenital mitral dysplasia would usually result in more marked cardiac changes (louder murmur, more regurgitation, dilated heart chambers) at a much earlier age. Systolic anterior motion or SAM of the mitral valve occurs with hypertrophic cardiomyopathy as one of the leaflets of the mitral valve obstructs the left ventricular outflow tract as the chamber contracts.

The most likely cause of mitral regurgitation in this 11-year-old Shih Tzu is myxomatous mitral valve degeneration. This common cardiac disease in dogs over 8 years old, especially small breeds, ranges from a soft murmur to congestive heart failure. It involves degeneration and mucopolysaccharide accumulation in the mitral valve. Bacterial endocarditis, congenital mitral dysplasia, and systolic anterior motion are less likely due to their distinct presentations and associated symptoms.

158
Q

This is an ECG recording of a 9-year old male castrated Golden Retriever that had a bleeding splenic mass surgically removed approximately 4 hours ago. The heart rate is currently 186 bpm. The patient received 0.08mg/kg of hydromorphone approximately 45 minutes ago. PCV is currently at 24 (normal 33-58%). What is the best treatment option?

Provide a lidocaine bolus followed by a continuous infusion of lidocaine
This patient is severely anemic and requires a blood transfusion immediately
This is not an alarming heart rate or ECG and requires no treatment
The patient is having breakthrough pain due to the low dose of hydromorphone and should be given an adequate dose of hydromorphone

A

Answer: Provide a lidocaine bolus followed by a continuous infusion of lidocaine

Explanation
This patient is exhibiting premature ventricular contractions which is commonly observed after splenectomy.

Any patient that has a splenectomy should be monitored closely for that reason. Most of the time these are not severe enough to require treatment. There are some general guidelines for when to treat VPCs which include excessive pulse deficits, the patient is clinically affected, a heart rate greater than 180 bpm, or greater than 20 second stretch of continuous VPCs.

In this case it is important to know that the patient was just given a dose of post-operative pain medication (at an appropriate dose) which rules out an elevated heart rate secondary to pain.

The patient is anemic, but he is not anemic to the point of requiring an emergency transfusion. Transfusions are usually not administered unless there is expected ongoing blood loss (which is not the case) or the PCV is less than 20.

Lidocaine is an anti-arrhythmic and is typically the drug of choice for treatment of VPCs. A bolus is administered first to determine if the rhythm is responsive to lidocaine and then a CRI is initiated at 20-80 micrograms per kilogram per minute.

The best treatment option is to provide a lidocaine bolus followed by a continuous infusion of lidocaine. This Golden Retriever is experiencing premature ventricular contractions (VPCs), common after splenectomy. Lidocaine, an anti-arrhythmic, is the drug of choice for VPCs, especially when the heart rate exceeds 180 bpm. Although the patient is anemic, the PCV level does not indicate the need for an emergency transfusion, and the recent hydromorphone dose rules out pain as the cause of the elevated heart rate.

Antiarrhythmics for Use in Animals: Comprehensive Overview

Definitions and Classes of Antiarrhythmics

•	Antiarrhythmics: Drugs used to treat abnormal heart rhythms.
•	Classes: Based on the Vaughan Williams classification.
•	Class I: Sodium channel blockers.
•	Class II: Beta-blockers.
•	Class III: Potassium channel blockers.
•	Class IV: Calcium channel blockers.

Class I: Sodium Channel Blockers

•	Examples: Quinidine, Procainamide, Lidocaine.
•	Mechanism: Inhibit sodium influx, reducing excitability and conduction.
•	Uses: Atrial and ventricular arrhythmias.

Class II: Beta-Blockers

•	Examples: Propranolol, Atenolol.
•	Mechanism: Block beta-adrenergic receptors, reducing heart rate and contractility.
•	Uses: Supraventricular and ventricular tachyarrhythmias.

Class III: Potassium Channel Blockers

•	Examples: Amiodarone, Sotalol.
•	Mechanism: Prolong repolarization by inhibiting potassium channels.
•	Uses: Atrial fibrillation, ventricular tachycardia.

Class IV: Calcium Channel Blockers

•	Examples: Verapamil, Diltiazem.
•	Mechanism: Inhibit calcium influx, reducing conduction through the AV node.
•	Uses: Supraventricular tachyarrhythmias.

Indications and Contraindications

•	Indications: Based on specific arrhythmias.
•	Contraindications: Vary per drug class and individual drug profiles.

Monitoring and Side Effects

•	Monitoring: ECG, blood pressure, and heart rate.
•	Side Effects: Hypotension, bradycardia, gastrointestinal disturbances, hepatotoxicity.

Diagnostics

•	ECG: Essential for diagnosing type and severity of arrhythmia.
•	Holter Monitoring: For intermittent arrhythmias.

Treatment Protocols

•	Acute Management: IV administration in emergency situations.
•	Chronic Management: Oral medications for long-term control.

Summary Points for BCSE Test

•	Understand Drug Classes: Mechanisms, examples, and uses.
•	Monitoring and Side Effects: Critical for safe administration.
•	Diagnostics: ECG and Holter monitoring.
•	Treatment Protocols: Acute vs. chronic management.
159
Q

**Which of the following is the treatment of choice for pulmonic stenosis in the dog?*

Surgical valvulotomy
Patch grafting of the outflow tract
Valvulotomy by catheterization device
Balloon valvuloplasty

A

Answer: Balloon valvuloplasty

Explanation
The correct answer is balloon valvuloplasty. This procedure is performed by inserting a catheter into either a jugular vein or a femoral vein and advancing it into the region of stenosis at the pulmonic outflow tract. Once in place, a balloon surrounding the catheter is inflated to stretch the stenotic region open. Surgical valvulotomy or patch grafting is the treatment used for severe cases that cannot be balloon-dilated.

The treatment of choice for pulmonic stenosis in dogs is balloon valvuloplasty. This procedure involves inserting a catheter into a vein and inflating a balloon at the stenotic pulmonic outflow tract to widen the narrowed area. Surgical valvulotomy or patch grafting is reserved for severe cases not responsive to balloon dilation.

160
Q

A 7-year old male intact Chesapeake Bay Retriever presents to your clinic with the presenting complaint of an intermittent cough. On exam, the dog is bright and alert with a temperature of 100.5F (38.1 C), heart rate of 110 beats per minute and respiratory rate of 30 breaths per minute. You perform chest radiographs which are shown below. A CBC shows a hematocrit of 39% (35-57%), neutrophil count of 8,659/ul (2,900-12,000/ul), monocyte count of 984/ul (100-1,400/ul) and eosinophil count of 1,980/ul (0-1,300/ul). What is the treatment of choice for the most likely diagnosis?

Pericardiocentesis
Immiticide (Melarsomine)
Enrofloxacin
Furosemide
Terbutaline

A

Answer: Immiticide (Melarsomine)

Explanation
The correct answer is Immiticide (melarsomine). The dog in this radiograph has the classic findings for heartworm disease, including right sided ventricular enlargement with very prominent pulmonary arteries.
Heartworm in the dog is caused by Dirofilaria immitis. The treatment of choice for heartworm is Immiticide which is given by intramuscular injection.

Furosemide is a treatment for congestive heart failure, which this dog shows no evidence of. Terbutaline is a bronchodilator, which would be of little use in this case. Broad-spectrum antibiotics are useful for the treatment of pneumonia and other infections but are not effective against heartworm. Pericardiocentesis is obviously the treatment for pericardial effusion, but this dog’s radiographs do not show the classic globoid heart you might see with pericardial effusion.

Annotations:
Green arrow: apex of the heart shifted towards the left hemithorax indicative of right cardiac enlargement
Blue: right sided cardiac enlargement of right atrium and right ventricle
Yellow: Severely enlarged main pulmonary artery
Red: pulmonary arteries are enlarged and mildly tortuous.

All the changes are indicative of pulmonary hypertension secondary to heartworm disease.

Throughout the lungs there is a moderate diffuse bronchointerstial pattern throughout, most suggestive of eosinophilic pneumonitis.

Radiographs provided by Adrien Hespel, DVM, MS, DACVR and University of Tennessee College of Veterinary Medicine.

*** PowerPage: Heartworm disease

The treatment of choice for this case is Immiticide (melarsomine). The dog’s radiographs and clinical findings, including right ventricular enlargement and prominent pulmonary arteries, are indicative of heartworm disease caused by Dirofilaria immitis. Immiticide is administered via intramuscular injection to treat heartworm. Other treatments, such as furosemide, terbutaline, antibiotics, or pericardiocentesis, are not appropriate for this diagnosis, as they address different conditions.

Heartworm disease in dogs, cats, and ferrets. Comprehensive list of key points relevant for the NAVLE:

General Information:

•	Causative Agent: Dirofilaria immitis (filarial nematode).
•	Vector: Mosquitoes (e.g., Aedes, Anopheles, Culex).
•	Species Affected: Primarily dogs, but also cats, ferrets, and occasionally other species like coyotes and foxes.

Pathogenesis:

•	Life Cycle: Mosquitoes transmit infective larvae (L3), which migrate into the host, molt, and mature into adults in the pulmonary arteries.
•	Pulmonary Disease: The disease affects the pulmonary arteries, causing inflammation, endarteritis, and pulmonary hypertension. Long-term infections can lead to chronic lesions, scarring, and heart failure.
•	Immunological Response: Wolbachia bacteria within the heartworms contribute to the inflammatory response and disease severity. Antigenic stimulation leads to immune responses, causing further damage.

Clinical Findings:

•	Dogs: Most common signs include cough, exercise intolerance, and signs of right-sided heart failure (e.g., ascites). Severe cases can lead to caval syndrome with signs like hemolysis, anemia, and hepatorenal dysfunction.
•	Cats and Ferrets: Often subclinical or mild, but severe cases can present with respiratory distress, weight loss, lethargy, vomiting, and sudden death. Cats can suffer from HARD (heartworm-associated respiratory disease).

Diagnosis:

•	Serology: Antigen testing is the primary method in dogs, while antibody testing is more useful in cats due to the lower sensitivity of antigen tests in cats.
•	Microfilaria Detection: Direct smear or modified Knott test; useful as an adjunct in dogs.
•	Imaging: Thoracic radiography, echocardiography (particularly important in cats and ferrets).

Treatment:

•	Dogs: Adulticide therapy with melarsomine (an organic arsenic compound), typically in a three-dose protocol. Adjunct therapy with doxycycline to target Wolbachia. Supportive care and exercise restriction are crucial.
•	Cats and Ferrets: No safe adulticide therapy; treatment focuses on managing symptoms with corticosteroids, oxygen therapy, and occasionally surgical removal of heartworms.

Prevention:

•	Macrocyclic Lactones: Regular administration of ivermectin, milbemycin, selamectin, or moxidectin for prevention. For dogs, these drugs also serve as microfilaricides.

Stages of Disease:

1.	Stage I: Subclinical or minimal signs.
2.	Stage II: Mild to moderate signs, typically including cough.
3.	Stage III: Severe signs, such as weight loss, respiratory distress, and right heart failure.
4.	Stage IV: Caval syndrome with severe cardiopulmonary compromise.
161
Q

You are working at a small animal clinic performing a cystotomy on a 7 year old male Dalmatian. You glance at your anesthetic monitoring equipment and notice that the oxygen saturation on your pulse oximeter is 90%. Assuming that your equipment is working normally, what does this mean about your patient’s oxygenation status?

This is concerning because normally, a patient’s oxygen saturation does not exceed 50%
Arterial oxygen pressure is 60 mm Hg, and oxygen saturation may drop precipitously if it decreases further
This oxygen saturation is low for a patient under anesthesia breathing 100% oxygen but would be normal for an animal breathing room air, which is 21% oxygen
Arterial oxygen pressure is 40 mm Hg and will result in potentially serious tissue hypoxia due to inadequate
oxygen delivery
Arterial oxygen pressure is 100 mm Hg, which is normal for this patient

A

Answer: Arterial oxygen pressure is 60 mm Hg, and oxygen saturation may drop precipitously if it decreases further

Explanation
When assessing an anesthetized patient with pulse oximetry, it is important to realize several things:
The shape of the oxygen saturation curve is sigmoid, and until you reach oxygen pressures of ‹ 60 mm Hg, there is very little change in oxygen saturation. Because an anesthetized patient breathing 100% oxygen should have about 500 mm Hg oxygen in the arterial blood, this can drop a long way before a dramatic change to oxygen saturation occurs.

However, the time to intervene is early when you see saturation percentages decrease because it means that your patient is oxygenating very poorly, especially if they are breathing 100% oxygen, and even a relatively small further decline could dramatically reduce their oxygen saturation, resulting in tissue hypoxia and dangerous consequences.

  • An oxygen saturation of 98-100% corresponds to > 100 mm Hg oxygen in arterial blood
  • An oxygen saturation of 95% corresponds to 80 mm Hg An oxygen saturation of 90% corresponds to 60 mm Hg
  • An oxygen saturation of 50% corresponds to 30 mm Hg
  • An oxygen saturation of 10% corresponds to 10 mm Hg

By the way, in room air, arterial oxygen pressure should normally be 80-100 mm Hg and oxygen saturation is
95-100%

The patient’s oxygen saturation of 90% indicates that the arterial oxygen pressure is approximately 60 mm Hg. This is concerning because further decreases could result in a rapid and dangerous drop in oxygen saturation, potentially leading to tissue hypoxia. Since the patient is under anesthesia and likely breathing 100% oxygen, this saturation level is lower than expected and warrants immediate attention to prevent serious complications.

162
Q

Why are supraventricular tachycardias considered less clinically important than ventricular tachycardias in the dog?

Supraventricular tachycardias may progress to atrial fibrillation and cause progressive left ventricular dysfunction, whereas ventricular tachycardias may lead to ventricular fibrillation and death.

Supraventricular tachycardia is actually just as clinically important as ventricular tachycardia because they can both lead to cardiac arrest and death.

Supraventricular tachycardias are easily treated with a beta blocker or calcium channel blocker whereas the treatment for ventricular tachycardia is less efficacious.

Supraventricular tachycardias can occur incidentally and will usually convert back to normal sinus rhythm by themselves. Ventricular tachycardia is caused by severe cardiomyopathies and is indicative of a serious underlying disease.

A

Answer: Supraventricular tachycardias may progress to atrial fibrillation and cause progressive left ventricular dysfunction, whereas ventricular tachycardias may lead to ventricular fibrillation and death.

Explanation
Supraventricular tachycardia can lead to atrial fibrillation, which is not life threatening. With atrial fibrillation, the atrial depolarization rate in a dog is usually from 400-700/minute. Usually, about one third of depolarizations get through the AV node to the ventricles to cause a ventricular contraction. Rapid supraventricular arrhythmias can wear out the LV myocardium causing LV dysfunction. Ventricular tachycardias, however, can cause electrical instability and progress to ventricular fibrillation, which leads to cardiovascular collapse and death if not rapidly converted back to a normal rhythm.

Supraventricular tachycardias are less clinically important than ventricular tachycardias because, while supraventricular tachycardias can lead to atrial fibrillation and progressive left ventricular dysfunction, they are generally not life-threatening. Ventricular tachycardias, however, can cause severe electrical instability, potentially progressing to ventricular fibrillation, which can result in cardiovascular collapse and death if not treated promptly.

163
Q

A 10-year old male mixed breed dog presents to you for intermittent cough since the owner obtained the dog from a rescue organization. On your physical exam, the dog is fearful and underweight (body condition of 2/9). No othe significant abnormalities are noted. Your complete blood count shows:
hematocrit of 33% (35-57%)
platelets- 270,000/ul ( 211,000-621,000/ul)
WBC- 11,500/ul (5,000-14,100/ul)
neutrophils 6,500/ul (2,900-12,000/ul)
monocytes 1,200/ul (100-1,400/ul)
eosinophils 3,800/ul (0-1,300/ul)
lymphocytes 1,500/ul (400-2,900/ul
Chemistry values are:
BUN 12 mg/dl (8-28 mg/dl)
creatinine 0.7 mg/dl (0.5-1.7 mg/dl)
glucose- 99 mg/dl (76-119 mg/dl)
globulin- 6.7 g/dl (2.7-4.4 g/dl) albumin- 2.4 g/dl (2.3-32 g/dl)
ALT-51 U/L (10-109 U/L)
You take the thoracic radiographs shown here.
What is the most important additional diagnostic test that you should recommend to the owner?

Urine cortisol: creatinine ratio
Serum protein electrophoresis
Trypsin-like immunoreactivity (TLI)
Serum lead levels
Fecal flotation
Heartworm antigen test

A

Answer: Heartworm antigen test

Based on the radiograph, you should be concerned about the likelihood of heartworm disease. On the V/D projection, the heart shape is a reverse D, indicative of right sided heart enlargement. There is enlargement of the caudal pulmonary arteries and a caudodorsal pulmonary interstitial pattern. There is no evidence of right heart failure in the films, but these changes are suggestive of heartworm. The antigen test is a sensitive test for this disease in dogs.

Serum lead levels are not important as the dog has no signs of lead toxicity. A fecal flotation would be a good idea in this dog with an eosinophilia, but given the radiographic findings, heartworm is a much more important condition to identify and rule out as a cause first.

A TLI is the test for exocrine pancreatic insufficiency. This dog is underweight, but this is likely due to previous neglect. If he does not gain weight now that he has a good home, you should investigate.

A serum electrophoresis would be a test to evaluate for a monoclonal gammopathy. The hyperglobulinemia in this case is much more likely due to heartworm and the chronic antigenic stimulation than an immunoglobulin-secreting cancer.

A urine cortisol:creatinine ratio to screen for hyperadrenocorticism is not indicated in this patient.

Annotations:
- green arrow: apex of the heart shifted towards the left hemithorax indicative of right cardiac enlargement
- Blue: right sided cardiac enlargement RA and RV
- Right:Left caudal pulmonary artery is enlarged and mildly tortuous.

All the changes are indicative of pulmonary hypertension, most commonly HW
Left: Pulmonary artery of the left cranial is also enlarged and mildly tortuous

Radiographs provided by Adrien Hespel, DVM, MS, DACVR and University of Tennessee College of Veterinary Medicine

The most important additional diagnostic test to recommend is a heartworm antigen test. The thoracic radiographs show a reverse D-shaped heart, indicative of right-sided heart enlargement, along with enlarged and tortuous pulmonary arteries, suggesting pulmonary hypertension commonly seen with heartworm disease. While eosinophilia warrants consideration of fecal flotation, heartworm testing takes priority due to the radiographic findings. Other tests, such as TLI or serum lead levels, are less relevant for this case.

164
Q

Once infected, for what period of time is canine parvovirus usually shed?

  • 7-10 days
  • 2-3 months
  • 1-2 months
  • 5-6 months
A

Answer: 7-10 days

The correct answer is 7-10 days. Canine parvovirus mainly affects puppies and young dogs less than a year of age. Transmission of the virus is through contact with infected feces and fomites such as hands, toys, the dog’s hair coat. The virus is very resilient in the environment and is resistant to many types of disinfectants. It replicates in the crypt epithelium of the gut and causes epithelial necrosis and hemorrhagic diarrhea. The virus can also affect the heart of young puppies, causing myocarditis. This occurs less commonly now since most bitches are immunized against the virus, which allows for maternal antibodies to protect young puppies from this form of the disease.

165
Q

A 10-year old mixed breed dog presents for a 1-month history of anorexia, lethargy, and inappetance. Physical exam reveals pale mucous membranes and a mild tachycardia. The CBC shows:
- MCV 50 fl (66-77 fl)
- MCH 12 pg (21-26 pg)
- MCHC 25 %g/dl (32-36.3 %g/dl)
- HCT 22% (35-57%)
- reticulocyte 40,000/ul (<80,000/ul)
What is the most likely diagnosis for this dog?

  • Babesiosis
  • Congenital portosystemic shunt
  • Iron deficiency
  • Mycoplasma felis infection
  • Anemia of chronic disease
A

Answer: Iron deficiency

Explanation
The correct answer is iron deficiency. The CBC shows a microcytic, hypochromic, non-regenerative anemia. Reticulocyte counts under 60,000/ul are typically considered non-regenerative in most labs. This is consistent with chronic iron deficiency. This most common reason for chronic blood loss is gastrointestinal tract disease such as an ulcer or cancerous lesion.

166
Q

What is the ideal incision site for a foreign body gastrotomy?

  • Along the greater curvature
  • Just proximal to the pylorus
  • There is no preferred site
  • Halfway between the greater and lesser curvature of the stomach
  • Along the lesser curvature
A

Answer: Halfway between the greater and lesser curvature of the stomach

Explanation
The correct answer is halfway between the greater and lesser curvature. At this site, you avoid any major bleeding and retain good blood supply from both sides to help with the healing of your incision.

Cutting along the greater and lesser curvature is not recommended due to the need to avoid blood vessels. Cutting along the pylorus has a risk of disturbing the sphincter and causing the formation of a pyloric stricture. Additionally, this does not provide a good view inside the stomach.

***PowerLecture: Surgery Of The Stomach

167
Q

A 13-year old female Dachshund presents for 2 lumps on her abdomen. Your physical exam reveals that both lumps are 2.5 cm mammary masses in the 5th mammary gland. The associated inguinal lymph node is prominent. Aspirates of the lymph node show sheets of epithelial cells on cytology. Both mammary masses are firm and nodular in shape. Which of the following is true?

  • The lymph node aspirate results are a negative prognostic indicator
  • Mammary tumors in the 5th gland is a negative prognostic indicator
  • Having 2 mammary masses is a negative prognostic indicator
  • The size of each mass is a negative prognostic indicator
  • The firmness and nodularity of the masses is a negative prognostic indicator
A

Answer: The lymph node aspirate results are a negative prognostic indicator

Explanation
You should never find sheets of epithelial cells in a lymph node unless it is infiltrated with metastasis (or unless you aspirated something else on accident, like a salivary gland). Metastasis of mammary gland tumors in dogs is a negative prognostic indicator, whereas none of the other choices above are thought to directly influence prognosis.

168
Q

Dutchess, a five year old female spayed Labrador Retriever, presents to the emergency clinic with a 2 day history of lethargy, inappetance, diarrhea, and vomiting. On physical exam, she is painful on abdominal palpation, 7% dehydrated, and febrile.

Abdominal radiographs were performed and showed evidence of ascites and gas distension in the small intestines.

Preliminary blood work showed a PCV of 32% (35-57 %), TP 2.2 g/dl (5.4-7.5 mg/dl), glucose 57 mg/dl (76-119 mg/dl), Na 140 mEq/L (142-152 mEq/L), Cl 115 mEq/L (110-124 mEq/L), K 3.9 mEq/L (3.9-5.1 mEq/L). Given the surgical findings (see image below) what is the likely abdominal fluid glucose level?

  • At least 20 mg/dl greater than the blood glucose
  • Approximately the same as the blood glucose
  • At least 10 mg/dl less than the blood glucose
  • At least 10 mg/dl greater than the blood glucose
A

Answer: At least 20 mg/dl less than the blood glucose

Explanation
The correct answer is at least 20 mg/dL less than the blood glucose. The image depicts a perforated colon, which will result in a septic abdomen due to the normal colonic flora emptying into the abdomen.

A concentration difference > 20 mg/dL between blood and peritoneal fluid glucose concentration provides a rapid and reliable means to differentiate a septic peritoneal effusion from a nonseptic peritoneal effusion in dogs and cats.

169
Q

A 6-month old Chocolate Labrador presents for limping and failure to gain weight. He is housed in an outdoor kennel with other hunting dogs. He is fed a large breed dry puppy food. He received his puppy shots at 8, 12, and 16 weeks old. He is quiet, alert, and responsive. His body condition score is 3/9, with rib exposure and poor fat deposition. His mucous membranes are pale pink, with a capillary refill time of 2 seconds. His heart and lungs auscult normally. No abnormalities are felt on abdominal palpation. He is an intact male, and both testicles are descended. The only abnormalities are the pads of his two front feet and left hind foot (see image) that he chews at frequently. What is diagnostic test of choice?

  • Biopsy
  • Fecal float
  • Radiographs
  • CBC and chemistry panel
  • PCV and TP
  • Skin scraping
A

Answer: Fecal float

Explanation
Hookworms (Ancylostoma and Uncinaria) are intestinal parasites that suck blood and can cause anemia, enteritis, coughing during larval migration, and dermatitis. Any young dog that is failing to thrive and/or has pale mucous membranes should be tested for intestinal parasites. Hookworm dermatitis, also called Ancylostomiasis is typically seen in conditions with poor sanitation and/or in kennels.

Hookworms can be transmitted in utero, during nursing, or via 3rd stage larva penetrating the skin. The most commonly affected skin areas are the pads and interdigital spaces of the feet, but can include any surface that contacts the ground. The larva migrate through the dog’s tissues before arriving in the intestines. They cause significant anemia, failure to thrive, or sudden death in young dogs. In mild cases, deworming protocols are often enough; with severe cases blood transfusions and parenteral treatments are often necessary.

Skin scraping and impression smears of the affected skin areas are typically unrewarding for isolating parasites.

A PCV/TP would show signs of anemia, but not the underlying etiology. Complete blood cell count and chemistry will often show anemia that is regenerative and an eosinophilia. Radiographs would be unrewarding in this case. A biopsy may show migrating larva if biopsied soon after trauma, however this is not a diagnostic test routinely used

Hookworms in Small Animals - Comprehensive Veterinary Information

Definitions and Terminology:

•	Hookworms: Intestinal nematodes, primarily affecting dogs and cats, leading to significant health issues.

Causative Agents:

•	Species:
•	Ancylostoma caninum (dogs)
•	Ancylostoma tubaeforme (cats)
•	Ancylostoma braziliense (dogs and cats)
•	Ancylostoma ceylanicum (dogs and cats)
•	Uncinaria stenocephala (dogs and cats in cooler regions)

Physiopathology:

•	Life Cycle: Eggs pass in feces, hatch in soil, and larvae infect hosts through ingestion, skin penetration, or transmammary routes.
•	Larval Migration: Skin penetration, blood to lungs, coughed up, swallowed, mature in intestines. Arrested larvae in tissues can reactivate.

Clinical Findings:

•	Acute Anemia: Normocytic, normochromic progressing to hypochromic, microcytic in puppies.
•	Chronic Infections: Anemia, melena, hypoproteinemia, weakness, diarrhea.
•	Lesions: Hemorrhagic enteritis, pneumonia in pups, dermatitis (interdigital spaces).

Diagnosis:

•	Fecal Flotation: Detects thin-shelled, oval eggs.
•	Antigen Tests: For hookworm detection.
•	Postmortem: Examination of intestines for adult worms.

Treatment:

•	Anthelmintics:
•	Fenbendazole
•	Moxidectin
•	Pyrantel pamoate
•	Milbemycin
•	Nitroscanate
•	Supportive Care: Blood transfusions, iron supplements, high-protein diet for severe anemia.
•	Drug Resistance: Monitor efficacy post-treatment, especially in A. caninum cases.

Control and Prevention:

•	Regular Deworming: Based on age and risk factors.
•	Sanitation: Dispose of feces promptly, maintain clean environments.
•	Preventive Measures: Treat pregnant bitches, routine fecal checks.

Key Points:

•	Zoonotic Potential: A. braziliense and A. ceylanicum can infect humans.
•	Drug Resistance: Emerging problem, particularly in the southeastern US.
170
Q

Which of these signs are least compatible with an estrogen secreting Sertoli cell tumor in a dog?

  • Cryptorchidism
  • Aplastic anemia
  • Truncal alopecia
  • Gynecomastia
  • Polyuria and polydipsia
A

Answer: PU/PD

Explanation
The answer is polyuria and polydipsia. Estrogen secreting Sertoli cell tumors are quite common in cryptorchid testes and cause the clinical signs of truncal alopecia, hyperpigmentation,
gynecomastia, urinating in a female position, and bone marrow dyscrasias such as aplastic anemia.

Attraction of other males can also occur. PU/PD is a sign commonly seen with endocrinopathies but not this one.

171
Q

You need to start treatment for a dog that has been diagnosed with Valley Fever (Coccidioides immitis). The fluconazole that you normally use is on backorder, and the human version is extremely expensive. You decide to start ketoconazole in the interim. Which of these is an important potential side effect of ketoconazole administration?

  • Elevated liver enzymes
  • Facial excoriation
  • Increased appetite
  • Decrease GFR
A

Answer: Elevated liver enzymes

Explanation
Ketoconazole is hepatotoxic and can cause elevated liver enzymes. It is a potent inhibitor of P450 enzymes and can significantly affect metabolism of other drugs. It can cause adrenal insufficiency and actually is sometimes used as an alternative treatment for Cushing’s. Anorexia is a common and important side effect, especially in cats.

The kidneys and GFR are not significantly bothered by ketoconazole. Facial excoriation is a potential side effect of methimazole, not ketoconazole.

172
Q

On the morning of a beautiful summer day, a five-year old, male German Pointer played unattended in the backyard garden. During the following 24 hours the right side of the dog’s face became swollen as seen in the image below. The dog developed urticaria and became lethargic, unable to walk and vomited several times. Which of the following are the best treatment options for the likely condition?

  • Diazepam, methocarbamol, and non-steroidal anti-inflammatories
  • Furosemide, oxygen, and nitroglycerin
  • Drain the swelling, administer antibiotics
  • Induce vomiting, administer activated charcoal, intravenous fluids
  • Antihistamines, corticosteroids, and epinephrine
A

Answer: Antihistamines, corticosteroids, and epinephrine

Explanation
The dog has angioedema +/- anaphylaxis, likely caused by an insect bite or sting. A snake bite should also be ruled out by trying to visualize the bite or questioning the owners. This is a type I hypersensitivity reaction and is treated by removing the offending agent if possible and providing supportive care with anthistamines, corticosteroids, and epinephrine as needed.

173
Q

Which of these tumor types is associated with causing Gl ulcers, perioperative hypotension, and coagulation abnormalities in dogs?

  • Melanoma
  • Hemangiosarcoma
  • Mast cell tumors
  • Lymphoma
A

Answer: Mast cell tumors

Explanation
The correct answer is mast cell tumors. Many of the clinical signs of mast cell tumors are related to the factors released by mast cells including histamine, heparin, and other vasoactive amines. Histamine can cause ulcers via activation of H2 receptors on gastric parietal cells. Heparin can cause coagulopathy. Histamine and other vasoactive amines can lead to vasodilation and hypotension.

174
Q

Which of these tumor types is associated with causing Gl ulcers, perioperative hypotension, and coagulation abnormalities in dogs?

  • Melanoma
  • Hemangiosarcoma
  • Mast cell tumors
  • Lymphoma
A

Answer: Mast cell tumors

Explanation
The correct answer is mast cell tumors. Many of the clinical signs of mast cell tumors are related to the factors released by mast cells including histamine, heparin, and other vasoactive amines. Histamine can cause ulcers via activation of H2 receptors on gastric parietal cells. Heparin can cause coagulopathy. Histamine and other vasoactive amines can lead to vasodilation and hypotension.

175
Q

A 9 year-old male neutered Labrador retriever named Manny is presented to you for intermittent lameness. The owners note that Manny is still very enthusiastic (sprinting after his tennis balls, e.g.) but that he seems to have trouble rising for the rest of the day after playing fetch. You examine Manny, BCS 8/9, crepitus in the left stifle, resistant to extension on hips bilaterally, no pain on spinal palpation. You can see that Manny has decreased muscle mass in the hind limbs bilaterally, but there are no neurologic deficits and no abnormalities in the forelimbs or neck. Manny’s owners are limited financially and do not want to pursue the hind limb radiographs, Lyme test or complete blood count and chemistry panel that you recommend. What treatment would you recommend for the condition that Manny is most likely dealing with?

  • Consult with a board-certified orthopedic surgeon for cruciate ligament repair
  • Tramadol 6 mg/kg every 8 hours, long-term
  • Amputation of the left hind leg
  • Carprofen 2.2 mg/kg twice daily, long-term
  • Weight loss, controlled leash walks, omega-3 fatty acid supplementation
A

Answer: Weight loss, controlled leash walks, omega-3 fatty acid supplementation

Explanation
Given that Manny is lame after intense exercise and is an older, overweight, large-breed dog, he is probably dealing with degenerative joint disease (DJD). It is not uncommon for owners to decline diagnostics when they just think their dog is getting old. Unfortunately this makes treating the patient appropriately a bit more difficult.

Ideally you would like to rule out evidence of a primary bone tumor, cruciate ligament tear, and see evidence of DJD on a radiograph. In certain regions of the country Lyme disease is a common cause of lameness, and a simple blood test can screen for exposure to this disease. Prior to starting Manny on long-term medications, you would recommend blood work to make sure there are no underlying diseases that could be worsened from medications like NSAIDs. Management of DJD or osteoarthritis consists of weight management, controlled exercise, and neutraceuticals that decrease inflammation such as glucosamine/chondroitin, Omega-3 fatty acid supplementation or diets that contain these nutrients. Some patients will show dramatic improvements with these simple changes, saving the pain medications for later. Physical therapy and swimming can also be helpful to strengthen the muscles, but punishing activities like sprinting, jumping and rough play should be avoided.

176
Q

A 2-year old Terrier mix presents for an alopecic, raised nodule on its head. The owner describes previous similar lesions that have ulcerated and regressed over time. What is your most likely diagnosis?

  • Hemangiopericytoma
  • Melanoma
  • Cutaneous histiocytoma
  • Mast cell tumor
  • Cutaneous hemangiosarcoma
A

Answer: Cutaneous histiocytoma

Explanation
The correct answer is cutaneous histiocytoma. Cutaneous histiocytomas in dogs occurs early in life at less than 3 years of age. The lesions are alopecic, raised, nodules that often ulcerate and regress on their own. Treatment is not needed unless the lesions persist. Treatment for persistent lesions is surgery. The other tumor types do not regress, although mast cell tumors may grow and shrink over time.

177
Q

A 4-week old intact male Gordon Setter presents for an acutely swollen face and lethargy. On physical examination you note pronounced submandibular lymphadenopathy, and occasional pustules on the face in addition to the swelling. Cytology does not reveal an infectious cause. Which of the following is an appropriate treatment?

  • Anti-inflammatory doses of glucocorticoids
  • Cephalexin
  • Immunosuppressive doses of glucocorticoids
  • Ketoconazole
A

Answer: Immunosuppressive doses of glucocorticoids

Explanation
This is a classic clinical presentation for juvenile cellulitis (puppy strangles). This disease is characterized by sterile granulomas and pustules that respond dramatically to glucocorticoids. The cause is thought to be an underlying immune dysfunction.

Immunosuppressive glucocorticoid therapy should not be started until other infectious diseases have been ruled out. At a minimum a deep skin scrape, skin cytology, and trichogram should be performed to rule out Demodex, bacteria, and fungal infection.

178
Q

A 3-year old West Highland White Terrier presents for flea allergic dermatitis. The owner is concerned because she is on an appropriate adulticide and insect growth regulator, but live fleas are still being found on the patient. Additionally, the dog also has moderate signs of flea allergy dermatitis; papular/pustular dermatitis in the lumbosacral region. Why might the treatment be failing?

  • Owner is not applying the product properly and is not treating other pets in the household
  • Owner is not applying the product properly
  • Owner is not treating the environment
  • Owner is not applying the product properly and not treating the environment
  • Owner is not applying the product properly, owner is not treating the environment, and other pets in the household are not on flea control
  • Other pets in the household are not on flea control
A

Answer: Owner is not applying the product properly, owner is not treating the environment, and other pets in the household are not on flea control

Explanation
This question is a review of the most common reasons for failure of a flea product to work.
Education is extremely important when starting flea treatment. Always question how the owner applies the product, whether the environment is being treated, and make sure all pets in the household are undergoing treatment. One is often surprised how many owners apply topical products just to the hair of their animals. Many owners (and veterinarians) blame lack of response to resistance against flea adulticides; however, it is much more likely (in the majority of cases) that it is due to one of these 3 listed reasons.

PowerPage” on this topic, view [por Flea Allergy Dermatitis

179
Q

A 6-week old mixed breed dog acquired from a shelter presents for diarrhea, increased respiratory effort, and seizures. You suspect the puppy has distemper. What is the definitive diagnostic test for canine distemper?

  • The combination of gastrointestinal, respiratory, and neurologic clinical signs
  • Hyperkeratosis of foot pads
  • Enamel hypoplasia
  • Immunofluorescent assay on affected epithelium
  • Detection of virus via serology
A

Answer: Immunofluorescent assay on affected epithelium

Explanation
The correct answer is immunofluorescent assay on affected epithelium. Although the clinical signs are supportive for distemper, the definitive diagnosis is examination of blood or tissue by immunofluorescent assay. This can be performed on tracheal, vaginal, respiratory, or other epithelium. These samples may be negative if the dog is only showing neurologic signs or if a circulating antibody is present. The diagnosis can then be made by serology for distemper IgM or increased CSF to serum virus specific IgG. In addition, a distemper PCR test is now available and is performed on swabs of ocular or nasal discharge.

The test can have false positives in some vaccinated animals; however, animals showing signs consistent with distemper that are PCR positive should be considered likely infected with distemper.

180
Q

A 4-year old dog presents for excessive licking of the right carpus of a 2-month duration. Upon examination you note an oval wound that is thickened, alopecic, and ulcerated. Based on clinical findings you make a presumptive diagnosis of acral lick dermatitis. Which statement is NOT true for acral lick dermatitis?

  • The lesion should resolve with topical antibiotic treatment
  • Arthritis may be contributing to the lesion
  • Allergies may be contributing to the lesion
    50% of cases are diagnosed as idiopathic or psychogenic
  • These lesions are frustrating to treat because an underlying cause can be hard to find
A

Answer: The lesion should resolve with topical antibiotic treatment

Explanation
Most acral lick lesions are solitary and affect the carpus or metacarpus. About 50% will be diagnosed as idiopathic or psychogenic, but other diseases must be ruled out first including bacteria, fungal, demodicosis, trauma, allergy, and underlying joint disease. A full work-up may include skin cytology, fungal culture, biopsy, diet trial, skin testing, radiographs of the joint, and addressing behavioral disorders.

Most of these lesions will have a deep bacterial infection and treatment should include systemic antibiotics based on a deep culture sample. Topical antibiotics may not penetrate the area well enough to resolve the infection.
Management also includes treating the underlying cause. Many veterinarians will also prescribe topical or intralesional steroids. If no cause is found, then antidepressants or anxiolytics may be started.

These lesions will rarely go away unless the underlying cause is identified and properly treated.

181
Q

Nina, a 3 year-old female spayed German Shepherd dog, was presented to your hospital for oral bleeding.

Unfortunately Nina is not a big fan of the veterinary hospital; it is extremely challenging to examine her mouth through the muzzle she needs to wear during your examination. As you lift Nina’s lip with a tongue depressor, you quickly find the source of the bleeding;
Nina has fractured her right maxillary canine. When extracting a tooth, what structure needs to be broken down in order to loosen the tooth?

  • Cementum
  • Periodontal ligament
  • Pulp chamber
  • Alveolar bone
  • Dentin
A

Answer: Periodontal ligament

Explanation
The periodontal ligament is made of collagen fibers and creates a strong connection between the root of the tooth and the alveolar bone. It can be extremely difficult to breakdown during a healthy tooth extraction, as in the case of a complicated crown root fracture.

182
Q

A 2 year old male dog presents after being hit by a car. You take thoracic radiographs to look for evidence of trauma. Which of these findings would not occur from trauma?

  • Pulmonary contusions
  • Rib fractures
  • Pleuroperitoneal diaphragmatic hernia
  • Peritoneopericardial diaphragmatic hernia
A

Answer: Peritoneopericardial diaphragmatic hernia

Explanation
The answer is peritoneopericardial diaphragmatic hernia. Pleuroperitoneal diaphragmatic hernias or what most of us think of as regular diaphragmatic hernias can be congenital or traumatic in origin. Trauma can cause an abrupt increase in intra-abdominal pressure forcing the lungs to rapidly deflate causing a large pleuroperitoneal pressure gradient. This pressure gradient can cause the diaphragm to tear at its weakest point, typically the muscular portions. Peritoneopericardial hernias are always congenital in dogs and cats (interestingly, they can be caused by trauma in humans). Hopefully, rib fractures and pulmonary contusions were obvious.

***PowerPage: Thoracic Trauma

183
Q

A 2-year old female spayed Miniature Poodle presents for a focal area of alopecia (5cm in diameter) on the right caudal-lateral thigh. The area is also hyperpigmented. The owner explains that the original lesion was a nodule, but over the last couple of months the hair started to fall out and the skin turned dark. The owner reports that otherwise the dog has been doing well. You ask if any topical medications or treatments have been done on or near the area.

The owner reports that she has not done any treatments. The only medications the dog has received are her yearly vaccines (4 months ago) and heartworm preventative. Based on the history and physical exam findings, what are you most concerned about and what diagnostic test would you like to perform?

  • You are concerned about an allergic reaction; food trial
  • You are concerned about a vaccine reaction; biopsy
  • You are concerned about an autoimmune disease; biopsy
  • You are concerned about an endocrinopathy; thyroid panel
A

Answer: You are concerned about a vaccine reaction; biopsy

Explanation
The lesion is highly suggestive of a vaccine reaction; particularly post-rabies vaccination panniculitis (inflammation of the sub-cutaneous) due to location and description of the lesion. Rabies vaccine is often given in the right hind limb.

Initially, this condition presents with a nodule where the vaccine occurred and then complete alopecia occurs 1-3 months afterward. Many times there is a long delay between vaccination and clinical signs (up to 6 months).

Lesions usually resolve on their own, but use of pentoxifylline or prednisone can be effective. Definitive diagnosis would require a skin biopsy.

An allergic reaction or endocrinopathy is less likely since the lesion is focal. Endocrinopathies usually result in bilaterally symmetrical alopecia. Hair loss due to allergies is usually a result of pruritus, which was not mentioned in the patient’s history.

This would be an atypical location and lesion for an autoimmune condition.

184
Q

A 7-year old, male, castrated shepherd mix presents for polyuria and polydipsia. The physical exam reveals a body condition score of 4/9 and 5% dehydration. The urine specific gravity is 1.006. The urinalysis, chemistry panel, and CBC are otherwise unremarkable. Which of the following is the most likely diagnosis?

  • Fanconi syndrome
  • Central diabetes insipidus
  • Diabetes mellitus
  • Chronic renal failure
A

Answer: Central diabetes insipidus

Explanation
The correct answer is central diabetes insipidus.

Diabetes mellitus is ruled out with a normal blood glucose on the chemistry panel and lack of glucose in the urine on the urinalysis. Chronic renal failure is ruled out because there is no azotemia on the chemistry panel.

Fanconi syndrome usually occurs in Basenjis and is ruled out because there is no glucose in the urine.

***PowerPage: Polyuria and Polydipsia

185
Q

What causes cutaneous larval migration in humans?

  • Echinococcus
  • Dipylidium
  • Ancylostoma
  • Toxocara
A

Answer: Ancylostoma

Explanation
The correct answer is Ancylostoma. These are hookworms; when they come in contact with unprotected skin, the infective larvae penetrate the epidermis but generally cannot penetrate the basement membrane.

They therefore migrate aimlessly, and the disease is usually self-limiting in humans.